Você está na página 1de 84

APROVADO DE A A Z

TURMA IME ITA


MATEMÁTICA ZERO Se n + n2 + 8 + 3 n − n2 + 8 = 8 , onde n é um número
3

inteiro, então o valor de n é igual a:


(A) 1 (B) −1 (C) 8
(D) 232 (E) 280
01
O número de triângulos de perímetro igual a 180 cujas
medidas dos lados são expressas por números inteiros é: 07
(A) 671
(D) 677
(B) 673
(E) 679
(C) 675
O valor de
(2
3
− 1)  ( 33 − 1)  (1003 − 1)
é:
(2
3
+ 1)  ( 33 + 1)  (1003 + 1)
02
3361 3363 3367
O triângulo ABC é isósceles com B = C e A = 20 . (A) (B) (C)
o
5050 5050 5050
Os pontos D e E são tomados em AC e AB
3369 3371
respectivamente de modo que CBD = 50 e BCE = 20 .
o o
(D) (E)
5050 5050
A medida do ângulo BDE é igual a:
o o o
(A) 10 (B) 20 (C) 30
o o
08
(D) 50 (E) 60
Seja P um ponto do interior de um triângulo isósceles ABC tal
que AB=BC, ABC = 80 , PAC = 40 e ACP = 30 . A
o o o
03
medida do ângulo BPC é igual a:
Se x = 8 + 2 10 + 2 5 + 8 − 2 10 + 2 5 então x : (A) 90o (B) 100o (C) 110o
(D) 120o (E) 130o
(A) 10 + 2 (B) 2 5 + 2 (C) 4
(D) 2 5 − 2 (E) 10 − 2 09
Observe que o número 399 quando escrito na base 5 , ele se
04 torna 3044 . A soma dos seus algarismos (abreviadamente
Considere as afirmativas: "soma digital") é igual a 11 . Com base nisto, certos números
1. A "soma digital" isto é, a soma dos algarismos do número quando escritos na base 4 , possuem soma digital igual a 17 .
N = ( 212211222111222211112222211111)3 quando este é Se k é a soma digital de um destes tais números quando
escrito na base 9 é igual a 69. escritos na base 2 , a soma dos valores mínimo e máximo de k
2. Se a representação binária de um número a é dada por é igual a:
1110000100111010101110100001 , o sétimo algarismo da (A) 20 (B) 22 (C) 24
direita para a esquerda da representação de a na base 8 é (D) 26 (E) 28
igual a 5 .
3. O número de 1's necessários para listarmos, na notação 10
A soma de todos os valores de n para os quais n − 19n + 99
2
binária, os inteiros de 0 a 1023 é 5120 .
4. A soma digital de um número a isto é, a soma dos seus é um quadrado perfeito é igual a:
dígitos, quando escrito na base 4 é igual a 17 escrevendo-se (A) 30 (B) 32 (C) 34
este número na base 2 , o valor mínimo da sua soma digital é (D) 36 (E) 38
9.
Conclua que: 11
(A) (2) e (3) estão certas. Colocando-se o número
(B) (1) está certa e (3) errada.  4 1  4 1  4 1   4 1 
(C) (1) está certa e (4) errada 1 +  3 +  5 +    11 + 
(D) todas estão erradas
 4  4  4  4
 4 1  4 1  4 1   4 1 
(E) (2) está errada e (4) está certa.
 2 +  4 +  6 +    12 + 
 4  4  4  4
05 p
sob a forma da fração irredutível , o valor de p + q é:
Sejam a e b números reais com a  1 e b  0 . Se ab = a e
b
q
a (A) 312 (B) 314 (C) 316
= a 3b , determine então o valor de b − a é igual a:
b (D) 318 (E) 320
(A) 9 (B) 16 (C) 27
(D) 64 (E) 81 12
Em um quadrado ABCD de lado unitário tomam-se os
06 pontos P e Q sobre os lados AB e AD respectivamente, de

MATEMÁTICA – Prof. DOUGLAS OLIVEIRA biblioteca


telegram t.me/bibliotecaPE @PROFDOUGLASOLIVEIRA _2020 1
APROVADO DE A A Z
TURMA IME ITA
modo que o perímetro do triângulo APQ seja igual a 2 . A
medida do ângulo PCQ é igual a: 19

(A) 30
o
(B) 45
o
(C) 60
o Sejam x1 , x2 , x3 , números que podem ser escritos como
o o uma soma de uma ou mais potências distintas de 3 com
(D) 75 (E) 90
x1  x2  x3  . Por exemplo, x1 = 30 = 1 , x2 = 31 = 3 ,
x3 = 30 + 31 = 4 . A soma dos algarismos de x2009 é igual a:
(A) 30 (B) 31 (C) 32
(D) 33 (E) 34

13
20
Seja ABC um triângulo isósceles com AC = BC e 40
Considerando todos os inteiros entre 1 e 2 cuja
ACB = 106o . Um ponto M do seu interior é tal que representação binária possui exatamente dois 1' s , o número
MAC = 7o e MCA = 23o . A medida do ângulo CMB daqueles que são divisíveis por 9 é igual a:
é igual a: (A) 130 (B) 131 (C) 132
(A) 81o (B) 83o (C) 85o (D) 133 (E) 134
(D) 87o (E) 89o

14 21

Se 1989 = 13 e 1989 = 17 , então o valor de 117


a b
1− a − b
2(1− b )
é
O valor do inteiro a tal que
3 3
2 −1 =
1
3
a
(
1− 3 2 + 3 4 é )
igual a: igual a:
(A) 13 (B) 3 (C) 17 (A) 2
(B) 4
(D) 13 (E) 3 13
(C) 6
(D) 8
15 (E) 9
Em um triângulo ABC , no qual BAC = 1200 traçam-se as
bissetrizes AF , BG e CH . A medida do ângulo GFH é 22
igual a: Para todo conjunto S , seja S o número de elementos de S , e
(A) 45o (B) 60o (C) 90o
(D) 100 o
(E) 120 o seja n ( S ) o número de subconjuntos de S . Se A , B , C são

( ) ( )
conjuntos tais que: n A + n B + n C = n A ( ) ( B C) e
16
A = B = 100 , então o valor mínimo possível para
Para k  0 , seja I k = 100 064 , onde existem k zeros entre
A B C é igual a:
1 e 6 . Seja N ( k ) o número de fatores iguais a 2 na
(A) 96
decomposição em fatores primos de I k . Qual o valor máximo (B) 97

de N k ?( ) (C) 98
(D) 99
(A) 6 (B) 7 (C) 8 (E) 100
(D) 9 (E) 10
23
17 Seja o heptágono regular ABCDEFG de centro P ( −5,8 ) e
A representação decimal de um natural a possui n vértice A ( 2,6 ) , pode-se afirmar que a soma das coordenadas
3
algarismos enquanto que a representação decimal de a
dos vértices é:
possui m algarismos. Dentre os valores abaixo, aquele que
a) 7.
não pode ser possível para m + n é: b) 14.
(A) 2006 (B) 2007 (C) 2008 c) 21.
(D) 2009 (E) 2010 d) 28.
e) 35.
18
1 1 24
Se x + = 5 o valor de x5 + 5 é igual a:
x x
(A) 3125 (B) 2525 (C) 2125 Sejam AOB e BOCdois ângulos adjacentes. Se OX, OY
(D) 2025 (E) 625 e OZ são respectivamente as bissetrizes de AOB, BOCe

MATEMÁTICA – Prof. DOUGLAS OLIVEIRA biblioteca


telegram t.me/bibliotecaPE @PROFDOUGLASOLIVEIRA _2020 2
APROVADO DE A A Z
TURMA IME ITA
XOC, sabe-se que AOC= 100º e ZOA= 55º . A (IME) Dados dois pontos do plano complexo, z1 = 2 + 3i e
medida do ângulo AOB é: z2 = 4 + 5i, determine e esboce o L.G. dos pontos que
 z − z1 
(A) 40 º satisfazem a relação Re  = 0 , com z1  z2
(B) 60 º  z − z2 
(C) 75º 08
(D) 80 º
(IME) Seja o conjunto A = {z  C| | z | = 1}. Determine a
(E) 90 º
imagem de A pela função g, complexa de variável complexa,
tal que g(z) = (4 + 3i).z + 5 – i.

09
NÚMEROS COMPLEXOS (IME-72/73) Determine as raízes primitivas de índice 24
da unidade.
01
(IME-68/69) Sejam:
i) A e B números reais,
ii) N e k, inteiros, maiores que zero.
iii) Para cda n, seja a raiz principal(menor
determinação) de índice n do número 10
. (IME-73/74) Determine o conjunto dos pontos z do plano
z+2
complexo tais que representa um número real.
Admitamos que: . z ( z + 1)
Determinar o valor de n de tal forma que A/B seja mínimo. 11
Obs: (IME-74/75) Considere o conjunto dos números reais  e o
conjunto dos números complexos C. Sabendo que a  , b 
02 , z1  C, z2  C e que z12 + az12 + b = 0 , e
(IME) Os complexos a, b, c têm como imagens os vértices
de um triângulo equilátero. Calcule
z22 + az22 + b = 0 , determine a relação r = a2/b para que
a2 + b2 + c2 – bc – ca – ab. os pontos z1, z2 e z0 (0, 0) no plano complexo formem um
triângulo equilátero, esboçando as soluções no plano
03 complexo.

(IME) Calcule e  +2i


na froma trigonométrica e na forma 12
algébrica, sabendo que  é a ordenada do ponto onde a (IME-74/75) São dados dois números complexos z1 e z2. As
curva y = f(x) corta o eixo dos y, sendo f(x) um polinômio de partes real e imaginária de um complexo são dadas por
terceiro grau que passa por um mínimo igual a 2 para x = 1 e Re(z) e Im(z). Determine z1 e z2, sabendo que:
cujo resto da divisão por x2 + 3x + 2 é – x + 3.
z1 + z 2 = 5
 2
4z1 + z 2 + 15[Re( z 2 )] = 0
2
04
(IME) A parte real de um número complexo é x2 – 2 e a Re( z ) = 4[Re( z )]
 2 1
parte imaginária x 2 . Determine o valor mínimo do
módulo desse complexo.
13
(IME-75/76) Considere três números complexos z0, z1 e z2.
05
(IME) Determine os valores máximos e mínimos de |z – 4|, Sabendo que z0  z1  z2 e que z 03 = z13 = z 23 = 1 , calcule
sabendo-se que |z + 3i|  1. z 02 + z12 + z 22 .
06
14
(IME) Determine n natural para que (x + k)n – xn – kn = 0,
(IME-76/77) Seja f: C → C
2i z → iz + 2 + 3i
onde k é um real diferente de zero e x = ke 3
. Seja o conjunto A = {x + iy  C |x2/9 + y2/4 = 1}. Determine
o conjunto B imagem de A pela função f.
07

15

MATEMÁTICA – Prof. DOUGLAS OLIVEIRA biblioteca


telegram t.me/bibliotecaPE @PROFDOUGLASOLIVEIRA _2020 3
APROVADO DE A A Z
TURMA IME ITA
(IME-76/77) Seja z = a + bi (a, b  R). Determine a e b
tais que z2 = 3 - 4i. 24
(IME-83/84) Sejam C uma constante real positiva e z um
16 número complexo. Determine os dois lugares geométricos
(IME-77/78) Sendo H = {z  C / z8 = 1} e G = {z  C / z12 = z +1
que satisfazem a equação: =C
1} e S = G  H, determine todos os pares (am, bm)  R2 tais z −1
que am + bmi  S.

25
17
(IME-84/85) Sejam z1 e z2 complexos de raios vetores OP1
(IME-78/79) Seja g: C → C a função definida por: g(x + yi)
e OP2, respectivamente. Mostre que OP1 e OP2 são
= i(x + yi) + 2 + 3i. Dada a elipse E = {x + iy |x2/9 + y2/4 = 1;
perpendiculares se e somente se z1z 2 é um imaginário puro.
x, y  }. Determine sua imagem g(E) pela função g.

18 26
(IME-79/80) Um velho manuscrito descrevia a localização (IME-85/86) Sabendo-se que x é um número real, –1  x 
de um tesouro enterrado: “Há somente duas árvores, A e B, 1, 0  arc cos x   e n é um número inteiro positivo, mostre
em um terreno plano, e um canteiro de tomates. A é uma que a expressão
mangueira, e B é uma jaboticabeira. A partir do centro k do fn (x) = cos (n arc cos x)
canteiro, meça a distância em linha reta até a mangueira.
Vire 90o à esquerda e percorra a mesma distância até o
ponto C. Volte ao canteiro. Meça a distância em linha reta
até a jaboticabeira. Vire 90o à direita e percorra a mesma
distância até o ponto D. O tesouro está no ponto médio T do pode ser desenvolvida como polinômio em x, de grau n, cujo
segmento CD”. Um aventureiro achou o manuscrito, coeficiente do termo de maior grau é igual a 2n – 1
identificou as árvores, mas como o canteiro desapareceria
com o passar do tempo, não conseguiu localizá-lo, e desistiu 27
da busca. O aluno Sá Bido, do IME, nas mesmas condições, (IME) Dois números complexos Z1 e Z2, não nulos, são tais
diz que seria capaz de localizar o tesouro. Mostre que você que , mostre que é imaginário
resolveria o problema, isto é, dê as coordenadas de T em
função das coordenadas de A = (5, 3) e B = (8, 2). puro.

19 28
(IME-80/81) Calcule i . i (IME-87/88) Seja z um número complexo. Mostre que
1
20
z+ é um número real se e somente se z é um número
z
(IME) Seja . Diz-se que um ponto h é um ponto de real ou z = 1.
Hurwitz se e para todo número natural n,
. Prove que o ponto é um ponto de 29
Hurwitz. (IME-88/89) Sejam z e w números complexos tais que |z|
Obs: . z−w
= 1 e |w|  1. Calcule .
1 − wz
21
(IME-81/82) Determine o número complexo z de menor
30
argumento tal que |z – 30i|  15.
(IME-88/89) Mostre que todas as raízes da equação (z +
1)5 + z5 = 0 pertencem a mesma reta paralela ao eixo
22
imaginário.
1
(IME-82/83) É dado 2 cos = + x ; demonstre que
x 31
1 (IME-89/90) Resolva a equação: (z + i)3 – (z – i)3 = 0, onde
2 cos m = m + x m
x z = x + yi, x, y   e i = −1 .
23 32
(IME-83/84) Quais as relações entre os coeficientes a, b,
c, d da equação x2 + 2(a + ib)x + c + id = 0 de modo que ela
(IME-89/90) Resolva a equação: z 5 = z , onde z éo
conjugado do número complexo z.
seja satisfeita para um valor real x = k?

MATEMÁTICA – Prof. DOUGLAS OLIVEIRA biblioteca


telegram t.me/bibliotecaPE @PROFDOUGLASOLIVEIRA _2020 4
APROVADO DE A A Z
TURMA IME ITA
 C, a rotação R em torno do ponto c e amplitude igual a /3
33 é dada por R(z) = – j2z – jc,  z  C – {c} pede-se: a)
(IME-90/91) Dado um número complexo z com |z| = 1 e z  Determinar as relações existentes entre a, b, c, j, j2, onde
1 + t .i a, b  C, de modo que o triângulo a, b, c seja equilátero; b)
1, mostre que existe um número real t tal que z= Determinar z para que o triângulo i, z, iz seja equilátero.
1 − t .i
43
34
(IME-97/98) Determine os parâmetros , ,  e  da
(IME-91/92) Determine a forma polar do número complexo
Z + 
dado: (1 + cos  + i.sen )n, n  Z*,   . transformação complexa W= , que leva os
Z + 
35 pontos Z = 0 : –i : –1 para W = i : 1 : 0, respectivamente,
(IME-91/92) Prove que z1 + z2 = z1 + z2 , onde z1, z2  bem como Z para W = –2 – i, onde i = −1 .
C.
44
36 (IME-98/99) Represente no plano xy o conjunto de valores
(IME-92/93) Faça o que se pede: a) Calcule o argumento de z = x + y i , onde i = −1 , que satisfazem à equação
do seguinte número complexo i(1 + i); b) Escreva sob forma | z – 1 – i | < 1.
trigonométrica o número complexo z = 1 + i 3.
37
(IME-92/93) Calcule o módulo e o argumento do número
9
    
complexo Z = 3 2  cos + i.sen  
  36 36  
45
(IME-98/99) Determine as raízes de z2 + 2iz + 2 – 4i = 0
38
(IME-93/94) Sendo c = a + b.i, um número complexo e localize-as no plano complexo, sendo i = −1 .
conhecido, e z = x + y.i, um número complexo a determinar,
ambos não nulos, tais que c.z = b + a.i, determine z. 46
(IME-2005/2006) Sejam as somas S0 e S1 definidas por
39
(IME-93/94) Considere os números complexos z = x + y.i e
3
|w|.
w = y – x.i, cujos módulos são tais que | z| = e x
,e Calcule os valores de S0 e S1 em função de n, sabendo que
[r] representa o maior inteiro menor ou igual ao número r.
1
|z |. Obs: Utilize o desenvolvimento em binômio de Newton de
| w| = e y
, onde e é a base dos logaritmos neperianos.
Obter a forma polar de z2.

40

(IME-94/95) Sabendo que −1 , calcule 1 + 2i + 3i2 + 4i3 MATRIZES


+ ... + 21i20

41 01
1 (Interno-ITA) Sendo A, B matrizes nxn, mostre que:
(IME-94/95) Dado Z= , calcule as partes a) (AB)t = BtAt b) A + At é simétrica
−7 + 24. i c) AAt é simétrica d) A – At é anti-simétrica
real e imaginária de Z. e) (A2)t = (At)2 f) (A3)t = (At)3

02
42
(Interno-ITA) Mostre que se a terceira linha de uma matriz mxn
(IME-95/96) Sejam w0 = 1, w1 = j, w2 = j2 as raízes cúbicas
A é quatro vezes a primeira linha, então a terceira linha de AB é
da unidade no plano complexo (considere w1 o número também igual a quatro vezes a primeira linha, sendo B uma
complexo de módulo 1 e argumento 2/3). Sabendo que se c matriz nxp.

MATEMÁTICA – Prof. DOUGLAS OLIVEIRA biblioteca


telegram t.me/bibliotecaPE @PROFDOUGLASOLIVEIRA _2020 5
APROVADO DE A A Z
TURMA IME ITA
03 15
(Interno-ITA) Mostre que se (Interno-ITA) O traço de uma matriz quadrada, representado por
 a11 a12   b1   c11 c12   b1  tr A, é a soma de seus elementos sobre a diagonal, isto é,
a   =     ,  b1, b2  ,
n

 21 a 22  b2  c 21 c22  b2  tr A = a


i =1
ii . Mostre que:

 a11 a12   c11 c12  a) Se k  , tr (k.A) = k.tr A, b) tr (A  B) = tr A  tr B,


então  = . c) tr AB = tr BA, d) tr (B– 1AB) = tr A,
a 21 a 22  c 21 c22  n n

04
e) tr (AAt) =  (a
i =1 j =1
ij )2 .
(Interno-ITA) Uma matriz anti-simétrica é definida como sendo
uma matriz tal que At = – A. Mostre que uma matriz anti- 16
simétrica é quadrada, e que os elementos da diagonal são nulos. (Interno-ITA) Uma matriz real nxn A que satisfaz as relações
AAt = AtA = I é chamada ortogonal.
05 a) Dê exemplo de uma matriz ortogonal 2x2, distinta da
(Interno-ITA) Mostre que toda matriz quadrada nxn pode ser identidade.
decomposta de maneira única na soma de uma matriz simétrica e b) Ache a matriz ortogonal geral 2x2.
de matriz anti-simétrica, isto é, dada A matriz nxn existem únicas c) Mostre que o produto de duas matrizes ortogonais é uma
matrizes nxn, B e C, tais que Bt = B, Ct = – C e A = B + C. matriz ortogonal.
d) Mostre que a inversa de uma matriz ortogonal é uma matriz
06 ortogonal.

(Interno-ITA) Dizemos que uma matriz quadrada é não-singular 17


se possui inversa. Sejam A e B matrizes nxn não-singulares, Prove que A2 é simétrica quer A seja simétrica quer seja A anti-
mostre que: a) (A– 1)– 1 = A b) (AB)– 1 = B– 1A– 1 simétrica.

07 18
(Interno-ITA) Achar todas as matrizes X 2x2 tais que Se A e B são ambas simétricas, prove que:
X2 = I, onde I é a matriz identidade 2x2. a) A + B é simétrica.
b) AB é simétrica se, e somente se, A e B comutam.
08
(Interno-ITA) Uma matriz nxn A é nilpotente se Ar = O para 19
algum inteiro positivo r. Dê exemplo de uma matriz não-nula 2x2 Resolver o sistema:
nilpotente.
 X + Y = 3A 2 0 1 5
 onde A=  e B= 
09  X − Y = 2B 0 4 3 0
(Interno-ITA) Uma matriz quadrada é idempotente se A2 = A.
Dê exemplo de uma matriz idempotente diferente da matriz nula 20
e da matriz identidade.
Determinar x e y de modo que as matrizes
10 1 2 0 1 
(Interno-ITA) Construa matrizes A e B, 2x2, sem coeficientes
A=  e B=  comutem.
nulos, e tais que AB = O. 1 0 x y

11
(Interno-ITA) Ache duas matrizes 2x2, X e Y, tais que nenhuma
21
delas seja a matriz nula, e tais que X2 + Y2 = O.
1 − 1
12 Obter todas as matrizes B que comutam com A= 
(Interno-ITA) Que matrizes reais satisfazem AtA = O? 3 0 
13 22
(Interno-ITA) Suponha que A é uma matriz 2x2 que comuta com Provar que se A e B são matrizes comutáveis, então valem as
qualquer matriz 2x2. Mostre que A deve ser múltiplo da matriz seguintes igualdades:
identidade. a) (A + B)(A – B) = A2 – B2
b) (A + B)2 = A2 + 2AB + B2
14 c) (A – B)2 = A2 – 2AB + B2
(Interno-ITA) Mostre que (At)– 1 = (A– 1)t, logo conclua que se A d) (A + B)3 = A3 + 3A2B + 3AB2 + B3
é simétrica então A– 1 também o é, sendo A uma matriz quadrada e) (A – B)3 = A3 – 3A2B + 3AB2 – B3
inversível. f) (AB)n = AnBn

MATEMÁTICA – Prof. DOUGLAS OLIVEIRA biblioteca


telegram t.me/bibliotecaPE @PROFDOUGLASOLIVEIRA _2020 6
APROVADO DE A A Z
TURMA IME ITA
24 C D F G
A= e B= , onde C, D, E, F e G são
Calcular as matrizes X, quadradas de ordem 2, tais que X2 = O.  F E 
E C 
25 matrizes 2x2.
Calcular as matrizes X, quadradas de ordem 2, tais que X2 = X.
36
26 1 − 2 2 
Sabendo que A, B e C são matrizes quadradas de ordem n Mostre que A3 – 9A + 10I = 0 quando A = 0 2 0 
inversíveis e AXB = C, prove que X = A– 1CB– 1 
1 − 1 − 3
27
Sendo A e B matrizes inversíveis de ordem n, isolar X a partir de
cada equação abaixo: 37
a) AX = B d) BAX = A A matriz quadrada A diz-se nilpotente se Ap = 0 para alguns
b) AXB = In e) (AX)t = B inteiros positivos p. Se p for o menor inteiro positivo para o qual
c) (AX)– 1 = B f) (A + X)t = B Ap = 0, então A diz-se nilpotente de índice p. Mostre que
1 5 − 2
28
Se A, B, C e X são matrizes do mesmo tipo, inversíveis e A = 1 2 − 1  é nilpotente de índice 3.
AX(B + C)– 1 = B, prove que: X = A– 1B(B + C) 3 6 − 3
29
38
Demonstrar que se A e B são matrizes 2x2 inversíveis então
(ABA– 1)– 1 = AB– 1A– 1. Uma matriz quadrada diz-se idempotente se A2 = A. Prove que, se
A for idempotente, então também o é I – A.
30
39
n
1 1 1 n Prove que a inversa de uma matriz triangular inferior A com
Prove que = elementos com os elementos da diagonal diferentes de zero é ela
0 1 0 1
própria uma matriz triangular inferior.

31 40
cos  − sin  cos n − sin n
n
Uma matriz A é congruente com uma matriz B com a mesma
Prove que = ordem se existir uma matriz real P não-singular tal que A =
sin  cos  sin n cos n PBPT.
a) Mostre que se A é congruente com B e B é congruente com C
então A é congruente relativamente a C.
32
b) Mostre que se A é congruente com B, então B é congruente
Determine X onde: com A.
2 1  − 3 2  − 2 4 
3 2. X . 5 − 3 =  3 − 1
     

41
Seja Ak = O. Prove que: (I – A)– 1 = I + A + A2 + … + Ak – 1 .
33
 0 2x 1  42

Determinar x para que a matriz 


A= x 2
0 − 4 x 
1 0 0
( x + 1) x 3 0 
Achar a potência enésima da matriz: A = 1 1 0
seja anti-simética.
1 1 1
34
Prove que se AB = BA, então A– 1B = BA– 1 43
Prove que a solução do sistema de equações matriciais
35
 AX + BY = C
Uma matriz diz-se fragmentada se for dividida em matrizes mais

pequenas (submatrizes) por traços horizontais ou verticais
traçados entre linhas e colunas inteiras. As operações aritméticas
 BX + CY = A
−1
é dada por Y = ( B − AC ) .( BC − A )
2 2
definidas para as matrizes contendo elementos escalares aplicam-
se também às matrizes fragmentadas. Determine AB se
X = C − B( B 2 − AC ) −1 ( BC − A 2 )

MATEMÁTICA – Prof. DOUGLAS OLIVEIRA biblioteca


telegram t.me/bibliotecaPE @PROFDOUGLASOLIVEIRA _2020 7
APROVADO DE A A Z
TURMA IME ITA
Em uma matriz B = ( bij ) m n , um elemento bij é
44
denominado ponto de sela caso satisfaça a uma das condições:
a c
Prove que toda matriz de segunda ordem A= satisfaz 1) bij é o maior elemento da linha i e o menor da coluna j.
b d
2) bij é o menor elemento da linha i e o maior da coluna j.
a equação X2 – (a + d)X + (ad – bc)I = 0.
De acordo com esta definição, na matriz
45  2 10 3 1
M e N são matrizes distintas nxn satisfazendo M3 = N3 e M2N = − 5 − 3 − 4 − 7 
N2M. Prove que M2 + N2 não é inversível. A=  
− 1 − 2 0 99 
46  
 15 5 −3 6
1 m  o ponto de sela é
(UNIFOR-99) Seja a matriz A =   , com

0 1  a) a 34 b) a 22 c) a 24 d) a 43 e) a 33
m  0. A matriz A10 é igual a:
52

47
(UFPB-99) Considere a matriz ( )
A = aij 2  2 tal que
(UFC-99) Dê exemplo de três matrizes 2x2, A, B e C tais que: a
sen(  i ), se i = j
matriz A não seja nula, A.B = A.C e B  C.
aij =  2
48
cos(  i ), se i  j
(UFPB-94) Uma matriz n x n representa um quadrado mágico de A transposta da matriz A2 é igual a
ordem n quando satisfaz, simultaneamente, as seguintes  0 2  0 1  0 1
condições: a) − 2 − 2  b) − 1 1 c) − 1 − 1 
I. os elementos da matriz são números inteiros de 1 até n2, sem     
repetição.
II. a soma dos elementos de qualquer linha ou coluna ou diagonal  2 − 2  0 − 1 
da matriz é igual a n(n2 + 1)/2
d) − 2 2 
e) 
1
Das matrizes abaixo, a que representa um quadrado mágico de  1
ordem 3 é
6 2 7  8 1 6  5 5 5
a) 8 4 3 b) 3 5 7  c) 5 5 5
     
5 9 1 4 9 2 5 5 5
1 4 7  2 3 1 
d) 2  e) 1 2 2 53
 5 8  
3 6 9  3 1 3 (UFRJ-92) Uma confecção vai fabricar 3 tipos de roupa
utilizando materiais diferentes. Considere a matriz A = (aij)
abaixo, onde aij representa quantas unidades do material j serão
empregadas para fabricar uma roupa do tipo i.
49
(UFPB-95) O traço de uma matriz quadrada é definido pela soma 5 0 2 
dos elementos de sua diagonal principal. Se A= aij ( )
, onde
33
A = 0 1 3
aij = i cos( j ) , então o traço de A é igual a: 4 2 1
a) – 2 b) – 1 c) 0 d) 3 e) 5 a) Quantas unidades do material 3 serão empregadas na
confecção de uma roupa do tipo 2?
50 b) Calcule o total do material 1 que será empregado para fabricar
cinco roupas do tipo 1, quatro roupas do tipo 2 e duas roupas do
(UFPB-95) Sejam A, B e C matrizes quadradas de ordem n,
tipo 3.
formadas com números reais, e I a matriz identidade de ordem n.
Das afirmações abaixo:
1 54
I. se A2 = I, então A = A–
(UFRJ-99) Antônio, Bernardo e Cláudio saíram para tomar
II. se AB = AC, então B = C
chope, de bar em bar, tanto no sábado quanto no
III. se AB = I, então BA = I
domingo. As matrizes a seguir resumem quantos chopes
está(ão) correta(s) apenas
cada um consumiu e como a despesa foi dividida:
a) I b) II c) III d) I e III e) I e II

51
(UFPB-99) Considere a seguinte definição:

MATEMÁTICA – Prof. DOUGLAS OLIVEIRA biblioteca


telegram t.me/bibliotecaPE @PROFDOUGLASOLIVEIRA _2020 8
APROVADO DE A A Z
TURMA IME ITA
4 1 4 5 5 3
(Covest-93) Nesta questão A, B, I e O são matrizes 2x2, I é a
matriz identidade, O a matriz nula e det A denota o determinante
S = 0 2 0 e D = 0 3 0 de A. Pode-se afirmar que:
0) Se A3 = O então det A = 0
3 1 5 2 1 3 1) Se A2 = O então A = O
S refere-se às despesas de sábado e D às de domingo. Cada 2) Se A2 = A então A = 1
elemento aij nos dá o número de chopes que i pagou para j, sendo 3) Se AB  BA então A2 – B2  (A + B)(A – B)
Antônio o número 1, Bernardo o número 2 e Cláudio o número 3 4) Se A2 = I então A = I
(aij representa o elemento da linha i, coluna j de cada matriz).
Assim, no sábado Antônio pagou 4 chopes que ele próprio bebeu, 61
1 chope de Bernardo e 4 de Cláudio (primeira linha de S).
(ITA-77) Seja X = 
 1 m  uma matriz quadrada 2x2 onde m
a) Quem bebeu mais chope no fim de semana
0 
b) Quantos chopes Cláudio ficou devendo para Antônio?  1 
é um número inteiro qualquer. Se P = (aij) é uma matriz definida
55 por P = X n + X n – 1 + X n – 2 + … + X, onde n é um número inteiro
1 1 positivo (n  1), então podemos afirmar que:
(UFRJ-99) Seja A=  a) um elemento aij da matriz P é igual a m.n.(n + 1)/2
0 1 b) um elemento aij da matriz P é igual a m.n.(n – 1)/2
a) Determine A3 = A.A.A c) um elemento aij da matriz P é igual a n.m.(m – 1)/2
b) Se An denota o produto de A por A n vezes, determine o valor d) P é uma matriz cujos elementos são todos inteiros, se, e
do número natural k tal que somente se, m é par.
e) nenhuma das respostas anteriores
A k − A 5k + A 6 = I
2
onde I é a matriz identidade.
62
56 (ITA-80) Sejam A, B e C matrizes reais quadradas de ordem n e
(UFLA-98) Dadas as matrizes A do tipo mxn, B do tipo pxq On a matriz nula, também de ordem n. Considere as seguintes
e C do tipo rxs, qual a condição entre m, n, p, q, r e s para afirmações:
que exista a matriz M = BC – AB ? 1. AB = BA 2. Se AB = AC, então B = C
3. Se A2 = On, então A = On 4. (AB)C = A(BC)
57 5. (A – B)2 = A2 – 2AB + B2
(UEM-98) Se A é uma matriz quadrada, definem-se: A respeito destas afirmações, qual das alternativas a
A2 = A.A; A3 = A.A.A; …; An = A.A…A (n vezes) seguir é verdadeira?
a)Apenas a afirmação 1 é falsa.
 1 0 e b o elemento da 2a linha e 1a
Considere A=  b) Apenas a afirmação 4 é verdadeira.
 3 1 c) A afirmação 5 é verdadeira.
d) A afirmações 2 e 3 são verdadeiras.
( A + A2 +...+ A20 ) . Então, b é
1
coluna da matriz B= e) As afirmações 3 e 4 são verdadeiras.
10
igual a …

58 63
(AFA-95) Dadas as matrizes: A = (aij)8x3 e B = (bij)3x7, onde aij
a b 
 c d  , onde a = 2
= 2i – j e bij = i.j, o elemento c56 da matriz C = (cij) = AxB é: (1+ log 2 5)
(ITA-83) Seja a matriz A = ;
a) 74 b) 162 c) 228 d) 276
 
59 b = 2 log2 8 ; c = log 3 81 e d = log 3 27 .
(UFU-2000) Seja A uma matriz de terceira ordem com Uma matriz real quadrada B, de ordem 2, tal que AB é a
1 − 1 matriz identidade de ordem 2 é:
   
elementos reais. Sabendo-se que A. 0 = 4 , conclui-se
 3 
    log 3 27 2  2 −
 2 d)  2 
log 3 81
a)
0  2   3 
 − log 2 5
que  2 
– 1, 4 e 2 são os elementos da:
 3 
a) diagonal da transposta de A.
− 2 2 log 2 5 3 log 3 81
b) primeira coluna da transposta de A. b)
 
e)
 5 −2 log2 81 
c) primeira linha da transposta de A. −5 
d) última linha da transposta de A.  3

60

MATEMÁTICA – Prof. DOUGLAS OLIVEIRA biblioteca


telegram t.me/bibliotecaPE @PROFDOUGLASOLIVEIRA _2020 9
APROVADO DE A A Z
TURMA IME ITA
 3 
− 2  69
c)  2
 5
 2 −  (IME-86/87) Seja A =
 1 0 .
 2  
−1 1
a) Encontre todas as matrizes B, 2x2, que comutam com A.
64 b) Calcule A– 1.
(ITA-86) Dizemos que duas matrizes reais, 2x1, A e B quaisquer
c) Mostre que A2 = 2A – I, onde I = 
1 0 .
são linearmente dependentes se e somente se existem dois
0 1
números reais x e y não ambos nulos tais que xA + yB = 0, onde 0  
 1  k − n + 1 d) Encontre a fórmula para An em função de A e I, e calcule A100.
é a matriz nula 2x1. Se A=  n , B =  
k − 1  2  70
onde k  * e n  N = (1, 2, 3, ...) (IME-89/90) Determine todas as matrizes X reais, 2x2 tais que:
a) A e B são linearmente dependentes,  k  *.  3 4
b) existe um único k  * tal que A e B não são linearmente X2 =  .
dependentes. 2 3
c) existe um único k  * tal que A e B são linearmente
dependentes.
d) existe apenas dois valores de k  * tais que A e B são
linearmente dependentes.
e) não existe valor de k  * tal que A e B sejam linearmente EXERCÍCIOS
dependentes.

65 01
(ITA-87) Considere P a matriz inversa da matriz M, onde M = a) Mostre que se x e y são números irracionais tais que
1 / 3 0 . A soma dos elementos da diagonal principal da seja racional não nulo, então x + y e x−y são ambos
1 / 7 1  irracionais.
 
matriz P é:
a) 9/4 b) 4/9 c) 4 d) 5/9 e) –1/9 b) Sabendo que a raiz quadrada de um número primo é
irracional, prove que se p e q são primos distintos, então
66 são números irracionais.
(IME-80/81) Mostre que não existem matrizes quadradas A e B
que verifiquem AB – BA = I, onde I é a matriz identidade de uma 02
ordem n qualquer.
a) Sabendo que
, prove que se
então
.
b)Use o resultado do item a) para resolver a equação

67
(IME-80/81) Seja M = (mij) uma matriz quadrada real nxn de
termos positivos. Defini-se o “permanente de M” como
permM =  m1+(1) . m2+( 2) ... mn+( n) onde S é o conjunto 03
S Considere o conjunto de todos os números naturais com
das permutações (t(1), t(2), …, t(n)) de {1, 2, …, n}. A matriz quatro algarismos tais que os algarismos lidos da esquerda
1 2 3 para a direita estão em ordem estrita- mente decrescente.
4 5 6 tem, por exemplo, como permanente 1x5x9 + (a) Quantos elementos possui tal conjunto?
  (b) Se escrevermos tais números em ordem crescente, que
7 8 9 número ocupa a posição
4x8x3 + 2x6x7 + 3x5x7 + 2x4x9 + 1x6x8. Seja a matriz nxn, H
(hij) onde hij = i(j + 1). Calcule o permanente de H. 04

68 Um cubo de 20cm de aresta, apoiado em um piso horizontal


e com a parte superior aberta, contém água até a altura de
(IME-81/82) Determine a matriz H tal que HA = B onde
15cm. Colocando uma pirâmide regular de base quadrada
4 2 6 sólida de altura 30cm com a base apoiada no fundo do cubo,
1 0 2  e  
A=  B =  3 1 5 . o nível da água atinge a altura máxima do cubo, sem
2 1 3  derramar.
2 0 4
MATEMÁTICA – Prof. DOUGLAS OLIVEIRA biblioteca
telegram t.me/bibliotecaPE @PROFDOUGLASOLIVEIRA _2020 10
APROVADO DE A A Z
TURMA IME ITA
(a) Qual o volume do tronco de pirâmide submerso? A menor cota superior de um conjunto A,
(b) Qual o volume da pirâmide quando existe, denomina-se supremo de A e
indica-se por sup A.
05
Sejam a,b e c inteiros tais que é divisível por 9.
A maior cota inferior de um conjunto A,
Mostre que pelo menos um dos inteiros a,b ou c é divisível quando existe, denomina-se ínfimo de A e
por 3 indica-se por inf A.
Se A admitir uma cota superior, então diremos
06 que A é ilimitado superiormente.
(a) Considere um conjunto formado por 11 números inteiros Se A admitir uma cota inferior, diremos que A
positivos diferentes, menores do que 21. Prove que podemos
é limitado inferiormente.
escolher dois desses números tais que um divide o outro.
(b) Exiba um conjunto com 10 números inteiros positivos,
menores do que 21, tais que nenhum deles é múltiplo de Vizinhança em R (unidimensional)
outro.
Denomina-se vizinhança unidimensional de
07 um ponto P0(X0) e de raio  (delta)   R a todo
Considere o seguinte sistema de congruências
conjunto de pontos P(x)  R / d (P, P0) < .
X ≡ 1 mod 9
V (P0, ) = {x  R / 0  d (P, P0) < }, onde x
X ≡ 5 mod 7 é a abscissa do ponto P.
X ≡ 3 mod 5

(a) Encontre o menor número natural que satisfaz o


sistema.
(b) Alguma solução do sistema é solução da congruência X ≡
926 mod 3?
Vizinhança perfurada em R
08
Denomina-se vizinhança perfurada
As diagonais AD e CE do pentágono regular ABCDE de lados unidimensional de um ponto P0(X0) e de raio  
de medida a, intersectam-se no ponto P. Determine AP e PD
R a todo conjunto de pontos P(x)  R / 0 < d (P,
em funçao de a.
P0) < .

LIMITES 
V ( P0 ,  ) = x  R / 0  d ( P, P0 )   
V ( P0 ,  ) = 0  x − x0  

Ponto de acumulação

Resumo teórico: Um ponto P0 (X0) é A se e somente se  V


(P0) existir pelo menos um ponto P  R / P  A
Seja A um conjunto de números reais. O e P  V (P0).
maior elemento de A, quando existe,
denominamos máximo de A e indica-se por máx
A. O menor elemento de A, quando existe,
denomina-se mínimo de A e indica-se por min A.
Dizemos que um número m é uma cota Função Real de Variável Real
superior de A se m for máximo de A ou se m for
estritamente maior que todo número de A. Seja F uma relação de um conjunto A em um
Diremos que m é uma cota inferior de A se m for conjunto B tal que para todo x pertence a A
mínimo de A ou se m for estritamente menor que corresponder um único y  B, então esta
todo número de A. relação denomina-se função.
Notação:
MATEMÁTICA – Prof. DOUGLAS OLIVEIRA biblioteca
telegram t.me/bibliotecaPE @PROFDOUGLASOLIVEIRA _2020 11
APROVADO DE A A Z
TURMA IME ITA
lim f(x) = L
x→a
F:A→B
y = F(x)
se e somente se para todo   0,   0 / f(x) − L  

Quando definimos uma relação como função sempre que 0  x − a  .


apenas pela lei de correspondência y – f(x),
estamos admitindo que o domínio ou campo de
existência da função é o conjunto de todo x  R
e y = F(x). Limite à direita

Função Par Seja f uma função definida em um intervalo


(a, c)
Seja y = f(x) definida em um domínio D,
dizemos que f é par, se e somente se para todo x
 D, -x  D e f(-x) = f(x).
Observe que o gráfico de funções pares são e L um número real, a afirmação lim f(x) = L,
x→a
simétricos ao eixo dos y.
significa que para todo   0,    0 / f(x) − L  
sempre que 0 x−a→a xa+→

Limite à esquerda

Função Ímpar Seja f uma função definida no intervalo (c, a)


e L um número real, a afirmação lim f(x) = L,
x→a −

Seja y = f(x) definida em um domínio D, significa que para todo


dizemos que f é ímpar, se e somente se para todo   0,   0 / f(x) − L  
x  D, -x  D e f(-x) = -f(x). sempre que
Observe que o gráfico de funções ímpares −  x − a  0 → a −   x  a
são simétricos em relação a origem.

Teorema

1- lim f(x) = L  lim f(x) = lim f(x) = L


x→a x→a+

x→a
sen u
lim = 1 (1º Limite Fundamental)
u→0 u

EXERCÍCIOS
Definição Formal de Limite
01. Determine, caso existam, o máximo, mínimo,
Sendo f(x) definida em um domínio D do
supremo é ínfimo.
qual a é ponto de acumulação dizemos que f(x)
a) A =  x R|− 3  x  4
tem limite L quando x tende para a, e se indica
por: b) A = x  R| −3  x  4
c) A = x  R|x  5

MATEMÁTICA – Prof. DOUGLAS OLIVEIRA biblioteca


telegram t.me/bibliotecaPE @PROFDOUGLASOLIVEIRA _2020 12
APROVADO DE A A Z
TURMA IME ITA
d) A =  x  R|x  2  5x − x2 

 
x−2 e) f ( x) = log  
e) A = x  R| 0  4 
x+3
f) A = x R||3x − 1|  1  x − 3
f) f ( x) = arc sen  − log ( 4 − x)
 2 
g) A =  −3, −1,0,2,1 1
g) f ( x) =
h) A= 
n
n+1
|n  N  h) f ( x)
x −x
= log ( cosx)

 x2
 

02. A= |x R  é limitada superiormente? 06. Esboce o gráfico das seguintes funções:
2
1 + x
 
 a) f(x) = x
Por quê?
b) f(x) = x2 − 4

c) f(x) = sen x
d) f(x) = x − x
e) f(x) = 3x + 5
03. Considere os conjuntos
f) f(x) = 3 cos (2x)
 x−1  g) y = f(x) = x + x2
 

 

A = x  R :  0 , B = x  R : log 1x  1 . h) f(x) = cosx + cosx
 

 x2 
 i) f(x) = x x + 2

Para cada um dos conjuntos A e B, indique o j) y = f(x) =


2
conjunto dos majorantes, o conjunto dos x + x2
minorantes e, no caso de existirem (em R), o 07. Determine a paridade das seguintes funções:
supremo, o ínfimo, o máximo, o máximo e o
a) f(x) = log  X + 1 + X 2 
mínimo.
 1 − X
b) f(x) = log 
04. Considere os conjuntos  1 + X 

c) f(x) = 2x 3 − x + 1

 
 
 0 , B = x  R : e x  e − x . f(x) = 4 − 2x4 + sen 2 x
x d)
A = x  R :
x
e (x + 1)
 
08. Dados n números Determine o
a1 ,a 2 ,...,a n .
Para cada um dos conjuntos A e B, indique o valor de x para o qual a função:
conjunto dos majorantes, o conjunto dos
f(x) = (x − a1) + (x − a 3 ) + ... + (X − a n )2
2 2 tem valor
minorantes e, no caso de existirem (em R), o
supremo, o ínfimo, o máximo e o mínimo. mínimo.

05. Ache o domínio de definição das seguintes 09. Prove que a seqüência com termo geral
2n − 1
funções: Xn = é crescente.
2n + 1
a) f ( x) = x − 1 + 6 − x
1
b) f ( x) = x2 − x − 2 + 10 n
10. Dada a seqüência com termo geral Xn = ,
3 + 2x − x 2 n!
x mostre que esta seqüência decresce para n  10.
c) f ( x) =
2
x −x−2
d) f ( x) = sen x − 1
11. Prove que a seqüência converge e ache seu
limite:

MATEMÁTICA – Prof. DOUGLAS OLIVEIRA biblioteca


telegram t.me/bibliotecaPE @PROFDOUGLASOLIVEIRA _2020 13
APROVADO DE A A Z
TURMA IME ITA
X1 = 2 ; 1
a) ; b) 1,043 ; c) lg 1,1.
1,02
X2 = 2 + 2 ;

X 3 = 2 + 2 + 2 ;..., 16. Calcule:


X n = 2 + 2 + ,..., + 2 lim 3n2 + 5n + 4
a)
n radicais n →  2 + n2
lim 5n 3 + 2n 2 − 3n + 7
b)
12. Achar o limite do ângulo interno de um n→ 4n 3 − 2n + 11
polígono regular de n-lados, quando n → . lim 4n 2 − 4n + 3
c)
n →  2n 3 + 3n + 4
13. Achar o limite dos perímetros de n-polígonos lim 12 + 22 + ... + n 2
regulares, inscritos numa circunferência de raio d)
n→ 5n 3 + n + 1
R e circunscritos ao seu torno, quando n → .
( )
lim
e) 2n + 3 − n − 1
n → +
14. Para cada número natural n  1, seja Fn a lim  2 2 
figura plana composta de quadradinhos de lados f)  n + n + 1 − n − n + 1
n→ 
iguais a 1 , dispostos da seguinte forma: lim  3 2 3 
n g)  n − n + n
n→ 

lim  n 2 + 1 + n 
h)  
n→4 3
 n + n − n 
lim  1 1 1 1 
i) + + + ... +
n →   1.2 2.3 3.4 
n(n + 1) 
lim  2 
j)  9x + 1 − 3x
x → + 
lim  2 
k)  2x − 3 − 5x
x → − 
Fn é formada por uma fila de n quadradinhos, lim
mais uma fila de (n - 1) quadradinhos, mais uma l) x( x2 + 1 − x)
x → +
fila de (n - 2) quadradinhos e assim lim  1 2 3 n − 1
sucessivamente, sendo a última fila composta de m)  + + + ... + .
n →  n 2 n 2 n 2 n2 
um só quadradinho (a figura ilustra o caso n = 7 ). lim (n + 1) (n + 2) (n + 3)
Calcule o limite da área de Fn quando n n) .
n→ n3
tende a infinito. lim  1 + 3 + 5 + 7 + ... + (2n − 1) 2n + 1 
o) − .
n →   n+1 2 
15. Demonstrar que, quando x → 0, verificam-se
as igualdades aproximadas seguintes:
lim 12 + 22 + 32 + ... + n 2
p) .
n→ n2
1
I)  1 − x;
( )
1+ x lim
q) n+1 − n .
x n→
II) a2 + x  a + (a  0) ;
2a 1 1 1 1 1 1
r) lim x→0 + + − − +
(1 + x)n  1 + nx (n é número natural) ;
+

III) x x x x x x
IV) lg (1 + x)  Mx, 1+ x −1
s) lim x→0
31+ x −1

onde M = lg e = 0,43429 ... 1 + 2x - 1 - 2x


t) lim
x→0 x
Partindo destas fórmulas, calcular 4 4
x + 1 − x2 + 1
aproximadamente: u) lim
x→ 0 x2

MATEMÁTICA – Prof. DOUGLAS OLIVEIRA biblioteca


telegram t.me/bibliotecaPE @PROFDOUGLASOLIVEIRA _2020 14
APROVADO DE A A Z
TURMA IME ITA
x2 − (x + h)2
v) lim
x→0 h 21. Se

17. Calcule os seguintes limites abaixo: lim 4x2 − 5x + 1 lim 1 − x + 5x 2


= L1 e = L2
lim  1 − cos x  x → + −3x + 7x2 x → − (x + 1)3 − (x − 1)3
a)
x → 0  x 2 
lim  tg x − sen x  e então:
b)
x → 0  x3

a) L1 < L2 b) L1 > L2 c) L2 = L2
 x  d) L1 = 20 e) L1 . L2 = 24
lim cos  2  L2 42 35
c)
x →1 1− x
lim   lim 1 + sen x − 1 − sen x
d)  n sen  . 22. (EN)
n→  n x→0 x
lim 1 − cos x
e) . a) é l
x→0 x2 b) é 0
1
f) lim x sen ; c) não existe
x
x→0 d) é + 
1
lim x se n . e) é – l
g) x
x→
GABARITO:
lim n se n n!
h) .
n →  n2 + 1 01.
sen x a) máx A = 4; mín A = –3; sup A = 4; inf A = –3
i) lim
x→ x −  +

b) inf A = –3; sup A = 4

j) lim 11− sen x


c) sup A = 5
 −x d) mín A = 2; inf A = 2
x→
2 2 e) máx A = 2; sup A = 2; inf A = –3
k) lim cosec3x f) não admite supremo, nem ínfimo
x→0 cotgx g) máx A = sup A = 1; mín A = inf A = –3
l) lim arc tg x cotgx h) min A = inf A = 0; sup A = 1
x→0 +

2
m) lim x 02. Sim
x→0 1 − cos(x)
03.
18. Determine as constantes k e b reais tais que inf A = mín A = 1 e inf B = 0 e sup B = Máx B = 0,1

 x1000 + 1  04. sup A = Max A = 0 e inf B = min B = 0


lim kx + b −  = 0.
x→+  x999 + 1 
lim
05.
19. (EN) x2 + 4x − x2 + 1 = a) 1, 6
x→
a) 0 b) 2 c) 3 d) 4 b)  2, 3
c)  −, − 1  2, +  
20. (AMAN) O valor de: 
d) x= + k . 2  , k 
2
lim  sen 2x + sen 4x − sen x  e) 1; 4
x → 0  5sen x + sen 2x + sen 3x 
f) 1; 4 
é: g)  −, 0 
a) 0,2 b) 0,333... c) 6 − 
h) + k . 2   x  + k . 2  , k 
2 2
d) 9 e) 0,6
MATEMÁTICA – Prof. DOUGLAS OLIVEIRA biblioteca
telegram t.me/bibliotecaPE @PROFDOUGLASOLIVEIRA _2020 15
APROVADO DE A A Z
TURMA IME ITA
x −1
b) lim x →1
06. x−1
1 + 2x − 3
07.
c) lim x →1
x −1
a) Ímpar
1 1 1 1 1 1
b) Ímpar d) lim x→0+ + + − − +
x x x x x x
c) Nem par, nem ímpar
d) par 1+ x −1
e) lim x → 0 3
1+ x −1
a1 + a 2 + ... + a n 3
x2 − 2 3 x + 1
08. X = f) lim x →1
n (x − 1)2
1 + 2x - 1 - 2x
09. Demonstração 10. Demonstração g) lim
x→0 x
11. Demonstração  
1 1
h) lim  − 
12. 
x →1 
 2 1 (
− x 3 1 − 3
x) (

 )
13. 2 R
2x + 1 − 3
1 i) lim
14. x→ 4
x−2 − 2
2 3
x+1 −1
15. Demonstração j) lim
x→0 x

16.
02.
5
a) 3 b) c) 0 lim 3n 2 + 5n + 4
4 a)
n→ 2 + n2
d) 1/15 e) + f) √2
lim 4n 2 − 4n + 3
g) 1/2 h) + i) 1 b)
n →  2n 3 + 3n + 4
j) 0 k) + l) 1/2
lim 12 + 2 2 + ... + n 2
m) 1/2 n) 1 o) -3/2 c)
n →  5n 3 + n + 1
p) 1/3 q) 0 r) 1
s) 3/2
V) -2X
t) 2 u) - ½ d)
n→(
lim
n2 + n + 1 − n2 − n + 1 )
n→( )
lim
e) 3
n2 − n3 + n
17.
1 1  lim  1 1 1 1 
a) b) c) f) + + + ... +
2 2 2 n →   1.2 2.3 3.4 
n(n + 1) 
1
d) π e) f) 0 lim  1 2 3 n − 1
+ 2 + 2 + ... + 2  .
2 g) 
n →  n 2
n n n 
g) 1 h) 0 i) -1
lim n + ( −1)n
 h)
j) 0 k) 1/3 l) n →  n − ( −1)n
4
m) 2
i)
lim
n→
( n+1− n . )
18. K = 1 e b = 0. lim 12 + 2 2 + 32 + ... + n 2
j) .
n→ n2
19. B 20. E 21. A 22. A

LIMITES PARTE 2

01. 03.
x −4
2
a) lim x → 2
x−2

MATEMÁTICA – Prof. DOUGLAS OLIVEIRA biblioteca


telegram t.me/bibliotecaPE @PROFDOUGLASOLIVEIRA _2020 16
APROVADO DE A A Z
TURMA IME ITA
lim 1 09. Achar o limite dos perímetros de n-polígonos
a) 1 ;
x → −0 1+ e x regulares, inscritos numa circunferência de raio
lim 1 R e circunscritos ao seu torno, quando n → .
b) 1
.
x → + 0 1+ ex
lim x −1 10. Para cada número natural n  1, seja Fn a
c) ;
x → 1− 0 x −1 figura plana composta de quadradinhos de lados
1
lim x −1 iguais a , dispostos da seguinte forma:
d) n
x → 1+ 0 x −1 .

lim x
e) x − 2;
x → 2−0
lim x
f) x − 2.
x→2+0

04. Calcule: lim x + x + x − x.


x →+

05. Determine as constantes k, b  R tais que


Fn é formada por uma fila de n quadradinhos,
 x1000 + 1 mais uma fila de (n - 1) quadradinhos, mais uma
lim kx + b − 999 =0
x →+
 x +1 fila de (n - 2) quadradinhos e assim
sucessivamente, sendo a última fila composta de
06. O que ocorre com a raiz da equação um só quadradinho (a figura ilustra o caso n = 7 ).
quadrada ax2 + bx + c = 0 se o Calcule o limite da área de Fn quando n
coeficiente a tende a zero e os coeficientes b e c tende a infinito.
são constantes, sendo que b  0 ?
11. Considere o triângulo T, de vértices A, B e
07. a poligonal da figura a seguir, de lados P0P1, C, tal que os ângulos  e B̂ são agudos. Seja H
P1P2, P2P3, ... cada lado é perpendicular ao a altura relativa ao lado AB. Para cada número
anterior e tem comprimento igual à metade do natural n, seja Fn a figura formada pela união de
comprimento do lado anterior. Se P0P1=1, então, n retângulos justapostos contidos em T (veja na
quando n tende para infinito, o limite da figura o caso n = 4). Cada retângulo tem dois
distância entre os vértices P0 e Pn vale: lados perpendiculares a AB medindo
a) 1
H
n +1
b) 2 5 /3
e um lado ligando AC a BC (o maior dos
c) 2 3 /5
retângulos tem um lado contido em AB).
d) 4/5

e) 2 5 /5

08. Achar o limite do ângulo interno de um


polígono regular de n-lados, quando n → .

MATEMÁTICA – Prof. DOUGLAS OLIVEIRA biblioteca


telegram t.me/bibliotecaPE @PROFDOUGLASOLIVEIRA _2020 17
APROVADO DE A A Z
TURMA IME ITA
x
lim  2
f) 1 +  .
x→  x
lim  x  x
g)   .
x →   x+1
lim 1g(1 + 10x)
h)
x→0 x

lim x [ ln( x + 1) − ln x ].
i)
x → +
 
lim  1 ln 1 + x 
j)  1− x .
x→0 x 

Sabendo que a área de T é a, calcule, em função


de a e de n, a diferença entre a área de T e a área 17.
de Fn . Qual o limite da área de Fn , quando n lim se n 3x lim se n 5x
a) . b) .
x→0 x x → 0 se n 2x
tende a infinito?
lim 1 − cos x lim se n x − cos x
c) . d)  .
x→0 x2 x→ 1 − tg x
12. Prove que a seqüência com termo geral 4
2n − 1 lim tg x
Xn = é crescente. e) . f)
2n + 1 x → −2 x + 2
lim se n x − se n a
.
10 n x→a x−a
13. Dada a seqüência com termo geral Xn = ,
n! lim cos x − cos a lim se n( x + h) − se n x
mostre que esta seqüência decresce para n  10. g) . h) .
x→a x−a h→0 h
1
lim lim x se n .
(1 + se n x)
1
14. Prove que a seqüência converge e ache seu i) x . j) x
x→0 x→
limite:
18. PROVE QUE Se f é uma função limitada e
lim g(x) = 0 Então lim f(x).g(x) = 0 .
X1 = 2; X 2 = 2 + 2 ; X 3 = 2 + 2 + 2 ;...,X n = 2 + 2 + ,..., + 2 ;... x→a x→a

n radicais
19.
lim se n x
15. Seja a seqüência real (xn ), n N* tal que a) .
x→ x
lim (xn − xn − 2 ) = 0, para n = 2, 3, .... Prove que 1
n→ b) lim x sen ;
x
 x − x n −1  x→0
lim  n
n →  n  = 0.
lim n se n n!
c) .
n →  n2 + 1
16.
lim  1 7 x 20.
a) 1 + 
x→  x lim arcsen x
a) .
lim  x  x x→0 x
b)   lim
x →   1+ x  b)
arctg 2x
.
mx x → 0 se n 3x
lim  k
c) 1 +  lim 1 + sen x − 1 − sen x
x→  x
c) .
lim  ln (a + x) − ln a  x→0 x
d)  
x→0  x 
lim  1
n
21.
e) 1 −  .
x→  n

MATEMÁTICA – Prof. DOUGLAS OLIVEIRA biblioteca


telegram t.me/bibliotecaPE @PROFDOUGLASOLIVEIRA _2020 18
APROVADO DE A A Z
TURMA IME ITA
lim  e x − x − x 
a)  
x → 0  sen x 

lim  e 4x − 1
b) c  
x → 0  tg x 
30. Sejam E0=[0,1] e f1, f2: E0 → E0 funções
lim ln(1 + x)
f1 ( x ) = x e f2 (x)= x + .
c) 1 1 2
x → 0 3x − 1 definidas por Se P(E0)
3 3 3
é o conjunto das partes de E0, seja F : P (E0) → P
e x − esenx (E0) a função definida por F (A) = f1(A)
22. Calcule lim
x→ 0 x − senx U f2(A), onde f1(A) é a imagem de A por
f1, i = 1,2. Agora, para cana n  1 definimos En =
23. Sabendo que y = lim e
x 1+ 2x
, o logaritmo F (En – 1).
x→0 a) Esboce graficamente E0, E1, E2 e E3. Mostre
neperiano de y vale: que En  En–1.
a) e2 b) e c) e e d) 2e e) -3e b) Calcule lim En , onde En é a soma dos
n →

2x + 4 comprimentos dos intervalos que formam En.


 x + 7
24. O lim   é igual a:
x→−  x + 5 
GABARITO PARTE 2
a) e b) e2 c) e3 d) e4 e) 1
01.
25. Sendo a) 4 b) 1/2 c) 2 3 / 3
 2 x.sen6x 
A = lim   e d) 1 e) 3/2 f) 1/9
2
x→ 0  cosec6x(1-cos 6x) 
  g) 2 h) 0 i) 4 2
2
B= lim  2(2x +1)  , A B vale: j) 1/3
x→log 2 3   2
02.
a) 2 3 b) 6 c) 12 a) 3 b) 0 c) 1/15
d) 6 3 e) 18 d) 2 e) 1/2 f) 1
g) ½ h) 1 i) 0
j) 1/3
26. Calcule:
1 1
a) lim x 1− x b) lim x 1− x 03.
x→ x →1
a) 1 b) 0 c) -1
1
d) 1 e) -  f) + 
27. Calcule lim x  e x
x →0
1
a) 0 b) 1 c) e d) e e)  04.
2

 1  05. k = 1 e b = 0.
28. Calcule lim (ln x) x − e  .
x→ e  
 
06. x1 → -c/b e x2 → 
n
 1 1 1 1
 1n + 2 n + 3 n  07. E 08.  09. 2  Rq 10.
29. lim   2
n →  3 
 
11.

MATEMÁTICA – Prof. DOUGLAS OLIVEIRA biblioteca


telegram t.me/bibliotecaPE @PROFDOUGLASOLIVEIRA _2020 19
APROVADO DE A A Z
TURMA IME ITA
a n
área (Fn ) = a − = a.
n+1 n+1
e MATEMÁTICA DISCRETA
lim área(Fn ) = a.
x →+

01
12. Demonstração 13. Demonstração Seja x0=1 e para , defina xn como sendo o determinante nxn
14. 2 15. Demonstração da Matriz abaixo
p p−q 0 0 ... 0 0
q p p−q 0 ... 0 0
16. 0 q p p − q ... 0 0
1 1
a) e7 b) c) e mk d) e) e-1 f) e2 g) e- 0 0 q p ... 0 0
e a
... ... ... ... ... ... ...
1
0 0 0 0 ... p p − q
h) 10 log e i) 1 j) 1 0 0 0 0 ... q p
Onde p e q são constantes distintas e não-nulas. Determine, e
17. resolva, uma equação de recorrência para a sequência xn

a) 3 b) 5/2 c) ½ 02
d) -√2/2 e) π f) ccos a
Sejam f,g: → funções tais que f é sobrejetiva, g é injetiva e
g) – sen a h) cos x i) e
f (n)  g (n) f (n)  g (n) para todo n  1 . Prove que
j) 1
f(n)=g(n), para todo n  1 .

18. Demonstração 03
Seja a1 , a2 ,... um sequência de números reais satisfazendo
19.
a) 0 b) 0 c) 0 ai + j  ai + a j para todos i,j=1, 2, 3, ... . Prove que

a2 a3 a
20. a1 + + + ... + n  an, n1 Dê um exemplo de uma
2 3 n
a) 1 b) 2/3 c) 1 sequência desse tipo.

04
21.
1 A sequência a1 , a2 ,..., an ,... é definida por a1=20, a2=30,
a) 2 b) 4 c)
ln 3 an + 2 = 3an +1 − an , n  1. Encontre todos os inteiros

positivos n tais que 1 + 5an an +1 seja um quadrado perfeito.


22. 1 23. C 24. D
25. B 05
Defina as sequências (an ; n  0) e (bn ; n  0) por
26.
an + bn 3 = (2 + 3)2
n

a) 1 b) e −1 . Obtenha equações de recorrência


pra essas sequências e use-as para calcular o resto da divisão de
 (2 + 3) 2  por
1 1000

27. E 28. e e
29. 1   17, onde  x  indica a parte inteira do

número real x.
30.
06
a)
Mostre que não existe função f: → tal f((n))=n+1.

07
+ + +
Considere uma função f: → , sendo o conjunto
dos números inteiros não negativos, com as seguintes
b) lim E n = 0 propriedades:
n →
(A) f(n+1)>f(n), para cada
+
n .

MATEMÁTICA – Prof. DOUGLAS OLIVEIRA biblioteca


telegram t.me/bibliotecaPE @PROFDOUGLASOLIVEIRA _2020 20
APROVADO DE A A Z
TURMA IME ITA
(B) f(n+f(m))=f(n)+m+1 para todo Seja a1 , a2 ,..., an ,... uma sequência satisfazendo a0 = 3 e
+
n, m  n
1
Prove que a única função com essas propriedades é (3 − an +1 )(6 + an ) = 18 calcule  .
f(n)=n+1 i =0 ai
08 16
Prove que a sequência definida por y0=1 e
A sequência a1 , a2 ,..., an ,... satisfaz
1
yn+1 = (3 yn + 5 yn2 − 4), n  0; contém apenas 1
2 a1 = a2 = 1 e an+ 2 = + an , n  1 , calcule a2014
números inteiros. an+1
09
Demonstre que se a1 , a2 ,..., an são n  3 números reais não TRIGONOMETRIA NO
nulos tais que
TRIÂNGULO RETÂNGULO
(a12 + a22 + ... + an2−1 )(a22 + a32 + ... + an2 ) = (a1a2 + a2 a3 + ... + an −1an ) 2
Então, esses números estão em progressão geométrica. Vale a 01
recíproca? Considere as proposições:
(i) Se um triângulo retângulo tem um ângulo  e hipotenusa
10
de comprimento a, então os catetos deste triângulo medem
Considere uma sequência de números reais não nulos
a  sen (o cateto oposto a  ) e a  cos  (o cateto
a1 , a2 ,..., an sendo n  3 . Prove que esta sequência é uma adjacente a  )
progressão aritmética se, e somente se, vale (ii) Se dois ângulos  e  são complementares
1 1 1 1 n −1 o
(  +  = 90 ) então sen = cos  (o cosseno de um
+ + + ... + = e em
a1a2 a2 a3 a3a4 an−1an a1an ângulo é o seno do ângulo complementar) e tg = 1 .
particular, conclua que tg
1
+
1
+
1
+ ... +
1
=
2011 (
(iii) Se   0 , 45
o o
) então
1.2 2.3 3.4 2011.2012 2012 sen2 = 2  sen  cos 
(iv) Se   ( 0 , 90 ) então
o o
11
1 3 5 2n − 1  1 − cos 
Calcule a soma + 2 + 3 + ... + n sen =
2 2 2 2 2 2
(v) Se  e  são agudos de um triângulo retângulo então:
12 sen ( +  ) = sen cos  + sen cos 
Calcule o primeiro termo e a razão da progressão aritmética
cos ( +  ) = cos s cos  − sen sen
a1 , a2 ,..., an sabendo que
tg + tg 
a1 + a2 + ... + an = a e a12 + a22 + ... + an2 = b 2 tg ( +  ) =
1 − tg tg 
Conclua que:
13 (A) Quatro são falsas.
Sejam a1 , a2 ,..., an ,... uma P.G. de razão q>0, com (B) Duas são verdadeiras e três são falsas.
(C) Três são verdadeiras e duas são falsas.
a1  0 e b1 , b2 ,..., bn ,... uma P.A. de razão r>0. Suponha que
(D) Quatro são verdadeiras e uma é falsa.
 é tal que log an − bn não depende de n. Determine uma (E) Todas são verdadeiras.
relação entre q e r.
02
14 Sabendo que 3senx + 4cos x = 5 , o valor de
Sabe-se que a P.A. a1 , a2 ,..., a16 satisfaz a7 + a9 = 16 . senx − cos x é igual a:
1 1 1
Determine todas as subsequências de três termos que formam uma (A) (B) (C)
P.G. 5 4 2
1
15 (D) − (E) −1
5

03

MATEMÁTICA – Prof. DOUGLAS OLIVEIRA biblioteca


telegram t.me/bibliotecaPE @PROFDOUGLASOLIVEIRA _2020 21
APROVADO DE A A Z
TURMA IME ITA
17 1 1
Sabendo que sen  + cos  = , o valor de sen 2 é (A) 0 (B) (C)
13 4 2
igual a : 3
(D) (E) 1
120 122 124 4
(A) (B) (C)
169 169 169
126 128 10
(D) (E) O valor do produto
169 169
sen 10o  sen 50o  sen 70o
04 é igual a:
O valor de sen18 é igual a:
o 1 1 1
(A) (B) (C)
5 −1 5 −1 5 2 4 8
(A) (B) (C) 1
2 4 4 (D) (E) 1
5 5 +1 16
(D) (E)
8 4

05
Em um triângulo retângulo ABC, de hipotenusa BC, seja D
um ponto do cateto AB tal que AD = 1, DB =10 e o ângulo 11
BCD = 30o . Se a medida do cateto AC possui a forma Em um triângulo retângulo ABC, de hipotenusa BC, a medida
m + n p com m,n e p positivos, o valor de m+n+p é igual a: do cateto AB é igual a 2 e a medida do ângulo agudo
formado pelas medianas que partem dos vértices B e C é
(A) 16 (B) 14 (C) 12
igual a 30o. A medida do cateto AC é:
(D) 10 (E) 8

(A)
3 3  11
(B)
9  33
(C)
11 ( 3 1 )
06
4 4 4
m
( )
6
O valor de sen15 + sen75
o o
possui a forma onde 9  11 11  3 3
n (D) (E)
4 4
m e n são primos entre si. O valor de m+n é igual a :
(A) 31 (B) 33 (C) 35
(D) 37 (E) 39
12
Simplificando a expressão
07
Se x = cos 36º − cos 72º então x é igual a : sen4 x + 4 cos2 x − cos4 x + 4sen2 x
1 1 Obtemos:
(A) (B) (C) 3 − 6 (A) cos x (B) cos 2x (C) sen x
3 2
2 2
1 (D) sen x (E) cos x
(D) 2 3 − 3 (E)
4

08 13
Seja
Sendo  e  as medidas dos ângulos agudos de um
1
triângulo retângulo e sabendo que fk ( x ) =
k
(
senk x + cosk x )
sen2 2 − 2 cos 2 = 0 então sen  é igual a:
para k=0,1,2,...

(A)
2
(B)
2 4
(C)
8 4
( ) ( )
O valor de f4 x − f6 x é igual a:
2 2 2 1 1 1
(A) (B) (C)
4
8 6 8 12
(D) (E) 0
4 1 1
(D) (E)
24 36
09
cos3 15o + sen315o 14
O valor de é igual a:
cos15o + sen15o

MATEMÁTICA – Prof. DOUGLAS OLIVEIRA biblioteca


telegram t.me/bibliotecaPE @PROFDOUGLASOLIVEIRA _2020 22
APROVADO DE A A Z
TURMA IME ITA
Suponha
cos 2x
que =2 para algum ângulo (1 − cotg 22 )  (1 − cotg 23 )
o o

cos x é igual a:
sen 2x (A) 0 (B) 1 (C) 2
0  x  900 . O valor de é igual a :
sen x (D) 3 (E) 4
(A) 1 + 3 (B) 1 − 3 (C) 3 − 1
20
(D) 3 (E) 2 + 3 22
Sabendo que sec x − tg x = e
7
m
15 cos sec x + cot g x = com m e n primos entre si, o
n
Sejam  e  números reais tais que
valor de m+n é:
2 6 (A) 40 (B) 42 (C) 44
sen  + sen  = e cos  + cos  =
2 2 (D) 46 (E) 48
o valor de sen ( +  ) é igual a:
21
1 2 3
(A) (B) (C) O valor de sen8 75o − cos875o é igual a:
2 2 2
3 3 3 5 3
6 (A) (B) (C)
(D) (E) 1 16 16 16
2
7 3 9 3
(D) (E)
16 16

16
Utilizando um triângulo retângulo de hipotenusa igual a 1 e
um dos ângulos agudos de medida  podemos provar que 22
3 2
 O valor de sen 18 + sen 18 é:
tg é igual a:
2 1 1 1
(A) (B) (C)
sen sen cos  2 4 8
(A) (B) (C)
1 + cos  1 − cos  1 + cos  1
(D) (E) 1
16
cos cos
(D) (E)
1 + s en 1 − s en 23
O valor de sen 3 é igual a :
17
Se  é um ângulo agudo tal que
(A)
1
2 (( )( )
5 − 1 6 + 2 − 10 + 2 5 6 − 2 ( ))
tg2 =
cos  − sen
cos  + sen
(B) 1
4 (( )( )
5 − 1 6 + 2 − 10 + 2 5 6 − 2 ( ))
O valor numérico de tg4 é igual a: (C)
1
8 (( )( )
5 − 1 6 + 2 − 10 + 2 5 6 − 2 ( ))
(A) 1 (B)
3
4
(C)
3
3
(D)
1
16 (( )( )
5 − 1 6 + 2 − 10 + 2 5 6 − 2 ( ))
(D) 3 (E) 3 +1 (E) 1
32 (( )( )
5 − 1 6 + 2 − 10 + 2 5 6 − 2 ( ))
18
24
( 2) ( 2)
o o
O valor de cos
4 15 − sen4 15 é O número de valores de x pertencentes ao intervalo (0, 90o)
que satisfazem à equação
6+ 2 6− 2 3
(A) (B) (C) 3 −1 3 +1
4 4 2 + =4 2
senx cosx
1 3 −1
(D) (E) é igual a:
2 2 (A) 0 (B) 1 (C) 2
(D) 3 (E) 4
19
O valor de 25

MATEMÁTICA – Prof. DOUGLAS OLIVEIRA biblioteca


telegram t.me/bibliotecaPE @PROFDOUGLASOLIVEIRA _2020 23
APROVADO DE A A Z
TURMA IME ITA
O menor inteiro positivo solução da equação
cos 96o + sen 96o 30
tg 19xo = Em quantos instantes entre 6,00h e 18,00h não é
cos 96o − sen 96o
possível dizermos a hora exata se não pudermos distinguir o
é:
(A) 151 (B) 153 (C) 155 ponteiro das horas do ponteiro dos minutos?
(D) 157 (E) 159 (A) 130 (B) 131 (C) 132
(D) 133 (E) 134

PROBLEMAS DE PONTEIROS 31
a
Seja a fração irredutível que traduz o número de
b
26 segundos decorridos, após o meio dia, para que o ponteiro
Após o meio dia a que horas os ponteiros de um relógio se dos segundos seja a bissetriz do menor ângulo formado
superpõem pela primeira vez? pelos ponteiros das horas e dos minutos. O valor de a + b é
(A) 1h5 min21311seg igual a
(A) 87821 (B) 87823 (C) 87825
(B) 1h5 min27 311seg
(D) 87827 (E) 87829
(C) 1h5 min23 311seg
(D) 1h5 min29 311seg 32
Em um relógio analógico comum, seja m um número inteiro
(E) 1h5 min25 311seg
tal que 1 m  720 . Precisamente m minut os após as
12 : 00 , o ângulo entre o ponteiro das horas e dos
27
minutos é exatamente 1o . A soma de todos os valores
Existem dois instantes entre 4h e 5h nos quais os
possíveis de m é:
ponteiros de um relógio formam um ângulo reto. A
(A) 700 (B) 720 (C) 740
diferença entre estes dois instantes é igual a:
(D) 760 (E) 780
(A) 32min41711seg
(B) 32min47 711seg
(C) 32min43 711seg VETORES E GEOMETRIA
(D) 32min49 711seg
ANALÍTICA
(E) 32min45 711seg
01
28 Calcule o volume do tetraedro de vértices (5; 8; 10); (10; 10;
Existem dois instantes entre 2h e 3h nos quais a distância 17); (4; 45; 46); (2; 5; 4).
angular do ponteiro das horas ao número 12 é igual à 1
a) b) 1 c) 2 d) 0 e) 5
a 2
distância do ponteiro dos minutos ao número 6. Se é a
b
fração irredutível que traduz a soma das partes 02
fracionárias do número de segundos destes dois instantes No tempo 0, uma formiga está em (1, 0) e uma aranha está
então a + b é:: em (−1, 0). A formiga começa andar no sentido anti-horário
(A) 260 (B) 26 2 (C) 264 ao longo da circunferência unitária, e a aranha começa a
(D) 266 (E) 268 andar para a direita ao longo do eixo x. A velocidade
horizontal da formiga é sempre metade da velocidade
29 horizontal da aranha. A menor distância entre a formiga e a
Ao meio dia os 3 ponteiros de um relógio estão aranha no percurso é:
superpostos. A que horas, pela primeira vez, o ponteiro dos 1 3 3 7
a) 1 b) c) d) e)
segundos será bissetriz do ângulo formado pelos outros 2 2 8 8
dois?
(A) 12h 1min1427
760 seg
03
(B) 12h 1min1427
770 seg
Seja um triângulo de vértices A ( 1, −1, −3 ) , B ( 2,1, −2 ) e
(C) 12h 1min1427
780 seg C ( −5,2, −6 ) , o comprimento da bissetriz do ângulo externo
do vértice A é igual a:
(D) 12h 1min1427
790 seg

(E) 12h 1min1427


800 seg

MATEMÁTICA – Prof. DOUGLAS OLIVEIRA biblioteca


telegram t.me/bibliotecaPE @PROFDOUGLASOLIVEIRA _2020 24
APROVADO DE A A Z
TURMA IME ITA
3 14 4 14 5 14 6 14 2 6 5 6 15 10 3 6
a) 2 14 b) c) d) e) a) b) c) d) e)
2 3 4 5 5 12 3 2 2

04 10
Num triângulo ABC, P, Q e R são pontos, respectivamente,
pertencentes aos lados AB, BC e CA, tais que
Se a+v =7 e a −v = 5, o valor do produto escalar
AP BQ CR 1
= = = e no triângulo PQR, S, T e U são a.v é:
PB QC RA n
a) 8 b) 7 c) 6 d) 5 e) 4
pontos, respectivamente, pertencentes aos lados PQ, QR e
PS QT RU 11
RP, tais que = = = n. Prove que:
SQ TR UP
Os pontos A, B e C estão sobre uma reta r e B está entre A
a) US AB, ST BC e TU CA. e C. Sendo O um ponto fora de r, considere os vetores
US ST TU ( n − n + 1)
2

= = =  a = OA , v = OC e w = OB . Sabendo que
AB BC CA ( n + 1)2
BC = 4 AB , determine x e y de forma que

05 w = xa + yv .
Num triângulo ABC, P, Q e R são pontos, respectivamente, a) x=4/5 e y=1/5 d) x=3/5 e y=2/5
pertencentes aos lados AB, BC e CA, tais que b) x=3/4 e y=1/4 e) x=1/3 e y=2/3
AP BQ CR 1 c) x= y=1/2
= = = e no triângulo PQR, S, T e U são
PB QC RA 3
pontos, respectivamente, pertencentes aos lados PQ, QR e 12
 
PS QT RU US u e v são vetores unitários tais que:
RP, tais que = = = 3. Determine     
SQ TR UP AB |u + 2v | = |u - v |
 
O ângulo entre u e v mede:
06 a) 30o b) 45o c) 60o d) 90o e) 120o
Seja o heptágono regular ABCDEFG de centro P ( −5,8 ) e
13
vértice A ( 2,6 ) , pode-se afirmar que a soma das
coordenadas dos vértices é:

O ângulo entre os vetores a e b é . Sabendo-se que
6
a = 3 e b = 1 , calcule o ângulo entre os vetores a + b
e a−b.

07
ABC é o triângulo, no plano cartesiano, com vértices 
a)
A ( 0, 0 ) , B ( 2, 1) e C (1, 5 ) . Sendo P o ponto do plano, tal 3
que a soma dos quadrados das distâncias de P aos vértices 
b)
de ABC seja a menor possível. O valor dessa soma mínima é: 4
a) 4 b) 6 c) 8 d) 12 e) 16  1 
c) arccos  
08  3
O vértice A de um triângulo acutângulo ABC é eqüidistante  1
d) arccos  
do circuncentro O e do ortocentro H do triângulo. O valor 4
do ângulo A é:  2 
a) 15º b) 30º c) 45º d) 60º e) 75º e) arccos  
 7
09
 14
Seja w um vetor unitário do R 3 , normal aos vetores
  O vetor x perpendicular aos vetores a = 3i + 2j + 2k e
u = (−1, 1, 1) e v = (0 , − 1, − 1) e com 2ª coordenada
  b = 18i − 22j − 5k , forma um ângulo obtuso com o eixo Oy.
positiva. Se  é o ângulo entre os vetores ( 2 w + u) e
  Sabendo que x = 14 , o vetor x é:
( − v ) , 0    , então cossec 2  vale:
2

MATEMÁTICA – Prof. DOUGLAS OLIVEIRA biblioteca


telegram t.me/bibliotecaPE @PROFDOUGLASOLIVEIRA _2020 25
APROVADO DE A A Z
TURMA IME ITA
a) 12i + 4j + 6k x + 43 y − 30 z + 23
d) = =
3 −2 −4
b) −4i − 6j + 12k
x − 2 y + 4 z −1
e) = =
c) 4i − 6j − 12k 3 −2 2
d) 6i − 12j − 4k

e) −6i − 4j + 12k 20
Sabendo-se que u e v são vetores que satisfazem as
15 seguintes condições:
Uma parábola tem foco F e vértice V, onde VF = a . Uma I - u é paralelo a w = i − j + k
corda PQ de comprimento b passa por F. Determine a área
II - v é ortogonal a w ¨
do triângulo VPQ em função de a e b.
ab III - a = u + v onde a = 2i + j − 3k
e) ( a + b )
2
a) ab b) b ab c) a ab d)
2 Podemos afirmar que o produto vetorial, u  v , é:
−16 2 14
a) i+ j+ k
16 9 9 9
Os vetores a e b são perpendiculares e c forma com a e −2 2 2
b) i+ j− k
 3 3 3
b ângulos iguais a rd . Se a e c são unitários, b = 2 e c) nulo
3
−4 10
p = 3a − b + c , então p é igual a: d) i− j − 2k
3 3
a) 5 b) 2 c) 15 d) 2 e) 2 3 16 2 14
e) i+ j− k
3 3 3
17
O módulo do produto vetorial dos vetores a e b , que 21
Sabendo que u, v e w são vetores tais que u + v + w = 0
formam um ângulo obtuso, é 41 e a = 7 e b = 3 . MP
3 1
(vetor nulo) e u = , v = e w = 2 , o valor de
tem a direção da bissetriz do ângulo de a e b e 2 2
MP = 2 42 ; MQ = a − b . A área do triângulo MPQ é: u  v + v  w + u  w é:
a) 3/2
a) 10 41 b) 8 41 c) 20 41 d) 4 42 e) 2 41 42 b) −3/4
c) 9/4
18 d) −11/4
Os vetores a e b no espaço são tais que o módulo de a é e) −13/4
igual a 12 e o módulo de b é igual a 2. Sabendo que m é um
número real tal que os vetores v = a + m  b e u = a − m  b
são perpendiculares, então o produto de todos os possíveis MISCELÂNIA
valores de m é:
a) 6
b) −6 CÁLCULO
c) 36
d) −36 01. Calcule as seguintes integrais, usando substituição
e) 144 trigonométrica:
16 − x 2 dx
19 a)  x2
dx b) x 3
x2 − 9
As equações da reta que passa pelo ponto P(3, −2, −4) ; é
dx dx
paralela ao plano 3x − 2y − 3z − 7 = 0 e intercepta a reta c) x 2
5 − x2
d)  x2 − 7
x − 2 −4 − y z − 1
= = são: dx x2
3 2 2 e) x 25 − x 2
f)  2x − x 2
dx
x −3 y +2 z+4
a) = =
(16 − 9x )
3
5 −6 9 2 2
dx
b)
x −3 y +2 z+4
= =
g)  h) 
x6
( 4x − x )
3
−43 30 −23 2 2

x −5 y +6 z−9
c) = =
3 −2 4

MATEMÁTICA – Prof. DOUGLAS OLIVEIRA biblioteca


telegram t.me/bibliotecaPE @PROFDOUGLASOLIVEIRA _2020 26
APROVADO DE A A Z
TURMA IME ITA
dx respectivamente. Se XN=19 e YM=22, a medida de XY é
i)  x2 + 2 dx j) 
(1 + x ) 2
1− x 2 igual a:
a) 24 b) 26 c) 28 d) 30 e) 32

02. Completando quadrados e usando substituição 08. Se em um triângulo ABC tem-se que AB=1, AC=2 e a
trigonométrica, calcule as seguintes integrais: medida da mediana relativa ao lado BC é igual à medida do
dx x lado BC então, a medida de BC é igual a:
a)  −3 + 8x − 4x 2
b)  1 − x + 3x 2
dx
1+ 2 1+ 3
a) b) c) 2
2x dx
c)  dx d)  2 2
( )
2
x 2 + 3x + 4 x2 + 3x + 5 3
d) e) 3
2
dx 5x + 3
e)  x2 − x − 1
f)  4x 2 + 3x + 1
dx
09. Um quadrilátero convexo ABCD de área igual a 2002,
dx 1 − 2x contem um ponto P em seu interior tal que PA=24, PB=32,
g)  4x − x 2 − 3
h)  2x − x 2 + 3
dx
PC=28 e PD=45. O perímetro de ABCD é igual a:
x x+2 a) 4 2002
i)  x − 3x + 4
2
dx j)  x + 6x + 34
2
dx
b) 4 8465

03. Calcule as seguintes integrais, usando frações parciais:


(
c) 2 48 + 2002 )
dx 4dx d) 2 8633
a)  3 b)  4
x +8 x −1 (
e) 4 36 + 113 )
x 5 + 4x 3 x 3 + 3x
c)  dx d)  (x )
dx
(x ) +1
3 2
2
+2 10. Dado um segmento AB, com centros em B e A
dx x + x −1
3 respectivamente, traçam-se os arcos AC e BC . Se o
e) x f)  dx
+ x2
(x )
4
comprimento do arco BC é igual a 12, então o comprimento
2
2
+1
x + 8x − x + 2x + 1
4 3 2
dx do círculo tangente aos arcos AC e BC e ao segmento AB
g)  (x 2
+x x +1 )( 3
)
dx h)  x (x
3 2
+1 ) é igual a:
a) 24 b) 25 c) 26 d) 27 e) 28
x+1 x +x+1
3
i)  j)  x (1 + x ) dx
(x ) 11. Seja ABC um triângulo isósceles (AB=AC) de modo que
2 2
2
+ 4x + 5
BAC = 12 . Sejam ainda D e E pontos sobre os lados AC e
AB respectivamente de modo que CBD = 42 e
GEOMETRIA PLANA BCE = 30 . A medida do ângulo BDE é igual a:
a) 12o b) 20o c) 24o d) 30o e) 40o
04. Seja ABC um triângulo isósceles (AB=AC) de modo que
BAC = 20 . Sejam ainda M a projeção do ponto C sobre o 12. Em um triângulo ABC o lado AC e a mediatriz de BC
lado AB e N um ponto sobre o lado AC de modo que intersectam-se no ponto D e BD é a bissetriz do ângulo
CN = BC . A medida do ângulo AMN é igual a: ABC . Se AD=9 e DC=7, a área do triângulo ABD é igual a:
2
a) 20o b) 30o c) 45o d) 50o e) 60o a) 14 b) 21 c) 28 d) 14 5 e) 28 5
05. Em um triângulo ABC de base AB e ABC = 45 , seja D
um ponto sobre o lado BC de modo que 2  BD=CD e o ângulo 13. Seja ABC um triângulo isósceles no qual AB = AC.
DAB = 15 . A medida do ângulo ACB é igual a: Supondo que a bissetriz do ângulo B intersecta o lado AC
em D e que BC=BD+AD, a medida do ângulo A é igual a:
a) 54o b) 60o c) 72o d) 75o e) 90o
a) 60o b) 90o c) 100o d) 120o e) 135o
06. São dados um triângulo ABC de base AB e um ponto P
situado no mesmo semiplano do vértice C de modo que P 14. ABC é um triângulo no qual BC = 4 , CA = 5 e AB = 6 .
seja eqüidistante de A e B e o ângulo APB seja o dobro BCA
Determine a razão .
do ângulo ACB . Se D é o ponto de interseção de AC e BP CAB
tal que PB = 3 e PD = 2 então o valor do produto AC  CD é a) 1 b) 1/2 c) 1/3 d) 2 e) 3
igual a:
a) 5 b) 6 c) 7 d) 8 e) 9 15. Na figura abaixo, os segmentos AB e CD têm
comprimento 1, enquanto os ângulos ABCˆ ˆ
e CBD medem
07. Seja XOY um triângulo retângulo XOY = 90 . Se M e
90° e 30°, respectivamente. A medida do segmento AC é:
N são os pontos médios dos catetos OX e OY

MATEMÁTICA – Prof. DOUGLAS OLIVEIRA biblioteca


telegram t.me/bibliotecaPE @PROFDOUGLASOLIVEIRA _2020 27
APROVADO DE A A Z
TURMA IME ITA
21. Um prisma hexagonal regular é cortado pelo plano
mediador de uma aresta da base segundo um quadrado de
12m2 de área.Calcule o volume desse prisma.

22. Em um tetraedro regular prove que todas as seções


paralelas a duas arestas opostas têm mesmo perímetro.

23. Nove esferas iguais de raio r estão no interior de um


cubo de aresta 1 sendo uma central e, cada uma das outras,
a) 2 b) 3 2 c) 3 d) 3 3 e) 2 tangentes a esta esfera central e a três faces. Calcule r.

16. Em cada lado de um triângulo arbitrário ABC é 24. Uma esfera de raio R está inscrita em um cone de base
construído para fora um triângulo eqüilátero, conforme a 2 1 1
de raio r e altura h. Prove que = + .
figura abaixo. Rh R 2 r 2

25. Um quadrado ABCD é tal que AB = AD = 1, B = D = 90°,


BC = DC = 3 . Determine o volume gerado pela rotação do
quadrilátero em torno de um eixo que contém BC.

26. Um triângulo eqüilátero ABC de lado 6 gira em torno de


um eixo que contém o vértice A e faz 15o com o lado AB.
Calcule o volume do sólido gerado.

27. Um triângulo eqüilátero de lado a gira em torno de um


eixo que contém um vértice. Determine o maior volume que
pode ser gerado.

28. Um semicírculo de raio R gira em torno de um eixo


tangente e paralelo ao seu diâmetro. Calcule o volume do
Sabendo que AP = x, o valor de BQ + CR é: sólido gerado.

a)
x
2
(
3 +1 )
b) 2 3x
eixo

c) 2( 3 + 1)x
d) x
e) 2x

17. Sejam A1, A2, A3, ..., A14 seja um polígono regular de 14
lados inscrito num círculo de raio R. Prove que A1A32 + A1A72
+ A3A72 = 7R2.

18. Sabendo-se que existe um hexágono convexo de área


máxima inscrito numa circunferência, prove que esse
hexágono é regular. PARTE 2

GEOMETRIA ESPACIAL 29. Seja um cone circular reto no qual o raio da base mede
1 cm e a geratriz mede 3 cm. P é um ponto na circunferência
19. Calcule o volume do paralelepípedo oblíquo, cujas faces da base e o caminho mais curto a partir de P ao redor do
são losangos de lado 2 com ângulo de 60o. cone e de volta a P é traçado conforme a figura. Calcule a
menor distância entre o vértice V e esse caminho.
20. Calcule o volume do prisma hexagonal regular
circunscrito a uma esfera de raio r.

MATEMÁTICA – Prof. DOUGLAS OLIVEIRA biblioteca


telegram t.me/bibliotecaPE @PROFDOUGLASOLIVEIRA _2020 28
APROVADO DE A A Z
TURMA IME ITA
33. Seja um cubo de aresta a. Seja N um ponto na diagonal
de uma face lateral, M um ponto no círculo que se encontra
no plano da base com centro no centro da base e raio
 5
 12  a . Encontre o menor valor da medida do segmento

MN.

34. Se um poliedro convexo tem 6 vértices e 12 arestas,


prove que toda face é um triângulo.

GEOMETRIA ANALÍTICA

35. O vértice A de um triângulo acutângulo ABC é


3 3 eqüidistante do circuncentro O e do ortocentro H do
a)
2 triângulo. O valor do ângulo A é:
3 2 a) 15° b) 30° c) 45° d) 60° e) 75°
b)
2
36. Sejam, M um ponto de uma hipérbole, N e N’ os pontos
3( 5 − 1)
c) de encontro da normal em M com o eixo transverso e com o
2
MN b2
d)
3 segundo eixo; demonstrar que =− .
2 MN' a2
e) 2
37. Uma elipse é traçada com eixos medindo 10 e 8. Usando
30. O volume de um tetraedro ABCD no qual um de seus focos como centro, um círculo é desenhado
tangente à elipse e contido nela. Calcule o raio do círculo.
AB = AC = AD = 5 , BC = 3 , CD = 4 e DB = 5 é:
a) 6
38. A área do triângulo cujos vértices são as coordenadas
b) 5
11
c) 6 3 do centro da circunferência x 2 + y 2 − x − 2y − = 0 , do
4
d) 5 3 foco da parábola y2 + 6y + 4x + 13 = 0 e do centro da elipse
e) 5 2
3x2 + y2 − 2y − 11 = 0 , é:
a) 1 b) 2 c) 3 d) 4 e) 5
31. É dada uma elipse de eixo focal 2a e excentricidade
2
igual a . Essa elipse é seção de um cone de revolução; o 39. Sejam x e y reais tais que x2 + y2 − 6x − 8y + 24 = 0 . O
3
ângulo que o plano da elipse forma com o eixo do cone é máximo valor de x2 + y 2 é:

 = 45 . A distância do vértice V do cone ao plano da elipse a) 6 b) 24 c) 12 d) 25 e) 36


é:
a) a 40. Os pontos ( ) ( )
A 1,1 , B 3,5 e C 6,6 ( ) são dados.
2 Encontre o quarto ponto D ( x,y ) tais que esses pontos
b) a 
3 sejam vértices de um trapézio isósceles. Calcule também a
a 3 área da região entre o trapézio isósceles e a circunferência
c) circunscrita a ele.
2
a 3
d) 41. Dois lados de um losango de área 24 encontram-se
4
sobre as retas 3y – 4x = 18 e 3y + 4x = 18. O raio do círculo
a 3 inscrito nesse losango é:
e)
8 a) 4/3 b) 3/2 c) 18/5 d) 12/5 e) 8/3
42. Um dos vértices de um triângulo é A (2, – 4), duas das
32. Num poliedro convexo com m faces triangulares (e bissetrizes internas encontram-se sobre as retas x + y – 2
possivelmente faces de outras formas), exatamente quatro = 0 e x – 3y – 6 = 0. Nesse caso, assinale a alternativa
arestas encontram-se em cada vértice. O menor valor incorreta.
possível de m é: a) o ponto (0, 6) é vértice do triângulo
a) 6 b) 8 c) 9 d) 10 e) 12
b) o raio do círculo inscrito no triângulo é 2

MATEMÁTICA – Prof. DOUGLAS OLIVEIRA biblioteca


telegram t.me/bibliotecaPE @PROFDOUGLASOLIVEIRA _2020 29
APROVADO DE A A Z
TURMA IME ITA
c) o ponto (3, – 1) é o incentro do triângulo 51. Se a  sin x + cos x = a , calcule o valor de cosx .
 4 2
d) o ponto  ,  é vértice do triângulo
 3 3 52. Calcule o valor de y = sin10  sin 20  sin 40 .

1
e) a área do triângulo é 21
3 1 3
53. Calcule o valor de y = − .
sin10 cos10
43. Numa parábola de parâmetro p (distância do foco à
diretriz), considera-se uma corda MM', normal em M e vista 
54. Resolva a equação arccos x + arccos 2x = .
do foco F sob um ângulo reto. Calcular FM e FM', em função 3
de p.
55. Resolva a equação sin x + sin 3x = sin 2x + sin 4x .
44. (ITA-07) Considere, no plano cartesiano xy, duas
circunferências C1 e C2, que se tangenciam exteriormente 1 − tan x
em P : (5,10). O ponto Q : (10,12) é o centro de C 1. 56. Resolva a equação = 1 + sin 2x .
1 + tan x
Determine o raio da circunferência C2, sabendo que ela
tangencia a reta definida pela equação x = y.
 
 x + y = 2
45. (ITA-07) Considere no plano cartesiano xy o triângulo 57. Resolva o sistema 
sin x
delimitado pelas retas 2x = y, x = 2y e x = – 2y + 10. A área  = 3
desse triângulo mede
 sin y
a) 15/2
b) 13/4 
sin x − sin y = 3
58. Resolva o sistema 
c) 11/6
 cos x − cos y = 1

d) 9/4
e) 7/2
59. Resolva a equação 2sin17x + 3 cos 5x + sin 5x = 0 .
46. (ITA-07) Sejam A : (a, 0), B : (0, a) e C : (a, a), pontos
do plano cartesiano, em que a é um número real não nulo.
Nas alternativas abaixo, assinale a equação do lugar PARTE 2
geométrico dos pontos P : (x, y) cuja distância à reta que
passa por A e B, é igual à distância à reta que passa por A e 60. A solução da equação sen7x + cos 2x = −2 é:
B, é igual à distância de P ao ponto C. 3 2k
a) x = + , k
a) x2 + y2 – 2xy – 2ay + 3a2 = 0 14 7
b) x2 + y2 + 2xy + 2ax + 2ay + 3a2 = 0 
b) x = + k , k 
c) x2 + y2 – 2xy + 2ax + 2ay + 3a2 = 0 2
d) x2 + y2 – 2xy – 2ax – 2ay – 3a2 = 0  2k
e) x2 + y2 + 2xy – 2ax – 2ay – 3a2 = 0 c) x= + , k
14 7
3  k
47. São dados sobre uma reta XY três pontos A, B e C (B d) x= + , k
14 7
entre A e C); achar o lugar geométrico da interseção das

tangentes traçadas de A e C aos círculos tangentes à XY no e) x = + 2k , k 
2
ponto B.
61. O valor de arcsen(1 / 3) + arcsen(3 / 4) é:
48. Com base nos lados de um paralelogramo e
externamente a este são construídos 4 quadrados. Prove  7 −6 2  7 +6 2
a) arcsen   b) arccos  
que os centros desses quatro quadrados formam um  12   12 
quadrado.
 2 14 − 3 
c) 120º d) arcsen  
49. ABCD é um quadrado. Se A(1;2) e B(2;5). Determine as  12 
coordenadas de C e D.  2 14 − 3 
e) arccos  
 12 
TRIGONOMETRIA

62. Prove que para 0  x  , cos2 x + xsenx  2 .
3 2
50. Demonstre que sin 6 x + cos6 x = 1 − sin 2 2x .
4

MATEMÁTICA – Prof. DOUGLAS OLIVEIRA biblioteca


telegram t.me/bibliotecaPE @PROFDOUGLASOLIVEIRA _2020 30
APROVADO DE A A Z
TURMA IME ITA
63. Asoma das soluções da equação
x x 1 70. Numa progressão geométrica real, sabe-se que a soma
arctg − arctg = arctg é:
2 3 5 dos seus quatro primeiros termos é 15 e a soma de seus
a) 3 b) 4 c) 5 d) 6 e) 8 quadrados é 85. O produto desses quatro termos é:
a) 36 b) 64 c) 81 d) 96 e) 128
64. Resolva a equação
71. Julgue as afirmativas a seguir em relação à equação
arcsen(x 3) = arcsen(2x) − arcsen(x) .
(x ) − 3(x )
2
2
− 3x − 2 2
− 3x − 2 − 2 − x = 0 :
65. No intervalo [0, 2] o número de soluções da equação
tg2x +4sen2x = 3 é: (I) As raízes da equação x2 – 4x – 2 = 0 são raízes da
a) 2 b) 3 c) 4 d) 6 e) 8 equação inicial;
(II) O produto das raízes da equação inicial é 8;
1 + tg x (III) Todas as raízes da equação inicial são reais.
66. A solução da equação = 1 + sen 2x , onde k  , é:
1 − tg x
a) x = k Agora marque a alternativa que apresenta a seqüência
b) x = 2k obtida:
 a) F – F – F b) V – F – V c) V – V – F
c) x = k − ou x = k d) V – V – V e) F – V – V
4

d) x = k − 72. Sendo z um número complexo, julgue os itens a seguir
4
em verdadeiro (V) ou falso (F):
k
e) x =
2
(I) Se z =2, os pontos 3z encontram-se na
67. (ITA-07) Seja x um número real no intervalo circunferência de raio 6 e centro na origem.
0  x  / 2 . Assinale a opção que indica o comprimento do (II) Se z  1 , os pontos 1 + 2 .
z encontram-se num
menor intervalo que contém todas as soluções da círculo de raio dois e centro no ponto (0, 1).
desigualdade (III) O lugar geométrico dos pontos para os quais
1    x 1 2  z  3 é uma coroa circular de raios interno e
tg − x − 3  cos 2 −  sec(x)  0
2  2   2 2
externo 2 e 3, respectivamente.
a)  / 2 b)  / 3 c)  / 4
d)  / 6 e)  / 12 Agora marque a alternativa que apresenta a seqüência
obtida.
68. (ITA-07) Assinale a opção que indica a soma dos a) F – F – F b) V – F – V c) V – V – F
elementos de A  B , sendo d) V – V – V e) F – V – V

  k2   

A = xk = sen 2   : k = 1,2 e 73. De um depósito de contém 729 litros de ácido puro,

  24  
 foram extraídos a litros e o depósito recompletado com
 (
 3k + 5   )  água.Depois de misturados até que a solução se tornasse
B =  y k = sen 2   : k = 1,2 . homogênea, foram novamente extraídos a litros da solução
  24  
e recompletado com água, misturando bem ao final. Depois
de repetir 6 vezes essa mesma operação, a solução no
a) 0 depósito continha 64 litros de ácido puro. O valor de a, em
b) 1 litros, é:
c) 2 a) 256 b) 243 c) 216 d) 128 e) 81
d)
2− 2+ 3  / 3
 
1+ 5
74. Sendo x + x −1 = , o valor de x2000 + x−2000 é:
 
e) 2 + 2 − 3 / 3 2
  2000
 1+ 5  1 1
a)   b) c)
69. Sabendo que 0     , calcule o valor máximo de  2  2 22000
 d) 2 e) 2 2000
sen  (1 + cos ) .
2

ÁLGEBRA I

MATEMÁTICA – Prof. DOUGLAS OLIVEIRA biblioteca


telegram t.me/bibliotecaPE @PROFDOUGLASOLIVEIRA _2020 31
APROVADO DE A A Z
TURMA IME ITA
75. Sendo a, b e c são as raízes
x − x − x − 1 = 0 , julgue os itens a seguir em verdadeiro
3 2
da equação
( ) ( ) ( ) (
P x5 + xQ x5 + x2R x5 = x4 + x3 + x2 + x + 1 S x )()
(V) ou falso (F): prove que x − 1 é um fator de P(x).

1 1 1 81. Quantas são as soluções racionais da equação x2008 +


(I) + + = −1 y2008 = x2007 + y2007 com x e y distintos e não nulos?
a b c
(II) a, b e c são distintas
a 2006 − b2006 b2006 − c 2006 c 2006 − a 2006 82. Sejam a, b e c as raízes de x3 − 9x2 + 11x − 1 = 0 e
(III) + + é um
a−b b−c c−a s = a + b + c . O valor de s4 − 18s2 − 8s é:
número inteiro. a) 0 b) 13 c) −1 d) 24 e) −37

Agora marque a alternativa que apresenta a seqüência


25x2 74
obtida. 83. A soma das raízes reais da equação x2 + =
a) F – F – F b) V – F – F c) V – V – F ( 5 + 2x) 2 49

d) V – V – V e) F – V – V é:
a) 0
76. Assinale a alternativa que apresenta o número complexo b) 1
que possua argumento positivo mínimo dentre os números c) 1/3
complexos z que satisfazem a condição z − 25i  15 : d) 2/7
a) 12 – 16i e) −436/287
b) 12 + 16i
c) – 16 + 12i 84. Seja Q(z) um polinômio do quinto grau, definido sobre o
d) 16 + 12i conjunto dos números complexos, cujo coeficiente de z5 é
e) 15 + 25i igual a 1. Sendo z3 + z2 + z + 1 um fator de Q(z), Q(0) = 2 e
Q (1) = 8, então, podemos afirmar que a soma dos quadrados
dos módulos das raízes de Q(z) é igual a:
()
77. Seja P x = x 3 − 3x + 1 . Encontre o polinômio Q cujas
a) 9 b) 7 c) 5 d) 3 e) 1
raízes são a quinta potência das raízes de P.
85. Encontre as raízes da equação polinomial x4 – 10x3 +

( ) 11x2 – 10x + 1 = 0.
n
78. Sendo 1 + x + x2 = a0 + a1x + a 2x2 + + a 2n x2n uma

identidade em x, o valor de a0 + a2 + a4 + + a2n é: 86. Encontre todos os números complexos z que são solução
2
a) 0 da equação 4z2 + z + k  z = 0 , onde k é um número real
b) 1 dado.
c) 3n
d) 3n + 1 87. A medida da menor área delimitada pelas
n
3 +1 representações geométricas no plano de Argand-Gauss dos
e)
2 subconjuntos A = {z  / z − 2 + i = 3} e
1
79. Analise as proposições a seguir: B = {z  / Im(z) = } , é:
2
4 − 3 3
I) O resto da divisão de 1 + x + x2 + + x100 por x2 − 1 é a) 3 − 3 b)
2
50x + 51.
II) O valor de a para o qual duas das raízes de x3 + ax2 – 4 − 3 3 3  (4 − 3 3)
c) d)
2x + 6 = 0 são simétricas é negativo. 4 2
III) A equação x1993 + 1993x = 1993 possui uma única raiz 3  (4 − 3 3)
e)
real que está no intervalo ]0, 1[. 4
IV) Sendo P(x) um polinômio do 2° grau tal que P(– 1) = 12,
P(0) = 6 e x = 2 é raiz de P(x), então o resto da divisão 88. Sendo z tal que z7 = 1 e z  1, então o valor numérico de
de P(x) por (x – 3) é 0. 1 1 1
z10 + + z30 + + z50 + é:
10 30
z z z50
O número de proposições VERDADEIRAS é:
3 5
a) 0 b) 1 c) 2 d) 3 e) 4 a) 0 b) −1 c) 1 d) e)
2 2

80. Se P(x), Q(x), R(x) e S(x) são polinômios tais que

MATEMÁTICA – Prof. DOUGLAS OLIVEIRA biblioteca


telegram t.me/bibliotecaPE @PROFDOUGLASOLIVEIRA _2020 32
APROVADO DE A A Z
TURMA IME ITA

89. Resolva o seguinte sistema de equações no conjunto dos


números complexos:
PARTE II
z = z = z =1
 1 2 3
a0 a1 a2 an − 1 an
z1 + z2 + z3 = 1
z  z  z = 1 −1 x 0 0 0
 1 2 3 0 −1 x 0
100. Calcule o determinante .
0 0 −1 0
ÁLGEBRA II x 0
0 0 0 −1 x
PARTE I

101. Seja A uma matriz quadrada inversível de ordem 3.


90. Seja A uma matriz nxn e  um autovalor de A. Se f(x)
Seja B a matriz dos cofatores de A. Sabendo que detA = –
é um polinômio, mostre que f(  ) é um autovalor de f(A).
2, calcule detB.

91. Seja J uma matriz real 4x4 tal que J2 = – I. Seja A uma
102. (IME) Calcule o determinante de uma matriz nxn que
matriz real satisfazendo ATJA = J.
possui zeros na diagonal principal e todos os outros
a) Mostre que J não é singular
elementos iguais a 1.
b) Mostre que se  é autovalor de A, então  −1 também é.

103. (IME) Sejam A = (aij )4  2 e B = (b ij )2  4 duas matrizes.


92. Seja A uma matriz real simétrica nxn. Mostre que
todos os autovalores de A são números reais. Mostre que AB não é inversível.

93. Seja A uma matriz real nxn satisfazendo AT = A2 – 2A + 104. Calcule os seguintes determinantes:
2I. Calcule os possíveis autovalores de A. Conclua que A e A
+ 1345I são inversíveis. 1 a1 a2 an
1 a1 + b1 a2 an
94. (IME-01) Seja A uma matriz quadrada tal que A3 = kA. a) 1 a1 a2 + b2 an
Mostre que se k  1 então A + I é inversível.

1 a1 a2 an + bn
95. Seja A uma matriz real nxn de posto (característica) 1.
a) Mostre que A pode ser escrita como abT onde a e b são
matrizes nx1. x a a a
b) Mostre que existe um único escalar   R tal que a x a a
A2 =  A . b) a a x a
c) Mostre que se   1 então I-A é inversível.
a a a x
96. Seja B uma matriz real 20x20 tal que :

bii = 0, 1  i  20
 , mostre que B não é singular. a + b ab 0 0
bij { −1,1}, 1  i , j  20, i  j
 1 a + b ab 0
c) 0 0 a+b
97. Mostre que toda matriz ortogonal 2x2 é uma matriz de ab
rotação ou de reflexão.
0 0 0 a+b

98. Mostre que todas as matrizes (com n>1) do tipo


 1 2 n a 0 0 b
 n+1 n+ 2 2n 0 a b 0
  possuem posto igual a 2.
  d) (ordem 2n)
 
 (n − 1)n + 1 (n − 1)n + 2 n2  0 b a 0
b 0 0 a
99. Mostre que se A é uma matriz real nxn tal que a soma
dos elementos de cada linha é 1, então A – I é singular.

MATEMÁTICA – Prof. DOUGLAS OLIVEIRA biblioteca


telegram t.me/bibliotecaPE @PROFDOUGLASOLIVEIRA _2020 33
APROVADO DE A A Z
TURMA IME ITA
1−b b 0 0 112. Prove que para todo natural n o número 4n + 15n – 1 é
−1 1 − b b 0 divisível por 9.
e) 0 −1 1 − b
b
0 0 −1 1 − b
1 2 2 2 n 
113. Se m e n são números naturais, definimos   como
2 2 2 2  m
f) 2 2 3 2 o número de subconjuntos de {1, 2, ..., n} com exatamente m
elementos.
2 2 2 n a) Prove que, para todos os naturais m e n,
n   n  n +1 
 m +  m + 1 =  m + 1 .
105. (IME) Seja um determinante definido por  1 = 1 e
b) (IME) Demonstre, por indução, o binômio de Newton, ou
1 1 1 1
seja,
−1 2 0 0
 n  n  n
 n = 0 −1 2 0 (x + y)n = xn +   xn −1y +   xn − 2 y 2 + ... +   xn − k y k + ...y n ,
1   2  k
para todo n  .
0 0 −1 2

114. Para todo natural maior que 1, prove que


a) Encontre a fórmula de recorrência. n
 n + 1
b) Calcule a expressão de  n . n!   .
 2 

106. (IME) Dadas as matrizes


115. Prove que se o produto de n  2 números reais
x − 2 0 0   0 −x 0 
positivos é igual a 1, então sua soma é maior ou igual a n.
A= 3 −1 1  e B =  −1 1 1  determine x,
   
 1 0 x + 1  1 0 −1
116. Determine todos os números naturais n tais que 1! + 2!
sabendo que existe uma matriz inversível P tal que + 3! + ... + n! = n2.
A = P−1BP .
n
117. Calcule  k  k! , ou seja, 1 . 1! + 2 . 2! + 3 . 3! ... + n . n!
PRINCÍPIO DE INDUÇÃO FINITA n =1

107. (UNICAMP) Demonstre que n3 – n é múltiplo de 6 para 118. Prove que, se o produto de n  2 números reais
todo natural n. positivos é igual a 1, então sua soma é maior ou igual a n,
determine em que casos ocorre a igualdade.
108. Prove que n7 – n é múltiplo de 7 para todo natural n.
119. Prove que a média aritmética de um conjunto finito de
109. Prove que, para quaisquer naturais m e n, am . an = am+n números positivos maior ou igual a sua média geométrica, e
e (am)n = am.n. determine em que casos ocorre a igualdade.

110. Prove que, para todo n  *, 120. Prove que, para todo inteiro positivo n e todo real x,
n(n + 1) sen nx  n sen x .
a) 1 + 2 + 3 + ... + n =
2
n(n + 1)(2n + 1) 121. Prove que, se a, b e c são números inteiros positivos
b) 1 + 2 + 3 + ... + n =
2 2 2 2
a+ b+c
6  a 2 + b2 + c 2 
distintos, então   aa bbcc .
 a + b + c 
2
 n(n + 1) 
c) 1 + 2 + 3 + ... + n = 
3 3 3

3

 2 
d) 1 + 3 + 5 + ... + 2n – 1 = n2 122. Prove que, para todo x positivo, x  1 ,
xn − 1 n −1
1  1 1
e) 1 + x + x + x + ... + x
2 3 n −1
=
x−1
(se x é diferente de 1)  log
k=1 2  log x 2
k k +1
= 1 −  
 n  log 2x 2
x

111. Prove que se n é um número natural maior ou igual a 4, 123. Calcule a soma a seguir, em função de m e n.
então n! > 2n.

MATEMÁTICA – Prof. DOUGLAS OLIVEIRA biblioteca


telegram t.me/bibliotecaPE @PROFDOUGLASOLIVEIRA _2020 34
APROVADO DE A A Z
TURMA IME ITA
m
 n + 1
 ( −1)
k=0
k
 k  Que bom, encontramos 250 = – 1 (mód. 101)! Agora
podemos elevar os dois lados a qualquer número inteiro
positivo!! Como 2002 = 50 . 40 + 2, temos
124. Uma certa organização tem n membros, e n + 1
comitês distintos de três membros. Prove que há dois
22002 = 250 . 40 + 2 = (250)40 . 22  (– 1)40 . 22  4
comitês com exatamente um membro em comum.

ou seja, 22002  4 (mód. 101). Logo o resto da divisão de


22002 por 101 é 4.
CONGRUÊNCIAS Uma boa estratégia na hora de calcular uma
potência an módulo m é procurar um expoente d tal que
1. Congruência módulo m an   1 (mód. m).

1.1. Definição Exercícios

Dizemos que a é congruente a b módulo m e escrevemos 125. Determine o resto das divisões de
a  b (mód. m) se, e somente se, a e b deixam o mesmo a) 41234 por 3
resto na suas divisões euclidianas por m. b) 20100 por 17
Em particular, se r é o resto da divisão de a por m, então c) 2000 . 22000 – 1 por 3
a  r (mód. m).
126. Encontre o resto da divisão de 520 por 26.
1.2. Propriedades Operacionais
127. Mostre que (8355 + 6)18 – 1 é divisível por 112.
Sejam a  b (mód. m) e c  d (mód. m), então
(1) a + c  b + d (mód. m) 128. Mostre que
(2) a − c  b − d (mód. m) a) 270 + 370 é divisível por 13
(3) na  nb (mód. m) para todo n natural b) 22225555 + 55552222 é divisível por 7
(4) ac  bd (mód. m)
(5) an  bn (mód. m) para todo n natural 129. Em cada casa do tabuleiro de xadrez, há o número de
grãos de trigo indicado, como mostra a figura. O cavalo de
As propriedades operacionais nos dizem que a Bruno & Bernardo começa a se movimentar no tabuleiro, de
congruência módulo m funciona quase como igual, pois dá acordo com as regras usuais, a partir de uma casa qualquer.
para somar, subtrair e multiplicar congruências. Porém Quando ele atinge uma casa, come todos os grãos nela
ainda não podemos dividir. existentes (mas ele não come os grãos da casa inicial).
Uma situação em que podemos dividir é quando Quando ele deixa uma casa, nós recolocamos a mesma
“cancelamos”: quantidade de grãos que nela existiam. Depois de um certo
(6) Se m é inteiro tal que mdc (m; n) = 1, na  nb (mód. m) tempo, o cavalo de Bruno & Bernardo retorna à casa inicial e
↔ a  b (mód. m) come os grãos nela existentes. Prove que o número de grãos
Mas atenção!! Isso só vale quando mdc (m; n) = 1!! que o cavalo de Bruno $ Bernardo comeu durante sua
viagem é divisível por 3.
Vejamos como congruências são úteis:

Exemplo 1.1.

Calcule o resto da divisão de 22002 por 101.

Resolução

Observe que quando falamos em resto da divisão por um


número m podemos pensar em congruências mód m.
Assim, vamos calcular 22002 mód 101.
Temos que uma potência de 2 “próxima” de 101 é 27 =
128. Temos
27  27 (mód.101)  214  729  22 (mód.101)
 2 28  484  −21 (mód.101)
 2 56  441  −64  −26 (mód.101)
 2 50  −1 (mód.101)

MATEMÁTICA – Prof. DOUGLAS OLIVEIRA biblioteca


telegram t.me/bibliotecaPE @PROFDOUGLASOLIVEIRA _2020 35
APROVADO DE A A Z
TURMA IME ITA
Se x = 0, temos 20 = 3y – 1  3y = 2, o que não é possível.
130. Encontre os dois últimos dígitos de Se x = 1, temos 21 = 3y – 1  3y = 3  y = 1, logo x = 1 e y =
a) 7100 1 é uma solução.
b) 22000
c) 5600 + 19200 Se x  2 , temos 2x  0 (mód. 4). Logo devemos ter
d) 72000 x 2300
( –1) y – 1  0 (mód. 4), o que ocorre se, e somente se, y é par.
Assim, y = 2a, onde a é natural e temos

2x = 32a – 1  2x = (3a – 1)(3a + 1)

131.
a) Mostre que 81| 99...9 . Como 3a – 1 e 3a + 1 são divisores de uma potência de 2,
nove noves então 3a – 1 e 3a + 1 são ambas potências de 2. Mas a
b) Determine o menor número da forma 99...9 que é diferença entre elas é (3a + 1) – (3a – 1) = 2, e as únicas
divisível por 17. potências de 2 tão “próximas” assim são 2 e 4.
Conseqüentemente, 3a – 1 = 2  a = 1 e logo y = 2 e x = 3.
132. Prove que, para todo n natural, 37n+2 + 16n+1 + 23n é Enfim, as únicas soluções são x = 3 e y = 2 e x = y = 1.
divisível por 7.
138. Encontre todos os pares de inteiros positivos (x; y) tais
133. Mostre que, para todo n natural, 72 2n+2
– 472n 2n – 1
+ 28 que 2x = 1 + 3y.
é divisível por 25.
139. Encontre todos os inteiros positivos x e y tais que 3x – 2y =
134. Mostre que, para todos k, m, n naturais, 55k+1 + 45m+2 + 7.
35n é divisível por 11.
ANÁLISE
135. Mostre que
a) 215 – 1 e 210 + 1 são primos entre si. 140. Seja f :  → uma função tal que f(1, 1) = 2,
b) 232 + 1 e 24 + 1 são primos entre si.
( ) (
f m + 1, n = f m,n + m ) ( ) (
e f m, n + 1 = f m,n − n ) para
todos os m, n  . Encontre todos os pares (p, q) tais que
136. Mostre que 41 divide 11...1 (onde há 5k dígitos 1, k
inteiro positivo). f(p,q) = 2001.

137. Mostre que 91 divide 11...1 (onde há 6k dígitos 1, k 141. Seja uma função f definida em [0, 1] tal que f(0) = f(1)
inteiro positivo). () ( )
= 1 e f a − f b  a − b , para todo a  b no intervalo [0, 1].

2. Usando congruências para resolver equações com () ( )


Prove que f a − f b 
1
2
.
números inteiros

142. Encontre todos os polinômios não nulos f(z) tais que


As congruências ajudam bastante também na hora de
resolvermos equações em inteiros (chamamos tais equações
de diofantinas).
( ) ()(
f z2 + f z f z + 1 = 0 .)

Exemplo 2.1. 143. Seja f : → uma função tal que f n + 1  f n( ) ( ) e

Encontre todos os números naturais x e y tais que 2x = 3y – 1.


( ( ))
f f n = 3n para todo n. Calcule f(2001).

Resolução 144. Suponha que f : + → + é uma função decrescente


tal que para todos os x, y  + ,
A primeira dúvida que poderia surgir é “que módulo vamos
usar?” Lembra que uma estratégia era obter algo congruente ( ) ( ( ) ( ))
f x+y +f f x +f y =f f x+f y +f y+f x (( ( )) ( ( ))) . Prove

que f ( f ( x ) ) = x .
a  1? Aqui não é diferente. Podemos, por exemplo, ver a
congruência módulo 4, que é pertinho de 3 e é uma potência
de 2:
145. Encontre todas as funções f: → tais que
2 = 3 − 1  2  3 − 1 (mód. 4)  2  ( −1) − 1 (mód. 4)
x y x y x y
( ) ( ) () ( )
f x + y + f x − y = 2f x + 2f y para todos os x,y  .

MATEMÁTICA – Prof. DOUGLAS OLIVEIRA biblioteca


telegram t.me/bibliotecaPE @PROFDOUGLASOLIVEIRA _2020 36
APROVADO DE A A Z
TURMA IME ITA
x3 + x − 1
146. Encontre todas as funções f : → tais que d)
2x(x + 1)
( ( ) ( )) ( ( ))2
f xf x + f y = f x + y , para todos os x,y  .
e)
x3 − x + 1
2x(x + 1)
147. Determine todas as funções f: R → R tais que f(x) =
f(–x) e f(x + y) = f(x) + f(y) + 8xy + 115 para todos os reais 154. A solução da desigualdade
x e y. ( logx 2)  ( log2x 2)  ( log2 4x)  1 é:
a) − 2  x  2
148. Se a equação ax2 + (c + b)x + (e + d) = 0 possui raízes
reais maiores que 1, mostre que a equação b) 2− 2  x  2 2

ax 4 + bx 3 + cx 2 + dx + e = 0 possui pelo menos uma raiz real. c) 2 −2  x  2 −1 ou 1  x  22


d) 2− 2  x  2−1 ou 1  x  2 2
149. Sendo  = log12 18 e  = log 24 54 , o valor de
e) 0  x  2 − 2 ou 1  x  2 2
  + 5  ( − ) é:
a) 0 b) 1 c) 2 d) 3 e) 5 155. Assinale a alternativa que apresenta todas as soluções
x
 1 da equação 3x  8 x + 2 = 6 é:
150. As raízes da equação loga (ax)  log x (ax) = log 2   , a) 1
a  a
b) 1 e − log 3 6
onde a > 0 e a  1, têm para sua soma :
5 1+ a c) 1, − log 3 6 e − log 3 36
a) 1 b) c)
2 a 2 d) − log 3 6 e − log 3 36
1+ a 1+ a a e) 1 e − log 3 36
d) e)
a2 a2
156. Quatro carros A, B, C, D viajam a velocidades
151. Supondo que x é um número real, x  0 e x  3, não é constantes na mesma estrada. A ultrapassa B e C às 8 horas
um possível valor da expressão: e 9 horas, respectivamente, e encontra D às 10 horas; D
encontra B e C às 12 horas e 14 horas, respectivamente.
x −x (
x−3 − x−3 ) Determine a que horas B ultrapassa C.
+
x x−3 a) 10: 20 h b) 10: 30 h c) 10: 40 h
a) –4 d) 11:00 h e) 11:20 h
b) −2
c) 0
DIVERSOS
d) 2
e) todos os valores acima são valores possíveis da
b
expressão 157. Dada a relação = 2 + 3 entre os catetos de um
c
152. Resolvendo-se a equação triângulo retângulo ABC, calcular os ângulos agudos desse
triângulo.
6x + 8 1 − x 2 = 5 ( 1+ x + 1− x ) no intervalo
3 
 , 1 ,
5  158. Em um triângulo retângulo ABC, a hipotenusa mede 5
encontra-se para x o valor: 1
a) 0,64 b) 0,76 c) 0,88 d) 0,92 e) 0,96 cm e tem lugar a relação senB = senC . Calcular os
2
catetos e o raio do círculo inscrito.
153. A expressão da função f que satisfaz à equação
 1  159. A área de um triângulo retângulo é de 144 m2 e um dos
f(x) + f  = x , x  1 , é:
 1 − x  ângulos é o triplo do outro. Calcular os lados do triângulo.
x3 − x − 1
a) 160. Demonstrar que é retângulo o triângulo ABC no qual
2x(x − 1)
existe uma das seguintes relações:
x3 + x + 1 a) b = a . sen B
b)
2x(x − 1) b) a2 cos (B − C) = 2 bc
x3 − x + 1 B
c) c) 2a sen 2 = a − c
2x(x − 1) 2

MATEMÁTICA – Prof. DOUGLAS OLIVEIRA biblioteca


telegram t.me/bibliotecaPE @PROFDOUGLASOLIVEIRA _2020 37
APROVADO DE A A Z
TURMA IME ITA
161. Em um triângulo retângulo ABC, exprimir a 173. Demonstrar que em qualquer triângulo ABC subsiste a
tg (A + B − 2C) em função dos catetos b e c. (B − C)
b − c tg 2
relação: = .
162. Calcular os ângulos agudos B e C de um triângulo b+c (B + C)
tg
retângulo ABC, sabendo que 3(tg B − 1) = 1 − tg C. 2
1
174. Num triângulo ABC são conhecidos cos A = ,
163. Num triângulo retângulo ABC, a hipotenusa mede 3 m 3
e os ângulos agudos B e C satisfazem a relação: cos B = −
1
e o raio do círculo inscrito r = 2cm. Calcular o
3 sen 2B = 2 sen C 5
Calcular os catetos desse triângulo. lado AB.

164. Num triângulo ABC, a altura traçada do vértice A 175. Demonstrar que em qualquer triângulo ABC subsistem
determina no lado BC os segmentos CD = m e BD = n. as relações:
Demonstrar: a) a(b cos C − cos B) = b2 − c2
m−n sen(A − B) a 2 − b2
sen(B − C) =  sen A b) =
m+n sen(A + B) c2
165. Num círculo de diâmetro AB = 2R traçam-se a corda
B+C B−C 2c A
AC e a corda AD que bissecta o ângulo BÂC. Determinar o c) tg − tg =  cot g
2 2 b+c 2
ângulo BÂC = x de modo que se tenha.
176. As diagonais de um paralelogramo são 2m e 2n e um
AC + AD = R (1 + 3) ângulo é  . Achar a área S.

166. Num triãngulo ABC, tem-se: A = 45º, b = 4 e c = 2 . 177. Demonstrar que é isósceles ou retângulo o triângulo
Calcular o seno e o cosseno de cada um dos ângulos B e C. ABC no qual subsiste uma das seguintes relações:
a) a cos A = b cos B
167. Demonstrar que é isósceles o triângulo ABC no qual b) a2 tg B = b2 tg A
existe uma das seguintes relações:
a) a = b cos B + c cos C 178. ABCD é um quadrilátero convexo no qual A = 105°, B
b) a = 2b cos C = 60°, D = 120° e cuja diagonal AC = 12 cm. Calcular a
b2sen B + c 2sen C diagonal BD.
c) = bc sen B  sen C
b+c
179. Os lados de um triângulo ABC estão em progressão
3
168. Os lados de um triângulo medem 5m, 6m, e 9m. aritmética e a sua área é igual a daquela de um triângulo
Determinar: 5
a) a área do triângulo eqüilátero de mesmo perímetro. Demonstrar que os lados do
b) a área do círculo inscrito triângulo ABC são proporcionais aos números 3, 5, 7 e achar
c) as tangentes dos ângulos internos o maior ângulo desse triângulo.

169. O ângulo sob o qual um observador vê uma torre 180. M é um ponto de uma semi-circunferência de diâmetro
duplica pelo fato de ter se aproximado 110m e triplica AB = 2R e P é a projeção desse ponto sobre AB. Determinar
quando ele se aproxima mais 50m. Calcular a altura da o ângulo MÂB = x de modo que se tenha:
torre. 3 1 3
+ =
MA MB MP
170. Demonstrar que em qualquer triângulo ABC subsiste a
B−C b−c A 181. Em qualquer circunferência, a corda que subtende o
relação: sen =  cos
2 a 2 arco de 108° é igual à soma das cordas que subtendem os
arcos de 36° e 60°. Prove.
171. Num triângulo ABC, tem-se: cotg A = 2 e cotg B = 3.
Calcular a altura relativa ao menor lado em função do lado 182. Demonstrar que é eqüilátero o triângulo ABC no qual
cuja medida está compreendida entre aquelas dos outros subsistem conjuntamente as relações:
dois lados. b3 + c 3 − a 3 3
= a 2 e sen B  sen C =
b+c−a 4
172. Calcular os lados de um triângulo ABC, conhecendo os
ângulos B = 60°, C = 45° e a área S = 25(3 − 3) 183. Num triângulo ABC subsiste a relação:
2 A
a 2 − b 2 − c 2 = bc . Calcular tg .
3 2

MATEMÁTICA – Prof. DOUGLAS OLIVEIRA biblioteca


telegram t.me/bibliotecaPE @PROFDOUGLASOLIVEIRA _2020 38
APROVADO DE A A Z
TURMA IME ITA
184. Em um programa de auditório, o convidado deve p
q sejam inteiros. Necessariamente devemos ter p = 1; se
escolher uma dentre três portas. Atrás de uma das portas
não vejamos: suponha que p  2 e que q = sp para algum
há um carro e atrás de cada uma das outras duas há um
bode. O convidado ganhará o que estiver atrás da porta; inteiro positivo s; daí sP < p + q < (s + 1)P
devemos supor neste problema que o convidado prefere (pois (s + 1)p – sp > p . sp – 1  p) e não teremos
p
p + q sendo
ganhar o carro. O procedimento para escolha da porta é o
um inteiro, o que nos obriga fazer p = 1.
seguinte: o convidado escolhe inicialmente, em caráter
provisório, uma das três portas. O apresentador do
190. Sejam três pontos A, B e C pertencentes a uma
programa, que sabe o que há atrás de cada porta, abre
 
neste momento uma das outras duas portas, sempre circunferência de centro O tais que AOB < BOC. Seja D o
revelando um dos dois bodes. O convidado agora tem a ponto médio do arco AC que contém o ponto B. Seja K o pé
opção de ficar com a primeira porta que ele escolheu ou
da perpendicular a BC por D. Prove que AB + BK = KC.
trocar pela outra porta fechada. Que estratégia deve o
convidado adotar? Com uma boa estratégia, que
191. Prove que existe uma seqüência a0, a1, …, ak, …, onde
probabilidade tem o convidado de ganhar o carro?
cada ai é um algarismo (ou seja, ai  {0, 1, 2, 3, 4, 5, 6, 7, 8,
9}) com a0 = 6, tal que para cada inteiro positivo n o número
xn = a0 + 10a1 + 100a2 + … + 10n–1 an–1 (cuja representação
decimal é an–1 an–2 …a1a0) é tal que x2n − xn é divisível por 10n.

185. Um móvel tem três gavetas iguais. Em uma gaveta há


duas bolas brancas, em outra há duas bolas pretas, e na 192. Seja A = {x1 < x2 <…< xn} um conjunto de números
terceira há uma bola branca e outra preta. Abrimos uma inteiros positivos tal que se x e y são dois números naturais
gaveta ao acaso e tiramos uma bola ao acaso sem olhar a que não pertencem a A então x + y não pertence a A. Provar
segunda bola que está na gaveta. A bola que tiramos é que xi  2i – 1 para i = 1, 2,…, n.
branca. Qual é a probabilidade de que a segunda bola que
ficou sozinha na gaveta seja também branca? 193. Considere a seqüência (xn) n  N definida por x1 = 19,
 1
x − , se xn +1  0
186. Mostre que se a, b e c são inteiros ímpares, a equação x2 = 98 e, para todo n  N, xn + 2 =  n xn − 1 .
0, se x
ax 2 + bx + c = 0 não tem raiz racional.  n +1 = 0
Prove que existe n  N tal que xn = 0 e encontre o menor n
187. Mostre que, dado um conjunto de n pessoas, existem com essa propriedade.
duas que possuem o mesmo número de amigos entre as
pessoas do conjunto. 194. Sejam ABC um triângulo, M o pé da bissetriz interna
do ângulo A e N o pé da bissetriz interna do ângulo B.
1 1 1 Suponha que MN seja bissetriz do ângulo AMC. Calcule o
188. Determine o número de soluções de + = com x
x y 1998 ângulo A.
e y inteiros positivos.
195. Ache todas as soluções reais de
y
189. Determine todas as soluções de x = y com x e y x
[x] +  1998 x  = 1998 ([y]denota o único inteiro tal que
 
racionais positivos.
[y]  y  [y] + 1 ).
i) É evidente que x = y  Q+ satisfaz a equação.
x
ii) Suponha x  y, seja então = a (a  Q+), logo: 196. Mostre que o produto de todos os números da forma
y
 1  2  3  ...  100 é o quadrado de um número
1
p p+q inteiro.
ay = ya → y = a a −1 e, fazendo a − 1 = , a = , temos
q q
 q Lemas de Kaplansky
  p + q p
y = 
  q  De quantos modos é possível formar um p-subconjunto (isto
 q+p é, um subconjunto com p elementos) de {1, 2, ..., n} no qual

x =  p + q  p não haja números consecutivos? Por exemplo, para n = 6 e p
  q  = 3, podemos obter a partir de {1, 2, 3, 4, 5, 6} os seguintes

3-subconjuntos nos quais não há elementos consecutivos: {1,
Já que p e q são primos entre si, p + q e q também são
3, 5} {1, 3, 6} {1, 4, 6} {2, 4, 6}
p
primos entre si, e portanto devemos garantir que p+q e

MATEMÁTICA – Prof. DOUGLAS OLIVEIRA biblioteca


telegram t.me/bibliotecaPE @PROFDOUGLASOLIVEIRA _2020 39
APROVADO DE A A Z
TURMA IME ITA
Lema 1: O numero de p-subconjuntos de {1, 2, ..., n} nos segmentos que eles determinam tem comprimento menor ou
quais não há números consecutivos é F(n, p) = Cn – p + 1, p. igual a 2.

197. Três provas devem ser realizadas na primeira semana Solução:


do ano. De quantos modos é possível escolher os dias das Divida o quadrado de lado 2 em quatro quadrados de lado 1.
provas de modo que não haja provas em dias consecutivos? Dos 5 pontos, pelo menos dois pertencerão a um mesmo
quadrado de lado 1. A distância entre esses dois pontos
198. Uma fila tem 15 cadeiras nas quais devem sentar-se 5
será no máximo igual à diagonal do quadrado que é 2, o
homens, de modo que não fiquem dois homens sentados em
que conclui a demonstração.
cadeiras contíguas. De quantos modos isso pode ser feito?

200. Depois de ter dado um curso, um professor resolve se


Suponhamos agora que o problema é determinar de quantos
despedir de seus 7 alunos oferecendo, durante 7 dias
modos é possível formar um p-conjunto de {1, 2, ..., n} no
consecutivos, 7 jantares para 3 alunos cada. De quantos
qual não haja números consecutivos, supondo que 1 e n são
modos ele pode fazer os convites se ele não deseja que um
consecutivos.
mesmo par de alunos compareça a mais de um jantar?

Lema 2: O numero de p-subconjuntos de {1, 2, ..., n } nos


Suponha que n carros estão em fila para entrar em um
quais não há números consecutivos é, considerando 1 e n
estacionamento que possui n vagas, lado a lado. Se o 1o carro
como consecutivos, igual a G(n, p) = [n/(n – p)].Cn – p, p.
pode escolher qualquer vaga e cada um dos outros carros ao
estacionar deve justapor-se a um carro já estacionado,
quantos são os modos possíveis dos carros ocuparem as n
vagas?
2n – 1

Princípio de Dirichlet

199. Hugo deve ter aula de tênis três vezes na semana,


durante um semestre. Quantos são os modos de escolher os
Permutação Caótica
dias de aula, se Hugo não deseja ter aulas em dias
consecutivos?
Uma permutação é chamada caótica quando numa seqüência
de a1a2,a3,...,an nenhum dos ai se encontra na sua posição
Também conhecido como Princípio das Gavetas de Dirichlet
original, isto é, na i-ésima posição.
ou como Princípio das Casas dos Pombos (PCP) é uma
Desta forma a2a1a5a3a4 e a5a4a1a2a3 são exemplos de
poderosa ferramenta para determinar a existência ou não
permutações caóticas tendo ai no i-ésimo lugar.
de conjuntos satisfazendo a certas propriedades.
Se definirmos por Ai o conjunto das permutações de a1,a2, ,
Princípio das Gavetas de Dirichlet: “Se n objetos forem
... , an, tendo ai no i-ésimo lugar, para calcularmos o número
colocados em no máximo, n – 1 gavetas então pelo menos
de permutações caóticas, denotados por Dn (a letra D vem
uma delas conterá pelo menos dois objetos.”
da palavra desarranjo, palavra sinônima de permutação
caótica), devemos calcular o numero de elementos no
Demonstração:
complementar da união dos Ai’s . Logo
Suponhamos, por absurdo, que cada uma das gavetas possui
n
no máximo um objeto. Assim o número total de objetos Dn = n! −  nAi + .... + (−1)n.n.(A1A2... An)
nelas colocados será no máximo, n – 1 que é um absurdo i =1
Como existem n termos na primeira soma, Cn, 2 termos na
Exemplos segunda, Cn, 3 na terceira , ... , Cn, n na última e
N(Ai) = (n −1)!
1. Dado um conjunto de 13 pessoas, prove que pelo menos N( AiAj) = (n − 2)!
duas delas aniversariam no mesmo mês Solução: Como ................. = ...........
existem 12 meses durante o ano, e temos 13 pessoas, pela n(AiA2A3...An) = 1
PCP temos que pelo menos duas delas fazem aniversário no
mesmo mês. Temos
Dn = n! – n!/1! + n!/2! − ... + (− 1)n.(n!/n!)
2. Escolhem-se 5 pontos ao acaso sobre a superfície de um Colocando-se n! em evidencia obtemos:
quadrado de lado 2. Mostre que pelo menos um dos Dn = n!(1 − 1/1! + 1/2! – 1/3! + ... (− 1)n.1/n! )

MATEMÁTICA – Prof. DOUGLAS OLIVEIRA biblioteca


telegram t.me/bibliotecaPE @PROFDOUGLASOLIVEIRA _2020 40
APROVADO DE A A Z
TURMA IME ITA
201. Determine o número de anagramas da palavra ESCOLA 214. (USAMO-78) Dado que a, b, c, d, e são números reais
de modo que nenhuma letra ocupa a posição original. tais que
a + b + c + d + e = 8, a2 + b2 + c2 + d2 + e2 = 16.
202. Quantas permutações dos inteiros 1, 2, 3, 4, ..., 8, 9 , Determine o máximo valor de e.
10 tem exatamente 4 dos números em suas posições
originais? 215. (Rússia-62) São dados quatro números positivos a, b,
c, d, e sabe-se, que abcd = 1. Prove que
203. Determinar o número de permutações caóticas de a1, a2 + b2 + c2 + d2 + ab + ac + ad + bc + bd + dc  10.
a2, a3, ..., a8 com a condição de que os 4 primeiros objetos
sejam transformados no conjunto{a5 ,a6 ,a7 ,a8} em alguma 216. Mostrar que 41 divide 220 – 1.
ordem
217. Achar os restos das divisões de 250 e 4165 por 7.
204. Seja n um inteiro positivo e sejam a1, a2, …, an números
reais  1. Mostre que 377 − 1
n 218. Mostre que é ímpar e composto.
2 2
(1 + a1 )(1 + a 2 )...(1 + a n )  (1 + a1 + a 2 + ... + a n )
n+1
219. Achar o resto da divisão de 111...111 (com 1995
205. Mostre que, para todos os reais positivos p, q, r, s, dígitos 1) por 1001.
(p2 + p + 1)(q2 + q + 1)(r2 + r + 1)(s2 + s 1)  81pqrs
220. Para quais dos seguintes valores de n 1n + 2n + 3n + 4n
206. Se a, b, c são os comprimentos dos lados de um é divisível por 5?
triângulo, mostre que: a) 1980 b) 1988 c) 1990 d) 1992 e) 2000
3(bc + ca + ab)  (a + b + c)2 < 4(bc + ca + ab)
221. Achar o resto quando 314162 é dividido por 163.
207. Se x, y, z são números positivos, mostre que:
x2 y2 z2 y z x 222. Sendo p e q dois inteiros positivos e se f é uma função
+ +  + + . definida nos números positivos que assume somente valores
y2 z2 x2 x y z
positivos e tal que f(xf(y)) = (xp).(yq). Prove que q
= p2.
208. Suponha que 0  xi  1 para i = 1, 2, …, n. Prove que:
2n – 1(1 + x1x2…xn)  (1 + x1)(1 + x2)…(1 + xn). 223. (Iberoamericana-90) Seja f uma função, definida no
209. Se S = x1 + x2 + … + xn, onde xi > 0 (i = 1, 2, …, n), prove conjunto dos inteiros maiores ou iguais que zero, que
que: verifica as duas condições seguintes:
S S S n2 •(I) Se n = 2j – 1, para n = 0, 1, 2, ... , então f(n) = 0
+ + ... +  .
S − x1 S − x2 S − xn n − 1 •(II) Se n  2j – 1, para n = 0, 1, 2, ... , então f(n + 1) = f(n) –
1.
210. Se a, b, c, d são números reais positivos, prove que:
a) Demonstrar que para todo inteiro n, maior ou igual que
2 2 2 2 2 2 2 2 2 zero, existe um inteiro k, maior que zero, tal que: f(n) +
a +b +c b +c +d c +d +a
+ + + n = 2k – 1
a+b+c b+c+d c+d+a , b) Calcular f(21990).
d2 + a 2 + b2
+ a+b+c+d
d+a+b 224. (IMO) Find all positive real solutions to:
Como igualdade se a = b = c = d.
(x12 - x3x5)(x22 - x3x5) ≤ 0
211. Mostre que, se k é um inteiro não negativo: (x22 - x4x1)(x32 - x4x1) ≤ 0
a) 12k + 22k + 32k  2.7k; (x32 - x5x2)(x42 - x5x2) ≤ 0
b) 12k + 1 + 22k + 1 + 32k + 1  6k + 1. (x42 - x1x3)(x52 - x1x3) ≤ 0
Quando ocorre a igualdade? (x52 - x2x4)(x12 - x2x4) ≤ 0

212. Suponha que x, y, z são números reais não negativos.


225. (IMO) a and b are real numbers for which the
prove que:
equation x4 + ax3 + bx2 + ax + 1 = 0 has at least one real
8(x3 + y3 + z3)  9(x2 + xy)(y2 + xz)(z2 + xy).
solution. Find the least possible value of a2 + b2.

213. Se a, b, c são os comprimentos dos lados de um


226. (AIME-90) Calcule ax5 + by5 se os números reais a,
triângulo, prove que:
b, x e y satisfazem as equação:
abc  (a + b – c)(b + c – a)(c + a – b).
ax + by = 3, ax2 + by2 = 7, ax3 + by3 = 16, ax4 + by4 = 42.

MATEMÁTICA – Prof. DOUGLAS OLIVEIRA biblioteca


telegram t.me/bibliotecaPE @PROFDOUGLASOLIVEIRA _2020 41
APROVADO DE A A Z
TURMA IME ITA
227. For how many reals b does x2 + bx + 6b have only
integer roots?

228. A hexagon is inscribed in a circle. Five sides have


length 81 and the other side has length 31. Find the sum of
the three diagonals from a vertex on the short side.

229. ABCD is a rectangle. P, Q, R, S lie on the sides AB,


BC, CD, DA respectively so that PQ = QR = RS = SP. PB =
15, BQ = 20, PR = 30, QS = 40. Find the perimeter of
ABCD.
We have y = 180o - 3x, so cos y = - cos 3x = 3 cos x - 4
cos3x = 9/5 - 108/125 = 117/125, and sin y = 44/125. Hence
AP = 117/5, AS = 44/5.

GABARITO

MATEMÁTICA I

01.
16 − x 2  x
a) − − arcsen   + c
x  4

x2 − 9 1  3 
b) − arctg   +c
2 54
18x  x −9
2

5 − x2
c) − +c
5x
 
d) In  x + x2 − 7  + c
 

230. O is the center of the circle. AC = BD = 78, OA = 42, 1  x 


e) In   +c
OX = 18. Find the area of the shaded area. 
5  5 + 25 − x 2 

3 1
f) − arcsen(1 − x) − (x + 3) 2x − x 2 + c
2 2
3

g) −
( 16 − 9x2 ) 2
+c
5
80x
x−2
h) +c
4 4x − x2
x 2  
i) x + 2 + In  x + x2 + 2  + c
2  
2  x 2 
j) arctg   +c
2 2 2 2 2 2
OY = OA - AY , so OY = 9√9. XY = OX - OY , so XY = 9 2  1 − x2 
and ∠OXY = 60o. Hence DX = AX = AY + XY = 48, BX = CX =
 
30. Also ∠AXB = 180o - 2·60o = 60o. So area triangle AXB = 2 1 + x2 + x 2
k) In   +c
½AX·BX sin 60o = 360√3. ∠ACB = ∠XBC and their sum is 4  1 + x2 − x 2 
∠AXB = 60o, so ∠ACB = 30o. Hence ∠AOB = 60o. So area
sector OAB = 422π/6 and area between AB and minor arc x2 − 4
l) +c
AB = 294π - 422(√3)/4. Hence required area = 294π - 81√3. 4x
7 2x2 + 9
m) +c
8 4x2 + 9
1 1
n) x 1 + x2 + x 2 + arcsen(x) + c
2 2

MATEMÁTICA – Prof. DOUGLAS OLIVEIRA biblioteca


telegram t.me/bibliotecaPE @PROFDOUGLASOLIVEIRA _2020 42
APROVADO DE A A Z
TURMA IME ITA
11. C 12. D 13. C 14. D 15. B 16. E 17.
02. 18.
1
a) arcsen(2x − 2) + c
2 MATEMÁTICA III
1 3  3
b) − 3x2 − x + 1 + In  3x2 − x + 1 + 3x −  +c
3 18  6  2 3
19. 4 2 20. a 21. 36m3
2
2  7(3x + 8)  3 + 2x  
c) − − 6 7arctg  +c 2 −1
49  x2 + 3x + 4  7   22. 23. 24. 25.
2
 2x + 3  7 3
d) arg senh  +c
 11  6
 
e) In  2x − 1 + 2 x 2 − x − 1 + c a 2
  26. 54 2 27. 28.
2
5 9  8x + 3 
f) 4x 2 + 3x + 1 + arg sinh  +c (3 − 4)R 3
4 16  7 
3
g) −arcsen(2 − x) 29. D 30. D 31. E 32. B
 x − 1 a 34
h) 2 3 + 2x + x 2 + arg sen  +c 33. 34. Fk = 0 para k > 3
 2  24
3  2x − 3 
i) 4 − 3x + x 2 + arg senh   +c
2  7 
MATEMÁTICA IV
 x + 3
j) 2
x + 6x + 34 − arg senh  +c
 5 
35. D 36. 37. 2 38. A 39. E

03. 40.
1  (x + 2)2  3  x − 1 D (2, −2) e S = 25−15 ou
a) In  + arctg  +c
24  x 2 − 2x + 4  12  3  D (2, 4) e S = 25−6 ou
D (10, 4) e S = 25−20
 x −1
b) In  − 2arctg(x) + c
 x + 1  41. D 42. E 43. 44. 45. A 46. A
47. 48. 49.
c)
1
2
() ( )
In x2 + 2 +
2
1
x +2 2
+c
MATEMÁTICA V
d) In ( x + 1) −
1 1 2
+c
2
2 x +1 a
50. Demonstração 51. 52. 1/8
1 2
a +1
e) − − arctg(x) + c
x
  1
f) In  x2 + 1 − arctg(x) −
x
+c 1
( )
  2 2
53. 4 54. 
2 x +1 2

 x3 − x2 + x  3 2  2x − 1  
g) In  − + arctg  +c 55. 2k , k + , (2k + 1)
 (x + 1)2  x + 1 3  3  2 5

h) −
1
2x2
1
(
− In(x) + In x2 + 1 + c
2
) 56. k

x+31
i) − − arctg(x + 2) + c  
( 2
2 x + 4x + 5 2 ) 57. x = k +
3
, y = − k +
6
 x 
j) x + In   +c  4
 58. x = 2k + , y = 2k  +
x2 + 1  3 3

MATEMÁTICA II k  (2k + 1) 


59. − , +
11 66 12 36
04. E 05. D 06. A 07. B 08. C 09. E 10. D

MATEMÁTICA – Prof. DOUGLAS OLIVEIRA biblioteca


telegram t.me/bibliotecaPE @PROFDOUGLASOLIVEIRA _2020 43
APROVADO DE A A Z
TURMA IME ITA
 1 1 127. 128. 129. 130. 131. 132. 133.
60. E 61. E 62. 63. C 64.  − , 0,  134. 135. 136. 137. 138. 139.
 2 2
65. C 66. C 67. D 68. C 69.
MATEMÁTICA iX

MATEMÁTICA VI
140. (2000, 1999) ou (1001, 999)

70. B 71. D 72. B 73. B 74. D 75. D 76. B


141.

( )
77. Q x = x 3 + 15x 2 − 198x + 1
142. −zm z − 1 ( )m para m 

78. E 79. E 80. 81. infinitas 82. E 83. D


84. B 143. 3816

144.
1 i 3 9  77
85. x = ou x =
2 2
( )
145. f x = kx 2 , k 
1
86. z = − 87. E 88. B
k+4 146. f(x) = x e f(x) = −x

89. 1, i e −i em alguma ordem 147. 148. 149. B 150. E 151. D 152. E 153.
C
MATEMÁTICA VII 154. D 155. E 156. C

90. 91. 92. 93. 94. 95. 96. MATEMÁTICA X


97. 98. 99.
157. 15° e 75°
100. a0 x + a1xn n −1
+ ... + an
3 5 −5
158. b = 5;c= 2 5; r=
101. 4 2

159.
102. (n − 1)( −1)n−1
1) a = 8 6 3 ; b = 4 6 3 ; c = 12 2 3
103.
a) b1b2 ...b n
2) a = 24 2 ; b = 12 2 ( )
2 + 1 ; c = 12 2 ( 2 −1 )
b) [x + (n − 1)a](x − a)n −1
160.
a n + 1 − bn + 1
c)
a−b
d) (a2 − b2 )n 161.
( 3b − c ) c
2 2

e) 1 − b1 + b1b2 − b1b2 b3 + ... + ( −1)n b1b2 ...bn ( 3c − b ) b


2 2

f) 1
162.
104. 1) b = 45° e c = 45°;
a) n = 2n −1 + n −1 2) B = 30° e C = 60°
b)  n = 2n − 1
163. b = 1 m ; c = 2 2m

105. S =  164.

MATEMÁTICA VIii 165. x = 60°

106. 107. 108. 109. 110. 111. 112. 166.


113. 114. 115. 116. 117. 118. 119. 2 5 10
120. 121. 122. 123. 124. 125. 126. sen B = ; sen C = ;
5 10

MATEMÁTICA – Prof. DOUGLAS OLIVEIRA biblioteca


telegram t.me/bibliotecaPE @PROFDOUGLASOLIVEIRA _2020 44
APROVADO DE A A Z
TURMA IME ITA
− 5 3 10 O erro comum aqui é achar que, após a eliminação de uma
cos B = ; cos C = porta (que foi aberta pelo apresentador, revelando um
5 10
bode), há uma simetria entre as duas outras portas e a
probabilidade de cada uma esconder o carro é 1/2. Não
167.
existe, entretanto, tal simetria, pois a porta escolhida pelo
convidado não poderia, pelas regras, ser trocada pelo
168.
apresentador, enquanto a outra poderia ter sido aberta,
a) 10 2m2 ; mas não foi.
b) 2 m2;
10 2 4 2 Este processo de fato era seguido em um programa nos
c) tg A = ; tg B = ; tg C = −2 2
23 7 Estados Unidos. Uma longa e áspera discussão ocorreu na
imprensa quanto a qual era o valor correto da probabilidade,
169. 88m e pessoas que deveriam ser capazes de resolver um
problema trivial como este passaram pela vergonha de
170. publicar soluções erradas. Julgamos melhor esquecer os
detalhes deste episódio deprimente.
a 2
171. h b = 185. A resposta correta é 2/3 (e não 1/2). As seis bolas
2
seriam de início igualmente prováveis, mas sabemos que a
primeira bola escolhida foi branca: assim, as três bolas
172. a = 10; b = 5 2 (3 − 3 ); c = 10( 3 − 1).
brancas têm igual probabilidade. Estamos interessados em
saber a cor da companheira de gaveta de cada bola branca:
173.
em dois casos é branca, em um caso é preta. Assim, a
probabilidade de que a segunda bola seja branca é 2/3,

174.
(
2 2 3+ 3 ) cm como já afirmamos.
3 Um raciocínio comum, mas errado, é dizer: as gavetas são
igualmente prováveis, mas obviamente não escolhemos a
175. gaveta que contém duas bolas pretas. Portanto, teríamos
probabilidade 1/2 de termos escolhido a gaveta com duas
176. S=(m2 − n2) tg  bolas brancas e 1/2 de termos escolhido a gaveta com uma
bola de cada cor; no primeiro caso, a segunda bola é branca
177. e, no segundo caso, a bola é preta. Assim, a resposta seria
1/2.

(
178. 2 6 1 + 3 cm ) O que há de errado neste raciocínio? O erro está em dizer
que as duas gavetas possíveis são igualmente prováveis.
179. 120° Inicialmente a probabilidade de cada gaveta é de fato a
mesma (inclusive para a gaveta com duas bolas pretas), mas,
180. x = 30° ao tirarmos uma bola e constatarmos que ela é branca, isto
deixa de ser verdade. Isto é bem óbvio para a gaveta com
181. duas bolas pretas: passou a ser impossível termos escolhido
esta gaveta. Entre as duas outras gavetas, entretanto, há
182. uma diferença que está sendo ignorada no raciocínio do
parágrafo anterior. Se pré-escolhermos a gaveta com duas
183. 2 bolas brancas, temos certeza de passar no teste: uma bola
escolhida ao acaso nesta gaveta será sempre branca. Por
outro lado, se pré-escolhermos a gaveta com uma bola de
184. A resposta correta é que, trocando de porta, a
cada cor, ainda temos probabilidade 1/2 de sacarmos uma
probabilidade de ganhar o carro é 2/3, enquanto não
trocando a probabilidade é apenas 1/3. Uma forma simples bola preta, o que estaria em contradição com o enunciado.
Assim, a probabilidade de termos escolhido cada uma
de ver isto é a seguinte: trocando de porta, o convidado
destas duas gavetas é 2/3 e 1/3, respectivamente.
ganha, desde que a primeira porta que ele escolher esconda
um dos dois bodes, como se pode facilmente perceber. A Podemos, a partir deste ponto facilmente deduzir a
resposta correta de 2/3.
melhor estratégia para o convidado é, portanto, trocar
sempre, e assim sua probabilidade de ganhar fica sendo
2/3. É fato empírico desencorajador que muitas pessoas teimam
em dizer que a probabilidade é 1/2 mesmo após esta
explicação. O seguinte exemplo serve como exercício para
aqueles que entenderam a explicação e é uma espécie de

MATEMÁTICA – Prof. DOUGLAS OLIVEIRA biblioteca


telegram t.me/bibliotecaPE @PROFDOUGLASOLIVEIRA _2020 45
APROVADO DE A A Z
TURMA IME ITA
redução ao absurdo do raciocínio "rival". Temos novamente = conjunto dos possíveis números de amigos de uma
três gavetas, uma com vinte bolas brancas, uma com vinte determinada pessoa em P, ou seja:
bolas pretas e a terceira com dez bolas de cada cor. P = { p1, p2, …, pn–1}
Abrimos uma gaveta e, sem olhar, retiramos ao acaso dez Q = { 1, 2, 3, …, n – 2}
bolas: elas são todas brancas. Qual a probabilidade de que Observe que há n – 2 valores no conjunto Q para n – 1
as dez bolas restantes sejam também brancas? valores em P; isto quer dizer que  n1, n2  P tais que
f ( n1 ) = f ( n2 ).
p
186. Imaginemos que o número racional seja raiz da Segunda Hipótese: Suponha que todas as n pessoas tenham
q
amigos entre si, ou seja:
equação ax 2 + bx + c = 0 onde a, b e c são inteiros ímpares. P = { p1, p2, …, pn} e Q = { 1, 2, 3, …, n – 1}
Logo, fazendo a substituição, devemos ter, Observe que agora o conjunto Q possui n – 1 valores, pois
cada pessoa de P possui no mínimo 1 amigo e no máximo (n –
 p
2
 p 1) amigos entre as (n – 1) pessoas restantes. Pelo mesmo
a  + b  + c = 0 motivo da primeira hipótese  n1, n2  P tais que
 q  q
f ( n1 ) = f ( n2 ).

p2 p Conclusão: há pelo menos duas pessoas com a mesma


a +b +c=0
q 2 q quantidade de amigos.

ap2 + bpq + cq2 = 0

Vamos acrescentar agora uma hipótese importante para


p
facilitar nosso trabalho. Vamos supor que a nossa fração
q
188.
seja irredutível, ou seja, que ela já foi simplificada ao
4
máximo. Por exemplo, no lugar de estaremos Temos 1998x + 1998y = xy .
6
Somando 19982 dos dois lados temos
2 xy – 1998x – 1998y + 19982 = 19982, logo
considerando o que é a mesma coisa. Consideramos
3 x(y – 1998) – 1998(y – 1998) = 19982, donde
então, para a solução do problema, que p e q não são ambos (x – 1998) (y – 1998) = 19982.
pares.
Desta forma o número de soluções é o mesmo que a
Observe agora a equação ap2 + bpq + cq2 = 0 nos seguintes quantidade de sistemas da forma abaixo que possamos
casos: obter:

a) p e q são ímpares: neste caso, ap2 é ímpar, bpq é x − 1998 = a



ímpar e cq 2 é ímpar. Como a soma de três números  y − 1998 = b
 2
ímpares é ímpar, o resultado não pode ser zero. ab = 1998
b) p é par e q é ímpar: neste caso, ap2 é par, bpq é par
com a observação de que os pares (x, y) solução devam ser
e cq 2 é ímpar. Como a soma de dois números pares e
inteiros e positivos, devemos ter
um ímpar é ímpar, o resultado não pode ser zero.
c) p é ímpar e q é par: vale o mesmo argumento do a + 1998  0 
 a  −1998
caso b).  

 b + 1998  0 b  −1998

Demonstramos então que nenhuma fração de numerador e
denominador inteiros pode ser raiz da equação logo, só servem a e b positivos, já que se –1998< a < 0 e –
1998 < b < 0 implica ab < 19982. O número de soluções é,
ax + bx + c = 0 onde a, b e c são inteiros ímpares.
2
portanto, o número de divisores positivos de 19982 = 22. 36.
372, que é dado por
187. Primeira Hipótese: há apenas uma única pessoa sem
(2 + 1) (6 + 1) (2 + 1) = 63.
amigos; logo entre as n – 1 pessoas restantes, cada pessoa é
amiga de no mínimo uma pessoa e no máximo n – 2 pessoas.
189.
Seja f: P → Q onde P = conjunto das pessoas restantes e Q

MATEMÁTICA – Prof. DOUGLAS OLIVEIRA biblioteca


telegram t.me/bibliotecaPE @PROFDOUGLASOLIVEIRA _2020 46
APROVADO DE A A Z
TURMA IME ITA
190. 192. Suponhamos que o enunciado é falso, ou seja que
existe k tal que

xk > 2k – 1, 1  k  n.
Então os conjuntos

B1 = {1, xk – 1}, B2 = {2, xk – 2}, …, Bk = {k, xk – k}

são disjuntos dois a dois e seus elementos são menores que


xk .
Além disso, para cada j, 1  j  k, j  A ou xk – j  A, pois
Sejam AB = x, BD = y; marcamos D' tal que D'C = y. Então
no caso contrario, ou seja, se j  A e xk – j  A, teríamos
D'D = x por ser D ponto médio de AC e resulta DD' // BC.
que xk = j + (xk – j)  A.
Se K' é o pé da perpendicular a BC por D', então temos
Portanto, para cada j, 1  j  k, A  Bj  , donde A tem
AB = DD' = KK' e BK = K'C
pelo menos k elementos menores que xk , absurdo.
AB + BK = KK'+ K'C = KC.

193. Se xn+1  0, temos xn+2 xn+1 = xn+1 xn –1. Definindo


191.
yn = xn xn+1 temos
O primeiro termo é a0 = 6; então x1 = 6 e
yn+1 = yn–1 para todo n tal que xn+1  0. Como y 1 = x1 x2 = 19 
1
x12 − x1 = 36 − 6 = 30 , que é divisível por 10 . 98 = 1862, temos yk = 1863 – k enquanto yk –1 for diferente
Seja n  1. Suponhamos que existem a0, a1,…,an–1 tais que de 0, e portanto y1862 = 1 e y1863 = 0  x1862 x1863 = 1 e x1863
xn = a0 + 10a1 + 102 a2 + ... + 10n−1an−1 x1864 = 0. Assim, x1863  0 e x1864 = 0, donde 1864 é o menor n
tal que xn = 0.
verifica que x2n − xn é divisível por 10n (ou seja
x2n n
− xn = 10 r, com r N)
AN c
194. Pelo teorema das bissetrizes, = e
Temos que encontrar an tal que NC a
xn+1 = a0 + 10a1 + 102 a 2 + ... + 10n−1a n−1 + 10n a n = xn + 10n a n BM c ab
=  MC = , e como MN é bissetriz de
MC d b+c
ˆ devemos ter MA = AN = c , donde MA = bc
seja tal que x2n+1 − xn+1 é divisível por 10n+1. AMC
MC CN a b+c
ab
(pois MC = pela lei dos senos aplicada aos
x 2n +1 − x n +1 = (x n + 10 n a n )2 − (x n + 10 n a n ) = b+1
x 2n + 2x n 10 n a n + 10 2n a 2n − x n − 10 n a n = triângulos ABC e ABM temos
ac
= (x 2n − x n ) + 10 n (2x na n − a n ) + 10 2n a 2n = senA a b + c BM sen(A / 2)
= = = = , e portanto sen
10 n r + 10 n (2x na n − a n ) + 10 2n a n2 = senB b bc MA senB
= 10 n (r + 2x na n − a n ) + 10 2n a 2n . b+c
(A/2) = sen A = 2 sen (A/2) cos (A/2)  cos
 A 1 A  2
Assim,  2  =  = A= .
2 2 3 3
x 2n +1 − x n +1 e divisível por 10n +1  r + 2xna n − a n é
divisível por 10  195.
r + (2x n − 1)a n é divisível por 10. 
 x  +  1998x  é sempre inteiro. Seja x0 a solução de

Dado que a0 = 6, temos que xn = 10t + 6 com t  N; então 2xn x + 1998x = 1998, ou seja x0 = 999 (3 –
= 10h + 2. (com h = 2t + 1). Logo r + (2xn – 1)an é divisível por
5) = 763,1...
10  r + (10h + 1)an é divisível por 10  r + an é divisível por
10. e 1998x0 = 1998 − x0 = 999( 5 − 1) = 1234,8... Temos
Sempre existe um único inteiro an entre 0 e 9 de modo tal
x  +  1998x0  = 1997.
que isto se verifique.  0   

Obs: A seqüência (an) começa por 6, 7, 3, 9, 0, 1, 7, 8, 7, 1,


8, 0, 0, 4, 7, 3… A função f(x) =  x  +  1998x  aumenta de uma unidade
Assim, por exemplo, x10 = 1787109376.
quando x ou 1998x torna-se inteiro. Os próximos valores

MATEMÁTICA – Prof. DOUGLAS OLIVEIRA biblioteca


telegram t.me/bibliotecaPE @PROFDOUGLASOLIVEIRA _2020 47
APROVADO DE A A Z
TURMA IME ITA
de x maiores que x0 para os quais x e 1998x são inteiros 201. Como temos 6 letras distintas, basta aplicar a fórmula
são respectivamente 764 e 12352 / 1998 < 764. para n = 6: D6 = 6!(1 – 1/1! + 1/2! – ... + 1/6!) = 265
Assim, f (12352 /1998) = 763 + 1235 = 1998 e f (764) =
764 + 1235 = 1999 202. Como não são fixados os quatro números que
permanecem nas posições originais, devemos escolher estes
(de fato 1998  764  1236). Como f (x) é não-
4 números, o que pode ser feito de C10, 4 maneiras distintas,
decrescente, o conjunto das soluções é o intervalo e, em seguida permutar os seis restantes caoticamente.
 12352  Logo a resposta é: C10, 4.D6 = C10, 4.6!(1 – 1/1! + ... + 1/6!) =
 ,764 = 763,3758758758...,764 . ) 55.650
 1998 

203. A resposta e 4!.4!, pois uma permutação que leva os


196. O número referido no enunciado é o quadrado do
quatro primeiros nos 4 últimos deverá levar os 4 últimos nos
produto de todos os 299 números da forma
quatro primeiros, e, com esta restrição, nunca haverá o
1  2  3  ...  100 (no produto do enunciado cada um perigo de algum elemento ser fixado.
desses números aparece uma vez, assim como seu
simétrico). Neste último produto, obtemos uma soma de 204.
termos do tipo 205.
206.
207.
( , a1 ,)(2 a 2 )...(m a m ), com m  299 ,
208.
a1 ,a 2 ,...a m {2,3,...,100} e  j {-1,1},j. 209.
210.
211.
Fixamos  2 ,  3 ,..., 100 N com  2 +  3 + ... + 100  299 , e
212.
consideramos todos os termos como acima que têm 213.
exatamente  k valores de a j = k, para 2  k  100 . Se 214.
todos os  j são pares esses termos são todos inteiros. Se 215. 216. 217. 218.

algum deles (digamos  r ) é ímpar, podemos associar de


219.
forma bijetiva a cada termo desses o termo obtido
trocando os sinais de todos os  j para os quais a j = r. I) 111...11 = 101994 + 101993 + 101992 + ... + 102 + 10 + 1
Assim, a cada termo associamos o seu simétrico, e portanto, II) 1  1 (mod. 1001) 10  10 (mod. 1001) 102  102
3
nesse caso a soma dos termos considerados é 0. Assim, o (mod. 1001) 10  – 1 (mod. 1001)
produto de todos os números da forma 104  – 10 (mod. 1001) 105  – 100 (mod. 1001) 106
7
 – 1 (mod. 1001) 10  – 10 (mod. 1001)
1  2  3  ...  100 é um inteiro, e portanto o
108  – 100 (mod. 1001) e assim por diante. Como
produto do enunciado é um quadrado perfeito.
1994 = 3.664 + 2, então:
1 + 10 + 102 + 103 + 104 + 105 + 106 + 107 + 108 + ... + 101993
197. Devemos formar um subconjunto de três elementos
+ 101994  1 + 10 + 102 – 1 – 10 – 102 – 1 – 10 – 102 – ... –
no conjunto dos sete dias da primeira semana, de modo que
102 (mod. 1001) 
não haja dias consecutivos no subconjunto. A resposta é
111...11  663(– 111) (mod. 1001)  111...11  – 73593
F(7, 3) = C7 – 3 + 1, 3 = C5, 3 = 10
(mod. 1001)
Como 73593 = (73).(1001) + 520  67064  520
198. Devemos inicialmente escolher 5 cadeiras sem que
(mod. 1001)  111...11  520 (mod. 1001)
haja cadeiras consecutivas. Isso pode ser feito de
F(15, 5) = C15 – 5 + 1, 5 = C11, 5 modos. Escolhidas as 5 cadeiras,
220.
devemos designar a cada homem uma cadeira, o que pode
ser feito de P5 = 5! modos. A resposta é C11, 5.5! = 55440.
I) 4  – 1 (mod. 5)  4n  (– 1)n (mod. 5)
II) 3  – 2 (mod. 5)  3n  (– 2)n (mod. 5)
199. Hugo deve escolher 3 dos elementos do conjunto
Domingo, Segunda, ..., Sábado, não podendo escolher dois
Então: 1n + 2n + 3n + 4n  1 + 2n + (– 2)n + (– 1)n (mod. 5) 
dias consecutivos e sendo o Domingo e o Sábado dias
1n + 2n + 3n + 4n  [1 + (– 1)n] + [2n + (– 2)n] (mod. 5)
consecutivos. O numero de modos a fazer isto é
Se n é ímpar, então temos que 1n + 2n + 3n + 4n é divisível por
G (7, 3) = [7/(7 – 3)].C7 – 3, 3 = 7.
5.
Se n é da forma n = 4k, temos: 1n + 2n + 3n + 4n  1 + 1 + 16k +
200.
16k (mod. 5) 
1n + 2n + 3n + 4n  2 + 2.16k (mod. 5)  1n + 2n + 3n + 4n 
2(1 + 16k) (mod. 5)

MATEMÁTICA – Prof. DOUGLAS OLIVEIRA biblioteca


telegram t.me/bibliotecaPE @PROFDOUGLASOLIVEIRA _2020 48
APROVADO DE A A Z
TURMA IME ITA
Como 16k sempre termina em 6, então 16k + 1 sempre If the second largest is x5, then we can use the last
termina em 7, implicando que (1 + 16k) nunca vai ser inequality to deduce that x2 = x4 = x5 and proceed as
divisível por 5. before.
Se n é da forma n = 4k + 2, temos: 1n + 2n + 3n + 4n  1 + 1 +
42k + 1 + 42k + 1 (mod. 5)  If the second largest is x3, then the fourth inequality gives
1n + 2n + 3n + 4n  2 + 4.42k + 4.42k (mod. 5)  1n + 2n + 3n + 4n that x1 = x3 = x5 or x1 = x3 = x4. In the first case, x5 is the
 2 + 8.16k (mod. 5)  second largest and we are home already. In the second
1n + 2n + 3n + 4n  2(1 + 4.16k) (mod. 5) case, the third inequality gives x32 = x2x5 and hence x3 = x2
Como 16k sempre termina em 6, implica que 4.16k sempre = x5 (or one of x2, x5 would be larger than the second
termina em 4, implicando que 1 + 4.16k termine em 5, largest). So x5 is the second largest and we are home.
implicando, finalmente, que 2(1 + 4.16k) termine em 0, ou
seja, 1n + 2n + 3n + 4n é divisível por 5 se n deixar resto 2
Finally, if the second largest is x4, then the second
quando dividido por 4.
inequality gives x1 = x2 = x4 or x1 = x3 = x4. Either way, we
have a case already covered and so the numbers are all
221.
equal.

222. Aplicando x = 1 obtemos f(f(y)) = yq


225. Put y = x + 1/x and the equation becomes
Aplicando y = 1 e fazendo f(1) = k, onde k é um número
y2 + ay + b - 2 = 0, which has solutions y = -a/2 ±√(a2 + 8 -
positivo, obtemos f(kx) = xp
2b)/2. We require |y| ≥ 2 for the original equation to have
a real root and hence we need |a| + √(a2 + 8 - 4b) ≥ 4.
Aplicando f no segundo resultado: f(f(kx)) = (kx)q = kqxq
Squaring gives 2|a| - b ≥ 2. Hence a2 + b2 ≥ a2 + (2 - 2|a|)2 =
f(kx) = xp => f(f(kx)) = f(xp) = f(k.(k – 1.xp)) = (k – 1.xp) p =
5a2 - 8|a| + 4 = 5(|a| - 4/5)2 + 4/5. So the least possible
k − px p
2
value of a2 + b2 is 4/5, achieved when a = 4/5, b = -2/5. In
Igualando as duas expressões: kq xq = k − pxp para todo x
2
this case, the original equation is x4 + 4/5 x3 - 2/5 x2 + 4/5
positivo. x + 1 = (x + 1)2(x2 - 6/5 x + 1).
Deve-se então, necessariamente, ter q = p2 e k = 1.
226. This needs a trick: (axn+1 + byn+1)(x+y) - (axn + byn)xy =
axn+2 + byn+2. Hence 7(x+y) - 3xy = 16, 16(x+y) - 7xy = 42.
223. We claim that if 2m <= n < 2m+1, then f(n) = 2m+1 - n - 1. Solving x+y = -14, xy = -38. Applying again: ax5 + by5 =
Put r = 2m+1 - n. Then the claim follows by induction on r. 42(x+y) - 16xy = 20.
Hence f(21990) = 21990 - 1.
227. If the roots are integral, so is their sum, so b must
224. Answer: x1 = x2 = x3 = x4 = x5. be an integer. The roots must be real, so b2 - 24b >= 0, and
b (b - 24) must be a square. b = 0, 24 clearly work.
Otherwise, put B = b-24 for b > 24, or -b for b < 0. Then if
The difficulty with this problem is that it has more
d = gcd(B, B+24), d must be one of 1, 2, 3, 4, 6, 8, 12, 24
information than we need. There is a neat solution in
and B/d and (B+24)/d are squares. Since the gap between
Greitzer which shows that all we need is the sum of the 5
consecutive squares increases, it is easy to check that the
inequalities, because one can rewrite that as (x1x2 - x1x4)2 +
only solutions are:
(x2x3 - x2x5)2 + ... + (x5x1 - x5x3)2 + (x1x3 - x1x5)2 + ... + (x5x2 -
x5x4)2 ≤ 0. The difficulty is how one ever dreams up such an
idea! b = -25, x = -5, 30 b = -8, x = -4, 12
b = -3, x = -3, 6
b = -1, x = -2, 3
The more plodding solution is to break the symmetry by
b = 0, x = 0, 0
taking x1 as the largest. If the second largest is x2, then
b = 24, x = -12, -12
the first inequality tells us that x12 or x22 = x3x5. But if x3
b = 25, x = -15, -10
and x5 are unequal, then the larger would exceed x1 or x2.
b = 27, x = -18, -9
Contradiction. Hence x3 = x5 and also equals x2 or x1. If
b = 32, x = -24, -8
they equal x1, then they would also equal x2 (by definition
b = 49, x = -42, -7
of x2), so in any case they must equal x2. Now the second
inequality gives x2 = x1x4. So either all the numbers are
equal, or x1 > x2 = x3 = x5 > x4. But in the second case the 228. Put AC = x, AE = y, AD = z. Note that the chord
last inequality is violated. So the only solution is all lengths depend only on the arc lengths, so for example,
numbers equal. CF = z. Applying Ptolemy to ABCD: 31·81 + z·81 = xy, and to
ACDF: xy + 812 = z2. Hence (z-144)(z+63) = 0, so z = 144.
Now Ptolemy on AFED gives 812 + 81z = y2, so y = 135. Now
the first equation gives x = 105. So x + y + z = 384.

MATEMÁTICA – Prof. DOUGLAS OLIVEIRA biblioteca


telegram t.me/bibliotecaPE @PROFDOUGLASOLIVEIRA _2020 49
APROVADO DE A A Z
TURMA IME ITA
c) 36;

(ITA-71) Qual o maior número de partes em que um plano


pode ser dividido por n linhas retas?
(Sugestão: usar indução finita).
a) n2; d) (n2 + n + 2)/2;
b) n(n + 1); e) N.d.r.a.
c) n(n + 1)/2;

(ITA-72) Sejam A um conjunto finito com m elementos e In


= {1, 2, ..., n}. O número de todas as funções definidas em In
com valores em A é:
a) C nm d) mn
229. b) m.n e) nenhuma das respostas
anteriores
230. c) nm

(ITA-72) Sejam m  n, Im = {1, 2, ..., m} e In = {1, 2, ..., n}. O


número de funções biunívocas definidas em Im com valores
em In é:
a) A nm d) m.n
COMBINATÓRIA b) C n
e) nenhuma das respostas
m
anteriores
NÍVEL l c) m!/n!

(ITA) De quantos modos se pode pintar um cubo, usando (ITA-77) Consideremos m elementos distintos.
seis cores diferentes, sendo cada face de uma cor? Destaquemos k dentre eles. Quantos arranjos simples
5.3! = 30 daqueles m elementos tomados n a n (Am, n) podemos formar,
de modo que em cada arranjo haja sempre, contíguos e em
(ITA-57) Uma urna contém 12 bolas das quais 7 são pretas qualquer ordem de colocação, r (r < n) dos k elementos
e 5 brancas. De quantos modos podemos tirar 6 bolas da destacados?
urna, das quais duas são brancas? 350 a) (n – r – 1)Ak, rAm – k, n – r b) (n – r + 1)Ak, rAm – r, n – k
c) (n – r – 1)Ak, rAm – r, n – k d) (n – r + 1)Ak, rAm – k, n – r
(ITA-65) O número de todas as diagonais de um octógono é e) nenhuma das anteriores
dado pela fórmula:
(ITA-80) O número de soluções inteiras e não negativas da
a) C n2 − n , n = 8 d) An2 − n , n = 8 equação x + y + z + w = 5 é:
b) C n2+1 , n = 8 e) nda a) 36 b) 48 c) 52 d) 54 e) 56

c) 2n – n/2, n = 8
(ITA-81) Se p1, p2, ..., pn forem fatores primos de um

p = p1 1 p2 2 ... pn
s s sn
(ITA-69) Resolvendo a equação C15, x −1 = C15, 2 x +1 , onde número inteiro positivo p e se ,
então o número de divisores de p será:
Cm,p significa que o número da combinações simples (sem
a) s1 + s2 + ... + sn d) (s1 + 1)(s2 + 1) ... (sn +
repetição) de m elementos tomados p a p, obtemos:
1) – 1
a) x = –2 e x = 5 d) x = 2
b) s1s2 ... sn e) (s1 + 1)(s2 + 1) ... (sn
b) x = 2 e x = –2 e) nda
+ 1)
c) x = 2 e x = 5
c) s1s2 ... sn – 1
(ITA-71) Dispomos de seis cores diferentes.
(ITA-83) Um general possui n soldados para tomar uma
Cada face de um cubo será pintada com uma cor diferente,
posição inimiga. Desejando efetuar um ataque com dois
de forma que as seis cores sejam utilizadas. De quantas
grupos, um frontal com r soldados e outro da retarguarda
maneiras diferentes isto pode ser feito, se uma maneira é
com s soldados (r + s = n), ele poderá dispor seus homens
considerada idêntica a outra, desde que possa ser obtida a
de:
partir desta por rotação do cubo?
a) 30; d) 18; n!
a) maneiras distintas neste ataque.
b) 12; e) N.d.r.a. ( r + s)!
MATEMÁTICA – Prof. DOUGLAS OLIVEIRA biblioteca
telegram t.me/bibliotecaPE @PROFDOUGLASOLIVEIRA _2020 50
APROVADO DE A A Z
TURMA IME ITA
n! Você concluiu que:
b) maneiras distintas neste ataque. a) apenas I é verdadeira d) todas são
r ! s! verdadeiras
n! b) apenas II e III são verdadeiras e) todas são falsas
c) maneiras distintas neste ataque.
( rs)! c) apenas III é verdadeira

2( n !) (ITA-94) Quantos anagramas com 6 caracteres distintos


d) maneiras distintas neste ataque.
( r + s)! podemos formar usando as letras da palavra QUEIMADO,
anagramas estes que contenham duas consoantes e que,
2(n !)
e) maneiras distintas neste ataque. entre as consoantes, haja pelo menos uma vogal?
r ! s! a) 7200 b) 7000 c) 4800 d) 3600 e) 2400

(ITA-95) Considere todos os números de cinco algarismos


(ITA-87) Quantos números de 3 algarismos distintos formados pela justaposição de 1, 3, 5, 7 e 9 em qualquer
podemos formar, empregando caracteres 1, 3, 5, 6, 8 e 9? ordem, sem repetição. A soma de todos esses números está
a) 60 b) 120 c) 240 d) 40 e) 80 entre
a) 5 x 106 e 6 x 106
(ITA-87) O número de arranjos de n + 2 objetos tomados
b) 6 x 106 e 7 x 106
cinco a cinco vale 180n. Nestas condições, concluímos que:
c) 7 x 106 e 8 x 106
a) n é um número par
b) n é um número primo d) 9 x 106 e 10 x 106
c) n está compreendido entre 100 e 200 e) 10 x 106 e 11 x 106
d) n é um número par
e) n é divisível por 5 (ITA-96) Três pessoas, A, B e C, chegam no mesmo dia a
uma cidade onde há cinco hotéis H1, H2, H3, H4 e H5.
(ITA-88) Considere (P) um polígono regular de n lados. Sabendo que cada hotal tem pelo menos três vagas,
Suponha que os vértices de (P) determinem 2n triângulos, qual/quais das seguintes afirmações, referentes à
cujos lados não são lados de (P). O valor de n é: distribuição das três pessoas nos cinco hotéis, é/são
a) 6 b) 8 c) 10 d) 20 e) Não existe correta(s) ?
polígono regular com esta propriedade. (I) Existe um total de 120 combinações diferentes.
(II) Existe um total de 60 combinações se cada pessoa
(ITA-91) Uma escola possui 18 professores sendo 7 de pernoitar num hotel diferente.
Matemática, 3 de Física e 4 de Química. De quantas (III) Existe um total de 60 combinações se duas e apenas
maneiras podemos formar comissões de 12 professores, de duas pessoas pernoitarem no mesmo hotel
modo que cada uma contenha exatamente 5 professores de a) Todas as afirmações são verdadeiras.
Matemática, no mínimo 2 de Física e no máximo 2 de b) Apenas a afirmação (I) é verdadeira.
Química? c) Apenas a afirmação (II) é verdadeira.
a) 875 b) 1877 c) 1995 d) 2877 e) n.d.a d) Apenas as afirmações (I) e (II) são verdadeiras.
e) Apenas as afirmações (II) e (III) são verdadeiras.
(ITA-93) Possuo 3 vasos idênticos e desejo ornamentá-los
com 18 rosas, sendo 10 vermelhas e 8 amarelas. Desejo que (ITA-98) O número de anagramas da palavra
um dos vasos tenha 7 rosas e os outros dois no mínimo 5. VESTIBULANDO, que não apresentam as cinco vogais
Cada um deverá ter 2 rosas vermelhas e 1 amarela, pelo juntas, é:
menos. Quantos arranjos distintos poderei fazer usando as a) 12! b)(8!)(5!) c) 12! – (8!)(5!) b) 12! – 8!
18 rosas? e) 12! – (7!)(5!)
a) 10 b) 11 c) 12 d) 13 e) 14
(ITA-99) Listando-se em ordem crescente todos os
(ITA-93) Analisando as afirmações classificando-as em números de cinco algarismos distintos, formados com os
verdadeiras ou falsas: elementos do conjunto {1, 2, 4, 6, 7}, o número 62417 ocupa
I. O número de maneiras que podemos distribuir 5 prêmios o n-ésimo lugar. Então n é igual a:
iguais a 7 pessoas de modo que cada pessoa premiada a) 74 b) 75 c) 79 d) 81 e) 92
receba no máximo um prêmio é 21.
II. O número de maneiras que podemos distribuir 5 NÍVEL II
prêmios iguais a sete pessoas de modo que 4 e apenas 4 28
(IME) Se 2 = n , calcule n. 6
5
sejam premiadas é 140.
Cn+
3
 n  n 
III. Para todo natural n, n  5,   = . (IME-65) Dados 20 pontos do espaço, dos quais não
 5  n − 5 existem 4 coplanares, quantos planos ficam definidos?

MATEMÁTICA – Prof. DOUGLAS OLIVEIRA biblioteca


telegram t.me/bibliotecaPE @PROFDOUGLASOLIVEIRA _2020 51
APROVADO DE A A Z
TURMA IME ITA
1140 Marcos - 20 Flávio - 10 Ralph - 9
Sabe-se que Flávio venceu a segunda partida. Encontre
(IME-66) Determinada organização estabeleceu um sistema quantos pontos cada um marcou em cada partida.
de códigos em que os símbolos são formados por um ou mais
pontos, até o máximo de 6 pontos, dispostos de maneira a (IME-90/91) Um batalhão de n soldados fará uma viagem
ocuparem os vértices e os pontos médios dos lados maiores de trem. No trem há n cabines numeradas de 1 a n. De
de um retângulo. Qual o número total de símbolos obtidos. quantas maneiras diferentes os soldados poderão ocupar as
63 cabines de modo que só a cabine 1 fique vazia.

(IME-84/85) Dois clubes do Rio de Janeiro participaram de (IME-90/91) Dado o conjunto A = {1, 2, 3, …, 102}, pede-se
um campeonato nacional de futebol de salão onde cada o número de subconjuntos de A, com três elementos, tais
vitório valia um ponto, cada empate meio ponto e cada que a soma destes seja múltiplo de três.
derrota zero ponto. Sabendo que cada participante
enfrentou todos os outros apenas uma vez, que os clubes do (IME-91/92) Sejam n, k  Z*. Chama-se potência fatorial
Rio de Janeiro totalizaram, em conjunto, oito pontos e que de ordem k do número n o inteiro nk obtido através da
cada um dos outros clubes alcançou a mesma quantidade k função
de pontos, determine a quantidade de clubes que participou
do torneio. n(n + 1)...(n + k − 1) para k  1
nk = 
n para k = 1
(IME-84/85) Um exame vestibular se constituiu de 10
provas distintas, 3 das quais da área de Matemática. Demonstre que: (n + 1) k +1 − n k +1 = (n + 1) k ( k + 1) .
Determine de quantas formas é possível programar a
sequência das 10 provas, de maneira que duas da área de (IME-92/93) Quantos números ímpares de 4 algarismos
Matemática não se sucedam. pode-se formar em pregando os algarismos 6, 2, 0, 3 e 9?

(IME-85/86) 12 cavaleiros estão sentados em torno de uma (IME-92/93) Numa escola há 15 comissões, todas com igual
mesa redonda. Cada um dos 12 cavaleiros considera seus número de alunos. Cada aluno pertence a duas comissões e
dois vizinhos como rivais. Deseja-se formar um grupo da cada duas comissões possui exatamente um membro comum.
cinco cavaleiros para libertar uma princesa. Nesse grupo Todos os alunos participam. a) Quantos alunos tem a escola?
não poderá haver cavaleiros rivais. Determine de quantas b) Quantos alunos participam de cada comissão?
maneiras é possível escolher esse grupo.
(IME-93/94) Seja um octógono convexo. Suponha que
(IME-87/88) Considere um conjunto de 12 letras distintas, quando todas as suas diagonais são traçadas, não há mais de
sendo 8 consoantes e 4 vogais; duas diagonais se inteceptando no mesmo ponto. Quantos
a) quantas palavras podem ser formadas contendo 3 pontos de interseção (de diagonais) existem neste
consoantes e 2 vogais sem repetição?; b) em quantas destas octógono?
palavras as vogais não estão juntas? a) 5!C8, 3C4, 2; b)
24192. (IME-95/96) Determine os números naturais n para os
quais existam poliedros convexos de n arestas.
IME(88/89) Em cada uma das faces de um cubo constrói-se
um círculo e, em cada círculo marcam-se n pontos. Unindo-
se esses pontos, (IME-95/96) É dado um tabuleiro quadrado 4x4. Deseja-se
a) quantas retas, não contidas numa mesma face do cubo, atingir o quadrado inferior direito a partir do quadrado
podem ser formadas; superior esquerdo. Os movimentos permitidos são
b) quantos triângulos, não contidos numa mesma face do representados pelas setas:
cubo, podem ser formados;
c) quantos tetraedros, com base numa das faces do cubo,
podem ser formados;
d) quantos tetraedros, com todos os vértices em faces
diferentes, podem ser formados.

De quantas maneiras isso é possível?


(IME-89/90) Imebol é um jogo de três jogadores. Em cada
partida o vencedor marca a pontos, o segundo colocado
(IME-96/97) Em cada uma das 6(seis) faces de um cubo,
marca b pontos e o terceiro marca c pontos, onde a>b>c são
construiu-se uma circunferência, onde foram marcados n
inteiros positivos. Certo dia Marcos, Flávio e Ralph
pontos. Considerando que 4 (quatro) pontos não
resolveram jogar Imebol, e após algumas partidas a soma
pertencentes à mesma face, não sejam coplanares, quantas
dos pontos era:

MATEMÁTICA – Prof. DOUGLAS OLIVEIRA biblioteca


telegram t.me/bibliotecaPE @PROFDOUGLASOLIVEIRA _2020 52
APROVADO DE A A Z
TURMA IME ITA
retas e triângulos, não contidos nas faces desse cubo, são (Brasil) Uma fábrica de brinquedos produz blocos cúbicos
determinados pelos pontos. de 2 cm de aresta, cujas faces são pintadas com uma das
cores: azul ou vermelho. Alguns blocos são totalmente
(IME-97/98) Uma embarcação deve ser tripulada por oito azuis, alguns são completamente vermelhos, e alguns têm
homens, dois dos quais só remam do lado direito e apenas uma mistura de faces azuis e faces vermelhas. Quantas
um, do lado esquerdo. Determine de quantos modos esta espécies distintas de cubos podem ser fabricadas?
tripulação pode ser formada, se cada lado deve haver
quatro homens. (Brasil) Dado um polígono regular de n lados, quantos
Observação: A ordem dos homens de cada lado distingue a triângulos possuem vértices que são vértices do polígono e
tripulação têm lados que não são lados do polígono?
n(n + 1)
Nível III a) b) n! c) (n – 1)! d)
(Brasil) Uma caixa contém 11 bolas, numeradas 1, 2, 3,...., 10,
2
11. Se 6 bolas são retiradas aleatoriamente e n(n + 1)(2n + 1) n ( n − 4)( n − 5)
e)
simultaneamente, qual é a probabilidade de que a soma dos 6 6
números das bolas retiradas seja ímpar. (NOTA: Para n = 7, existem sete desses triângulos)
100 115 1 118
a) b) c) d) e)
231 231 2 231 (Argentina-95) Com os dígitos 1, 2, 3, 4, 5, 6, 7, 8, 9
formam-se três números A, B, C, de três dígitos distintos
6
cada um, usando-se os nove dígitos. Pode-se dividí-los de
11 modo que nenhum seja múltiplo de 3?

(Brasil) Na contagem de n bolas coloridas, algumas (Noruega-95 2 Fase) De quantas formas é possível pintar
vermelhas, algumas azuis, achamos que 49 das primeiros 50 os seis lados de um cubo de preto e branco? (Não considere
contados eram vermelhas. Depois disso, encontramos 7 as combinações que podem ser obtidas por rotação do cubo)
vermelhos para cada 8 contados. Se, no total, 90% ou a) 7 b) 10 c) 20 d) 30 e) 36
mais dos bolas contadas foram vermelhas, o maior valor
possível para n é: (Noruega-96 2 Fase) Quantas contas de banco de 11
a) 225 b) 210 c) 200 d) 180 e) 175 dígitos existem usando apenas os dígitos 1 e 2, tais que não
ocorram dois 1’s consecutivos?
(Brasil) Um grupo de 21 amigos planejam jantar juntos a (Noruega-96 2 Fase) Em uma classe, a professora
cada semana. O restaurante escolhido não pode acomodar descobriu que todo aluno possui exatamente 3 amigos na
mais que 5 amigos numa mesma mesa e todos os amigos classe, que dois amigos nunca possuem um amigo em comum,
querem sentar juntos. Eles decidem que 5 grupos e que todo par de alunos que não são amigos possuem
diferentes poderiam se encontrar a cada semana, de modo exatamente um amigo em comum. Quantos alunos existem
que não se forme exatamente o mesmo grupo de cinco na sala?
amigos em duas semanas consecutivas. Quantas semanas a) 6 b) 9 c) 10 d) 12 e) 17
serão necessárias para que todas as possíveis combinações
de cinco amigos possam jantar juntos? (Noruega-97 2 Fase) Em um cubo 27 pontos são marcados
da seguinte maneira: um ponto em cada vértice, um ponto
(Brasil) De quantas maneiras distintas podemos pintar as em cada ponto médio de cada aresta, um ponto em cada
faces de um cubo com seis cores, de forma que a pintura centro de cada face e um ponto no meio do cubo. O número
apresente cores distintas em cada uma das faces? de linhas contendo 3 destes pontos é:
a) 33 b) 42 c) 49 d) 72 e) 81
(Brasil) De quantas maneiras distintas podemos numerar as
faces de um cubo usando os algarismos 1, 2, 3, 4, 5 e 6, se (Noruega-98 2 Fase) Nós temos um quadrado formado por
a soma dos números de cada par de faces opostas é 7? 4 fileiras
cada uma com 4 pontos. Quantos triângulos existem com
(Brasil) Considere uma mesa com três gavetas. A primeira vértices
gaveta contém duas moedas de ouro; a segunda uma moeda nos pontos? (Os três vértices não podem estar em uma
de ouro e uma moeda de prata; e a terceira duas moedas de mesma linha)
prata. Uma gaveta foi escolhida arbitrariamente e uma
moeda de ouro foi retirada. Qual é a probabilidade de que a) 252 b) 256 c) 360 d) 516 e) 560
a Segunda moeda retirada naquela gaveta seja também de
ouro? (Noruega-94 Fase Final) Em um grupo de 20 pessoas,
(ATENÇÃO: a resposta não é 1/2 !!) simultaneamente cada pessoa envia uma carta para 10

MATEMÁTICA – Prof. DOUGLAS OLIVEIRA biblioteca


telegram t.me/bibliotecaPE @PROFDOUGLASOLIVEIRA _2020 53
APROVADO DE A A Z
TURMA IME ITA
outras pessoas. Prove que existe pelo menos duas pessoas sorteia aleatoriamente um número de cartões e calcula para
que enviaram cartas uma para a outra. cada um a soma dos dígitos escritos. Quantos cartões Maria
deve sortear para que pelo menos três cartões tenham a
(Bélgica-90) Esta prova possui 30 questões. É dado que são mesma soma dos dígitos?
atribuídos quatro pontos para uma resposta correta, – 1 a) 53 b) 54 c) 55 d) 56 e) 57
para uma resposta incorreta e 0 para uma questão em
branco. Qual o número mínimo de participantes necessário (Bélgica-96) Cinco diferentes estradas levam a cidade A à
para que se possa afirmar com certeza de que dois cidade B, três diferentes estradas levam a cidade B à
participantes alcançaram a mesma pontuação na prova? cidade C e três diferentes estradas levam a cidade A à
a) 121 b) 145 c) 146 d) 151 e) 152 cidade C sem passar por B. De quantas formas diferentes é
possível ir de A para C e voltar para A passando por B pelo
(Bélgica-91) O alfabeto alemão consiste de 6 vogais e 20 menos uma vez.
consoantes. Assuma que uma palavra consistindo de 3 a) 22 b) 90 c) 270 d) 315 e) 324
caracteres possui pelo menos 1 vogal e pelo menos 1
consoante. Determine o máximo número de palavras de 3 (Bélgica-96) Determine o último dígito da seguinte soma:
caracteres que é possível formar no alfabeto alemão. S = 1! + 2! + 3! + … + 1995! + 1996!
a) 2880 b) 3120 c) 8640 d) 9360 e) 18000 a) 9 b) 7 c) 5 d) 3 e) 1

(Bélgica - Fase Final-93) Em dezembro, cada um dos 20 (Bélgica-97) Quantos triângulos não-congruentes podem ser
alunos de uma mesma classe enviaram 10 cartões para 10 construídos tais que os seus lados assumem valores inteiros
diferentes membros da classe (diferentes deles mesmos). menores ou iguais que 5?
a) Mostre que ao menos 2 estudantes enviaram cartões uma a) 10 b) 21 c) 22 d) 25 e) 35
ao outro.
b) Suponha uma classe consistindo de n estudantes, cada um (New Brunswick-98) ABCD é um quadrado. O número de
enviando m cartões para diferentes m membros da classe triângulos no diagrama é:
(diferentes deles mesmos). Como n e m devem se relacionar A B
de modo que existam pelo menos 2 alunos enviando cartões
um ao outro.

(Bélgica-94) Uma pequena classe possui 4 estudantes. Uma


professora aplica um teste e depois de recolher D C
imediatamente redistribui aos alunos para que possam
corrigir as provas. De quantas maneiras isto pode ser feito
sem que um mesmo aluno receba o seu próprio teste para a) 8 b) 12 c) 16 d) 20 e) nda
corrigir.
a) 6 b) 9 c) 14 d) 23 e) 24 (Canadá) Em uma multiplicação de inteiros de 1 a 15 existem
225 produtos. Quantos destes produtos resultam em
(Bélgica-94) Defini-se n! = 1.2.3…(n – 1).n. Seja k o menor números pares? 161
número natural diferente de 0 tal que k! é divisível por
1000. A soma dos dígitos de k é: (Canadá) Três dígitos são selecionados randomicamente da
a) 1 b) 2 c) 5 d) 6 e) 7 lista 1, 2, 3, 6, 9. Qual é a probabilidade de que os dígitos
formem um número que é divisível por 3? 2/5
(Bélgica-95) Um teste contém 3 questões com uma
pontuação máxima de 10 pontos. Para cada questão são (Canadá) Determine por quantos zeros termina 1000!
conferidos ao menos 2 pontos. De quantas formas
diferentes os 10 pontos podem ser divididos pelas três (Canadá-69) Encontre o valor da soma: 1.1! + 2.2! + 3.3! + … +
questões, com cada questão podendo assumir somente (n – 1).(n – 1)! + n.n!
valores inteiros de pontos?
a) 4 b) 13 c) 15 d) 20 e) 30 (Canadá-70) É dado um conjunto de bolas. Cada bola é
colorida de vermelha ou azul, e existe pelo menos uma de
(Bélgica-95) Quantos triângulos podem ser construídos tais cada cor. Cada bola pesa 100 ou 200 gramas, e existe pelo
que os comprimentos dos lados sejam três números ímpares menos uma de cada peso. Prove que existem 2 bolas
consecutivos, e o perímetro é estritamente menor que possuindo diferentes pesos e diferentes cores.
1000?
a) 165 b) 166 c) 167 d) 331 e) 332 (Canadá-83) Determine todas os inteiros positivos w, x, y e
z satisfazendo w! = x! + y! + z!.
(Bélgica-95) Uma urna contém 900 cartões, cada um
possuindo um número inteiro entre 100 e 999 escrito. Maria

MATEMÁTICA – Prof. DOUGLAS OLIVEIRA biblioteca


telegram t.me/bibliotecaPE @PROFDOUGLASOLIVEIRA _2020 54
APROVADO DE A A Z
TURMA IME ITA
(Canadá-91) Na figura, o comprimento do lado
do triângulo equilátero é 3 e f(3) (número de
paralelogramos construídos com os lados da figura)
é 15. Para a situação geral análoga, determine uma
fórmula para f(n) (número de paralelogramos, para
um triângulo de lado de comprimento n.

(Báltica-96) Usando os dígitos 1, 3, 4, 5, 6, 7, 8 e 9


exatamente uma vez, uma número A de três dígitos, dois
(Canadá94) Calcule o valor da soma: números B e C de dois dígitos, B < C, e um número de um
1994
n2 + n +1 dígito D são formados. Os números são tais que A + D = B +
 (−1) n
n =1 n!
. C = 143. De quantas maneiras isto pode ser feito?

(Torneio Internacional das Cidades-96) Demonstrar a


(South Alabama) Em uma urna existem 30 bolas vermelhas, desigualdade:
30 brancas, 30 azuis e 30 pretas. Determine de quantas 2 7 14 23 k2 − 2 9998
formas diferentes: + + + + ... + + ... + 3
2! 3! 4! 5! k! 100 !
a) Duas bolas da mesma cor podem ser retiradas
(i) com reposição NÍVEL COMPLEXO
(ii) sem reposição
b) Quatro bolas podem ser retiradas, duas da mesma cor e EM UM CONJUNTO A={1,2,3,...,2000} , QUANTOS
outras duas de cor diferente (sem reposição). SUBCONJUNTOS EXISTEM TAIS QUE A SOMA DOS
ELEMENTOS SEJA MÚLTIPLO DE 5?
(South Alabama) De quantas formas N pessoas podem
sentar em uma mesa circular se certas três pessoas devem
sentar-se juntas. Assuma N  3. DERIVADAS
(International Mathematical Talent Search) Encontre todos
REGRA DA CADEIA
os pares de inteiros n e k, 2 < k < n, tais que os coeficientes
 n  n  n  Se f e g são funções tais que f(x) = [g(x)]r, onde r é um
binomiais  ,  ,   formam uma
número racional qualquer, e se g’(x) existe, então:
 k − 1 k   k + 1
progressão aritmética.
f’(x)= r [g(x)]r-1 . g’(x)

(International Mathematical Talent Search) Uma empresa


Convém a memorização das seguintes derivadas:
multinacional possui 250 empregados, cada um falando
várias línguas. Para cada par de empregados (A, B), existe
Se f(x) = senx , f’(x) = cosx
uma língua falada por A e não falada por B, e existe uma
outra língua falada por B e não por A. Quantas línguas, pelo
Se f(x) = cosx, f’(x) = -senx
menos, são faladas pelos empregados da empresa?
Se f(x) = tgx , f’(x) = sec2x
(International Mathematical Talent Search) Em um
tabuleiro m x n uma pessoa deseja ir de um vértice (0, 0) ao
Se f(x) = cotgx , f’(x) = -cossec2x
outro vértice oposto (m, n). Um movimento legal é definido
como andar uma unidade positiva na direção y ou uma
Se f(x) = secx , f’(x) = secx tgx
unidade positiva na direção x. A casa (i, j), onde 0  i  m e
0  j  n, do tabuleiro é removida, ou seja, não é possível
Se f(x) = cossecx , f’(x) = -cossecx cotgx
passar por ela no caminho de (0, 0) até (m, n). De quantas
1
formas diferentes é possível ir de (0, 0) até (m, n). Se f(x) = arc senx , f’(x) =
1 − x2
(International Mathematical Talent Search) Em um grupo −1
Se f(x) = arc cosx , f’(x) =
de oito matemáticos, cada um considera que existe 1 − x2
exatamente três outros que possuem a mesma área de 1
Se f(x) = arc tgx , f’(x) =
interesse. É possível separar os oito matemáticos em pares, 1 − x2
de tal modo que cada par seja formado por dois
matemáticos que não possuem área de interesse em comum?

MATEMÁTICA – Prof. DOUGLAS OLIVEIRA biblioteca


telegram t.me/bibliotecaPE @PROFDOUGLASOLIVEIRA _2020 55
APROVADO DE A A Z
TURMA IME ITA
1 f'(x)
Se f(x) = arc secx , f’(x) = limf(x) = 0 e limg(x) = 0 e se lim = L segue que
x x −1
2 x →a x →a x →a g'(x)
−1 f(x)
Se f(x) = arc cossecx , f’(x) = lim = L.
x→a
x x −12 g(x)
−1
Se f(x) = arc cotgx , f’(x) = Sejam f e g funções diferenciáveis para todo x > N,
1 + x2
onde N é uma constante positiva e suponhamos que para
DERIVAÇÃO IMPLÍCITA todo x > N, g’(x)  0 .

Se y é uma função de x definida pela equação Então, se lim f(x) = 0 e lim g(x) = 0 e se:
x →+ x →+
y = 3x 2 − 5x + 6 então y é definida explicitamente em
termos de x e podemos escrever y = f(x) onde f(x) = 3x2 - f '(x) f(x)
lim = L segue que lim =L
5x + 6. x →+ g'(x) x →+ g(x)

Entretanto, nem todas as funções estão definidas


explicitamente. Por exemplo, a equação:
O teorema também é válido se tivermos x → -
x 2 3 + y 2 3 = 4xy (A) Outras formas indeterminadas onde o Teorema de
L’hôpital se aplica:
Não pode ser resolvida para y explicitamente como uma
função de x. Além disso, podem existir uma ou mais funções − − + +
; ; ; , quando x→a ou quando
f tais que se y = f(x), a equação (A) é satisfeita, isto é, a + − + −
equação é verdadeira para todos os valores de x no domínio x → -/+.
de f(x). Neste caso dizemos que y é definida implicitamente
como uma função de x ou que a função f(x) é definida Formas indeterminadas:
implicitamente pela equação dada.
Como poderemos obter y’ = f’(x), a partir da equação 0. 0 .(-)
(A)? 
 - -+  devemos alterá-las para uma
 forma indeterminada para que
0 0
(-)0  
 possamos aplicar a REGRA DE
O procedimento é fácil: 0 1-  
L’HÔPITAL.
1 

1º) Se as variáveis x e y estiverem em termos
independentes entre si, a derivada em relação a x, será
dada pelas fórmulas anteriores. Entretanto, nos termos de
Aplicações para traçar um Esboço do gráfico de uma
y, deve-se multiplicá-los por y’:
função:
2 −1 3
Dx (x 2 3 ) = x Iremos aplicar, agora, toda a teoria desenvolvida ao
3
longo dos tópicos anteriores sobre limite (Cálculo
2
Dx (y 2 3 ) = y − 1 3 . y' Infinitesimal) e derivada (Cálculo Diferencial) para o
3 derivada
traçado do ESBOÇO DO GRÁFICO DE UMA FUNÇÃO. É
primeira simples, tendo conhecimento da teoria anterior.
Primeiro determinamos as raízes da função suas
2º) Se houver termos de x e y dependentes (isto é,
restrições, se houver. Achamos suas assíntotas horizontais
produtos destes termos) devemos aplicar a derivada do
(limites no infinito), assíntotas verticais (limites infinitos) e
produto:
assíntotas oblíquas. Depois encontramos f’(x) e f’’(x). Logo,
os números críticos de f são os valores de x no domínio de
Dx (4xy) = 4.Dx (x).y + 4x Dx (y) = 4y + 4xy'
f, para os quais f’(x) não existe ou f’(x) = 0. Em seguida
1 y'
aplicamos o TESTE DA DERIVADA PRIMEIRA OU O
TESTE DA DERIVADA SEGUNDA para determinarmos se
TEOREMA DE L’HÔPITAL
temos um valor máximo relativo, um valor mínimo relativo ou
nenhum dos dois, num número crítico. Para determinarmos
Sejam f e g funções diferenciáveis (deriváveis) em um
os intervalos onde f é crescente, encontramos os valores de
intervalo aberto I, exceto possivelmente no número a em I.
x para os quais f’(x) é positiva; para determinamos os
Suponhamos que para todo x  a em I, g’(x)  0. Então, se
intervalos onde f é decrescente, encontramos os valores de
x para os quais f’(x) é negativa. Ao estabelecermos os
intervalos nos quais f é monótona, comprovamos também os

MATEMÁTICA – Prof. DOUGLAS OLIVEIRA biblioteca


telegram t.me/bibliotecaPE @PROFDOUGLASOLIVEIRA _2020 56
APROVADO DE A A Z
TURMA IME ITA
números críticos nos quais f não tem extremo relativo. Os dy dx
a) x 2 + y 2 = 4, , no ponto P(1, 3 ) , e , no ponto
valores de x para os quais f’’(x) = 0 ou f’’(x) não existe, nos dx dy
dão os possíveis pontos de inflexão, e verificamos se f’’(x)
Q( 3 ,1)
muda de sinal em cada um destes valores de x para
determinarmos se realmente temos um ponto de inflexão. dy
Os valores de x para os quais f’’(x) é positivo e aqueles b) y 4 + 3y − 4x 2 = 5x + 1, , no ponto P(0,-1)
dx
para os quais f’’(x) é negativo, serão os pontos nos quais o
gráfico é côncavo para cima e os pontos nos quais o gráfico c) y - x -
1
sen(y) = 0,
dy
, no ponto de ordenada
π
é côncavo para baixo. Também é conveniente encontrarmos 4 dx 2
a declividade de cada uma das tangentes de inflexão. Assim
d) e y + xy = e , y , no ponto de ordenada 1
sendo, podemos esboçar o gráfico.
e) xy 2 + y 3 = 2x − 2y + 1, y , no ponto em que a abscissa e a
EXERCÍCIOS
ordenada possuem o mesmo valor.

01. Para cada uma das funções seguintes, determinar a


04. Calcule as expressões das derivadas e os seus
expressão da derivada indicada, e o seu valor no ponto
respectivos valores nos pontos dados:
dado:

a) f(u) = u 2 ,u(x) = x 3 − 4, (f u)(x), no ponto x o = 1  x = sen t   dy 


a)  , t  [− , ], , no ponto t=
 y = sen 2 t 2 2 dx 6
dy  dy 
b) y = u.sen(u), u = x2 , e 
dx  dx  ( xo =  ) x = 6t(1 + t 2 )−1
 dy
b)  2 −1
,0  t  1, , no ponto de abscissa
 y = 6t (1 + t )
2
 dx
x+1
c) f(u) = 3 u 2 , u (x)= , (f u) '(x), no ponto xo = 1
x2 + 1 12
.
5
d) f(x) = 1 + x , f '(x) e no ponto xo = 4
d2y
  c) , função dada na letra a
e) f(x) = x.sen( + 3x) + cos 2 ( + x), f (0) e no ponto
5 5 dx 2
xo = 0
 x = e − t d 2 y
d)  ,
f) f(t) = 2 + 2 3t −3t
, f (t) e no ponto xo = 0  y = e
3t
dx 2

 1 + sen x   4 
g) f(x) = ln  , f '(x) e f   
 1 − sen x   3
05. Deve-se drenar uma piscina. Se Q é o número de litros
ex − e− x
h) f(x) = ,f (x), no ponto xo = 0 de água na piscina t minutos após o início da drenagem e
ex + e− x
Q(t) = 200 (30 – t)2, qual é a velocidade de escoamento da
( )
i) f(x) = ln  tg x 3 − x + e x  , no ponto x o = 0 água após 10 min?

06. Determine a área do retângulo máximo, com base no



02. Determine a expressão de ( f ) ( f ( x)) , lembrando-se
−1 eixo dos x e vértices superiores sobre a parábola y = 12 –
x2.

que f( )−1
( f ( x)) = f (x)
1
:
07. Uma escada de 10m de comprimento está apoiada numa
parede vertical. Se a extremidade inferior da escada
a) f(x) = x 2 + 4x − 2; começa a deslizar horizontalmente à razão de 0,5m /seg,
com que velocidade o topo da escada percorrerá a parede,
3x − 2
b) f(x) = ; quando a extremidade inferior estiver a 6m do solo?
x+2
c) f(x) = 3 + cos2x, 0  x   / 2; 08. Determine o volume do maior cilindro circular reto que
pode ser inscrito numa esfera de raio r.
d) f(x) = sen (ln x), e − / 2  x  e  / 2 ;

e) f(x) = x + e x 09. O raio da base de um cone cresce à razão de 1cm/min


e sua altura decresce à razão de 2cm/min. Como variará o
volume total do cone quando o raio é 4cm e sua altura 6
03. Calcule a expressão e o valor no ponto dado das
cm?
derivadas indicadas abaixo:

MATEMÁTICA – Prof. DOUGLAS OLIVEIRA biblioteca


telegram t.me/bibliotecaPE @PROFDOUGLASOLIVEIRA _2020 57
APROVADO DE A A Z
TURMA IME ITA
2x
a) f(x) = 10 + 12x − 3x 2 − 2x 3 b) f(x) =
10. Determine o maior comprimento que deve ter uma x +1 2

escada para passar de um corredor de 5 metros de largura x 2 − 4x + 3 x3


c) f(x) = d) f(x) = 2
a outro, perpendicular, de 8 metros de largura? x2 x −1
x
e) f(x) = f) f(x) = e− x
2

11. Uma reta passando por (1,2) corta io eixo dos x em (x + 1)2
A = (a, 0) e o eixo dos y em B = (0, b). Determine o triângulo ex
AOB de área mínima para a e b positivos. g) f(x) = h) f(x) = ln (x 2 + 1)
x
x
i) f(x) = j) f(x) = x2 + 2x + 5
12. Determine os catetos de um triângulo retângulo de 2ln x
área máxima sabendo que sua hipotenusa é h. 2
k) f(x) = 3 (x 2 − 1)2 l) f(x) = 1 + (x − 2) 3

13. Se uma droga é injetada na corrente sanguínea, sua m) f(x) = x 9 − x2 n) f(x) = 3 x2 (4 + x)


concentração t minutos depois é dada por
C(t) = k (e–2t – e–3t), onde k é uma constante positiva. ln x + 1
17. (EN) O gráfico da função f (x) = é:
a) Em que instante ocorre a concentração máxima? ln x − 1
b) Que se pode dizer sobre a concentração após um longo
período de tempo?

14. Para fazer um cilindro circular reto de um retângulo de


folha de aço colam-se duas bordas paralelas da folha. Para
dar rigidez ao cilindro cola-se um arame de comprimento l
ao longo da diagonal do retângulo. Ache a tangente do
ângulo formado pela diagonal e o lado não colocado, de tal
modo que o cilindro tenha volume máximo.

15. Calcular os seguintes limites aplicando L’Hospital:

2 x − 3x
a) lim b)
x→0 x
sen(  x)
lim
x→ 2 2−x

c) lim ( sec x − tgx )


x→  / 2
d) lim 3X 1 − e1/ x 
x →+  ( )
 1 1  1 / ln x )
e) lim  − f) lim+ x (
x →1  ln x x − 1  x→ 0

x
g) lim (1 − x) .tg( ) h) lim+ ( x.ln x)
x →1 2 x→0

x + sen x
( tgx)
sen 2x
i) lim j) lim − 18. (EN) Dois trens se deslocam sobre trilhos paralelos,
x →+ 3x − cos x x → / 2
separados por 1/4 km. A velocidade do primeiro é de 40
km/h e a do segundo 60 km/h, no mesmo sentido que o
x
 1
l) 2x(sen(x 2 ) + x2 cos(x2 )) m) lim  1 + 
x →+  x primeiro. O passageiro A do trem mais lento observa o
passageiro B do trem mais rápido. A velocidade com que
sen(1/ x) ln (1 − 2x) muda a distância entre eles quando A está a 1/8 km à
n) lim o) lim −
x →+ arctg(1/ x) x →1 / 2 tg( x) frente de B é, em km/h:
20
e x + sen x − 1 3
1 + 2x + 1 a) b) 5 c) 0
p) lim q) lim 5
x→0 ln (1 + x) x →−1 x+ 2+x −20
d) − 5 e)
5
( )
2/x
r) lim e x + x
x →+

20. (EN) Considere um cone circular reto de raio da base 5


cm e altura 12 cm. As dimensões do raio e da altura do
16. Esboce os gráficos das funções abaixo:
cilindro circular reto, de maior volume, que pode ser
inscrito neste cone, são respectivamente:

MATEMÁTICA – Prof. DOUGLAS OLIVEIRA biblioteca


telegram t.me/bibliotecaPE @PROFDOUGLASOLIVEIRA _2020 58
APROVADO DE A A Z
TURMA IME ITA
a) 10/3 e 4 b) 4 e 10 4 −2 3
c) 3 e 14/3 d) 9/5 e 23/4 f ( )=
3 3
e) 5/2 e 5
h)
x2 + 1
20. Podemos observar que o gráfico da função y = 2 : 4
x −1 f (x) =
(e x + e − x )2
a) cresce em ]-  , -1[  ]0, 1[.
f (0) = 1
b) tem (0, -1) como ponto de inflexão.
c) tem assíntota horizontal em y = 1 e assíntota vertical em
x = 1 e x = -1. i)
d) tem concavidade voltada para cima para qualquer f (x) = 2.(3x2 − 1 + ex ).cossec[2.(x3 − x + ex )]
x  ]-1, 1[ f (0) = 0
e) está definido para todo x .

02.
1
a) (f −1 )(f(x)) =
01. 2x + 4
a) (x + 2)2
b) (f )(f(x)) =
−1

( f u ) ( x) = 6x2 (x3 − 4)
8
c) (f −1 )(f(x)) = −2.cossec(2x)

( f u) (1) = −18 x
d) (f −1 )(f(x)) =
cos(ln x)
b) 1
dy e) (f −1 ) (f(x)) =
= 2x(sen(x ) + x cos(x )) 2 2 2 1 + ex
dx
 dy  03.
 dx  = −2  
(x o=  ) a)
x 3
y = − yp = −
c) y 3
2 3 x2 + 1 1 − 2x − x2 y
(f u)(x) = . x = − xq = − 3
3 x + 1 (x2 + 1)2 x
dy 8x + 5
1 b) = yp = −1
(f u)(1) = − dx 4y 3 + 3
3
4
c) y = yp = 1
d) 4 − cos y
1 y 1
f (x) = d) y  = − y p = −
4 x+x x ey + x e
2 − y2 31
3 e) y  = y p =
f (4) = 2xy + 3y 2 + 2 37
24

  2 04.
e) f (x) = sen( + 3x) + 3x.cos( + 3x) − sen( + 2x)
5 5 5 a)
 2 dy 2.cos(2t)
f (0) = sen( ) − sen( ) =
5 5 dx cos(t)
 dy 
f)  dx   = 2 3
(t = )
f (t) = 3ln(2).(23t − 2−3t ) 6

f (0) = 0
dy 2t 12 1
b) = ; para x = , temos t =
g) dx 1 − t 2 5 2
f (x) = sec x d 2 y 2 cos(2t).sen(t) − 4.sen(2t).cos(t)
c) =
dx 2 cos 3 (t)

MATEMÁTICA – Prof. DOUGLAS OLIVEIRA biblioteca


telegram t.me/bibliotecaPE @PROFDOUGLASOLIVEIRA _2020 59
APROVADO DE A A Z
TURMA IME ITA
d2 y
d) = 12.e 5t
dx2 b)

05. 8000 l/m

06. 32 u.a.
2
07. m / seg.
3
4 3 2
08. r
9

09. Aprox. 16,75 cm/seg.

10. 18.22m

11. 4u.a.

h
12. Comprimento de cada cateto
2
c)
13.
a) ln (3) – ln (2)
b) a droga é completamente eliminada.

14.

2
tg() =
2

15.
2
a) ln( ) b) − c) 0
3
1
d) −3 e) f) e d)
2
2 1
g) h) 1 i)
 3
j) 1 l) e 2 m) e
n) 1 o) 0 p) 2
1
q) r) e 2
2

16.

a)

e)

MATEMÁTICA – Prof. DOUGLAS OLIVEIRA biblioteca


telegram t.me/bibliotecaPE @PROFDOUGLASOLIVEIRA _2020 60
APROVADO DE A A Z
TURMA IME ITA

j)
f)

g)

k)

h)

l)

i)

MATEMÁTICA – Prof. DOUGLAS OLIVEIRA biblioteca


telegram t.me/bibliotecaPE @PROFDOUGLASOLIVEIRA _2020 61
APROVADO DE A A Z
TURMA IME ITA
Seja f derivável até a ordem n no intervalo I e seja x0 ÆI.
f n (x 0 )
O polinômio P (x) = f (x0) + f2 (x0) (x – x0) + (x – x0)2
2!
f (n) (x0 )
+ ... + (x − x0 )n .
n!
Denomina-se polinômio de Taylor, de ordem n, de f em volta
de x0.
O polinômio de Taylor, de ordem n, de f em volta de x0 é o
único polinômio de grau no máximo n que aproxima localmente
f em volta de x0 de modo que o erro E (x) tende a zero mais
rapidamente que (x – x0)n, quando x  x0.
m) O polinômio de Taylor, de ordem n, de f em volta de x0 = 0
denomina-se também polinômio de Mac-Laurin, de ordem n, de f.

PRIMITIVAS IMEDIATAS

Sejam  ≠ 0 e c constantes reais. Das fórmulas de


derivação já vistas seguem as seguintes de primitivação:
a)  c d x = c x + k

x+1
b)  xa d x = + k (  −1)
+1
c)  ex d x = ex + k
1
d)  d x = ln x + k (x  0)
n) x
1
e)  d x = ln ( − x) + K (x  0)
x
1
f)  d x = ln x + k
x
g)  cos x d x = sen x + k
h)  sen x d x = − cos x + k
i)  sec2 x d x = tg x + k
l)  sec x d x = ln sec x + tg x + k

m)  tg x dx = − ln cos x + k
1
n)  d x = arc tg x + k
1 + x2
17. C 18. A 19. A 20. C 1
o)  d x = arc sen x + k
1 − x2

QUESTÕES

01. Calcule o polinômio de Taylor de ordem 1 da função


dada, em volta de x0 dado:
a) f (x) = x , x0 = 1 b) f (x) = sen x, x0 = 0
INTEGRAIS E VOLUMES c) f (x) = 3
x , x0 = 8 d) f (x) = ex, x0 = 0
1
e) f (x) = cos 3x, x0 = 0 f) f (x) = , x0 = 0
1+ x
RESUMO TEÓRICO
02. Determine o polinômio de Taylor, de ordem 2, de f em
volta de x0 dado.
POLINÔMIO DE TAYLOR DE ORDEN n
a) f (x) = ln (1 + x) e x0 = 0 b) f (x) = ex e x0 = 0

MATEMÁTICA – Prof. DOUGLAS OLIVEIRA biblioteca


telegram t.me/bibliotecaPE @PROFDOUGLASOLIVEIRA _2020 62
APROVADO DE A A Z
TURMA IME ITA
1 dy
c) f (x) = 3 x e x0 = 1 d) f (x) = e x0 = c)  x + 5 dx d) 
1 − x2 b − ay
0
4x 2
e)  y(b − ay2 ) dy f)  dx
03. Determine o polinômio de Taylor de ordem 5 em volta x3 + 8
de x0 dado. 6x dy
g)  dx h) 
2 2
a) f (x) = sen x e x0 = 0 (5 − 3x ) (b + ay)3
b) f (x) = cos x e x0 = 0 i) ∫sec3 (x) dx j) ∫(x – 1) e-x dx
c) f (x) = ln x e x0 = 1 1
ex x3
04. Calcule e verifique sua resposta por derivação: k)  dx l)  dx
x3 1 − x2
a)  sen x dx b)  sen 2x dx
m) ∫ xcosc2 (x) dx n) ∫ x sec (x) tg (x) dx
c)  cos 5x dx d)  sen 4t dt
e)  cos 7t dt f)  cos 3 t dt o) ∫x3 sem (5x) dx p) ∫ x4 cos (2x) dx

1 1   1  q) ∫ x4 ex dx r) ∫ (x5 – x3 + x) e-x dx
g)   − cos 2x dx h)   2 + sen 2x dx
2 2   3 
 1  1  07. Calcule as seguintes integrais que envolvem potências
i)   x + cos 3x dx j)   + 4 sen 3x dx
 5  x  de funções trigonométricas:

1 5    sen 2 (x)
l)   + cos 7x dx
1
m)   cos 3x + sen 4 a)  b) ∫ tg5 (x) sec3 (x) dx
3 2   2  cos 4 (x)

1 1  sen 2x sen 5 (x)


n)   sen 2x + cos 3x dx o)  dx c) ∫ sen2 (x) cos2 (x) dx d)  dx
3 2  cos x cos (x)
1 1  1  sen (x)
p)   cos 3x − sen 7x dx q)   e 3x + sen 3x e)  dx f) ∫ (cotg2 (2x) + cotg4 (2x) dx
3 7  3  tg 2 (x)
x2 + x + 1 3 2 cos 4 (x)
r)  dx s)   + g)  h) ∫ sen4 (ax) dx
 x x 2 
2 dx
x sen6 (x)
t)  e 2x dx sen 4 (x)
i) ∫ sen3 (y) cos4 (y) dy j)  dx
cos6 (x)
05. Calcule as seguintes integrais usando o método de
substituição:
1 x
x 3x 08. (EN)  dx é igual a:
a)  dx b)  2 dx 2
+ x4
x +1 0 2 − 2x
5
x2 − 1
a) −/8 b) −/4 c) /8 d) /4 e) 0
dy
c)  x + 5 dx d)  b − ay
e 3y
4x 2 09. A solução de  dy é:
e)  y(b − ay f) 
3 3y
+ 3
2
)dy dx e
x +8 3
1 3y 1
6x dy a) (e + 3)3/2 + c b) 3
(e 3y + 3)2 + c
g)  (5 − 3x2 )2 dx h) 
(b + ay)3
2 3
1 3 3y 1 3y
c) (e + 3)2 + c d) (e + 3)3/2 + c
in(x) + 2 3
x j) 
i) 3
a + bx dx 4
dx 2
x 1 3y
e) (e + 3)-2/3 + c
 x  x
k)  se n(2x)cos (2x) dx
2
l)  tg   sec 2   dx 2
 2  2
cos (ax) 1  x − 1
m)  b + sen(ax)
dx n)  x(In(x)) 2
dx sen 
 3 
10. Calculando  dx , encontramos:
 x − 1
cos2 
06. Calcule as seguintes integrais usando o método de  3 
substituição: x−1 x−1
a) sen +C b) 3 sec +C
x 3x 3 3
a)  dx b)  dx
5
x2 − 1 x2 + 1 x−1 x−1
c) cos +C d) sec +C
3 3

MATEMÁTICA – Prof. DOUGLAS OLIVEIRA biblioteca


telegram t.me/bibliotecaPE @PROFDOUGLASOLIVEIRA _2020 63
APROVADO DE A A Z
TURMA IME ITA
x−1 3
In2 (x)
e) 3csc +C i)
1
j)
3 (a + bx4 ) 2 + c; b  0. + 2 In (x) + c
6b 2
1  x
01. k) − cos 3 (2x) + c l) sec 2   + c
6  2
1 1
a) 1 + (x – 1) b) x c) 2 + (x – 8) 2 1
2 12 m) b + sen(ax) + c; a  0 n) − +c
d) 1 + x e) 1 f) 1 – x a In(x)

02. 06. [4]


1 2 1 2 a) ex (x – 1) + c b) 2x sen(x) – (x2 – 2) cos(x) +
a) x – x b) 1 + x + x
2 2 c
1 1 ex e −1(  sen( t) − cos( t)
c) 1 + (x – 1) – (x – 1)2 d) 1 + x2 c) +c d) +c
3 9 1+ x 2 + 1
1
03. e) x (sen(In(x))) + c f) − 1 1 − 4x2 + xarccos(2x) + c
2 2
3 5 2 4
x x x x
a) x – + b) 1 – + x
3 (sen(x) + cos(x)In(3) (x2 + 1)arctg(x) x
3! 5! 2! 4! g) + c h) − +c
2
1 + (In(3)) 2 2
1 2 1 1
c) (x – 1) – 5 (x – 1) + (x – 1)4 + (x – 1)5 1
2 3 5 (sec(x)tg(x) + In(|sec(x) + tg(x)|))
i) j) – xe-x +
2
c
1

04. e2
k) (x − 1) + c
1 x
a) – cos x + k b) – cos 2x + k 1
2 l) − 1 − x 2 (x 2 + 2) + c sugestão: faça u = x2
1 1 3
c) sen 5x + k d) – cos 4t + k
5 4 m) – x cotg (x) + In (|sen (x)|) + c
1 1 n) x sec (x) – In (|sec (x) + tg (x)|) + c
e) sen 7t + k f) sen 3 t + k  6x x 3   3x 2 6 
7 3 o) cos 5x ( ) −  + sen(5x)  −  +c
1 1 1  125 5   25 625 
g) x– sen 2x + k h) 2x – cos 2x + k
2 4 6  x 4 3x 2 3   3x 
x 2
1 4
p) sen 5x ( ) − +  + cos (2x)  x 3 −
 2 
+c
i) + sen 3x + k j) In |x|– cos 3x + k  2 2 4 
2 15 3
q) ex (x4 – 4x3 + 12x2 – 24x + 24) + c
1 5 1 1 r) – e-x (x5 + 5x4 + 19x3 + 57x2 + 115x + 115) + c
l) x + sen 7x + k m) sen 3x − cos 4x + k
3 14 3 8
1 1 07.
n) − cos 2x + sen 3x + k o) – 2 cos x + k
6 6 1 3
a) tg (x) + c
1 1 1 3x 1 3
p) sen 3x + cos 7x + k q) e − cos 3x + k
9 49 9 3 sec 2 (x)
4 b) (35 cos(4x) − 28 cos(2x) + 57) + c
x 1 2 840
r) + In|x| − + k t) e 2x
+k
4 x 2 x sen (4x)
c) − +c
8 32
05.
 2 1 
4 d) −2 cos(x)  1 − cos 2 (x) + cos 4 (x) + c
a)
5 2
8
x – 1( ) 5 +c b)
3
2
In (x 2 + 1) + c
 5 9 
e) cos (x) + In (|cosec (x) – cotg(x)|) + c
3 1
2 2 f) − cot g (2x) + c
3
c) (x + 5) 2 + c d) − b − ay + c; a  0. 6
3 a
1 8 1
e) (2by 2 − ay 4 ) + c f) x3 + 8 + c g) − cot g 5 (x) + c
4 3 5
1 1 h)
g) +c h) − + c; a  0. 1
5 − 3x2 2a (b + ay)2 (3ax − 3cos (ax) sen (ax) − 2 sen 3 (ax)cos(ax) + c;a  0.
8a

MATEMÁTICA – Prof. DOUGLAS OLIVEIRA biblioteca


telegram t.me/bibliotecaPE @PROFDOUGLASOLIVEIRA _2020 64
APROVADO DE A A Z
TURMA IME ITA
cos5 (y)  2
i) − 2
 sen (y) + 5  + c
7
tg 5 (x)
j)
5

08. C 09. C 10. B QUESTÕES:

01. Calcule as seguintes integrais:


RESUMO TEÓRICO: x
sec 2 (x)
a)  3−1 2x + 3 dx b)  a
x
tg 3 (x)
dx
De um modo geral, se f é uma função contínua definida
4
x
2 e 2x + cos(x) x
sec 2 (2x)
 c)  d) 
n

 f xi ix se
2
em [a, b], o número do qual as somas
dx 8
dx
0 e 2x + sen(r) 0
1 + tg(2x)
i =1

aproximam arbitrariamente à medida em que todos os ix x


1 e2x
pequeno, i = 1, 2, ... n, temos à igualdade aproximada.
e)  0
4
sen(x) cos(x) dx f)  0 e +1
2x
dx
x x
sec 2 (x)
g)  4
sen(x) ln(cos(x)) dx h)  4
dx
( )
n

 f(x) dx   f xi  i x e tg(x)
b 0 0
a
x
i =1 1
i) 
0
xe − x dx j) 
0
2
e 2x sen(3x) dx

No caso da área A que estávamos calculando, podemos  1

escrever:
l) 
0
3x cos(x) dx m) 
0
x 4 e − x dx
x
1
n)  x
2
cos(x) ln(sen(x)) dx o)  x 5x dx
A =  f(x) dx
b 0
4
a
a x
x
p)  0
x5 cos(x3 ) dx q)  3
0
tg(x) sec 3 (x) dx
Considerando o sólido de revolução
 3 x
r)  0
cos(3x) cos(4x) dx s)  0
(x + 4)5
2
dx

1 b
02. O número  =
b−a a
f(x) dx é chamado valor médio da

função f no intervalo [a, b]. Calcule o valor médio das


funções nos intervalos indicados:
a) f(x) = sen2(x); [0, ] b) f(x) = 2 + 5 cos(x); [-,
]
x
c) f(x) = ln(x); [1, 2] d) f(x) = ; [0,1]
1 + x2
gerado a partir da rotação do gráfico de f em torno do eixo cos(x) 
e) f(x) = ; [0, ] f) f(x) = x2 ex; [0,1]
dos x, sendo f(x)  0 em [a, b] (ver figura). sen(x) 2
Não sendo f constante, vamos dividir [a, b] em pequenos
sub-intervalos e em cada um deles, aproximando f(x) por
03. Calcule a área sob o gráfico de y = f(x) entre x = a e x =
uma função constante, vamos calcular o volume da fatia do
b, esboçando cada região, se:
sólido gerado como se fosse o de uma fatia cilíndrica:
a) ∫ (x) = 1 – x2, x = – 1, x = 1
Assim, o volume V será, aproximadamente, a soma dos
b) ∫(x) = x3 – x, x = – 1, x = 1
volumes das fatias cilíndricas consideradas, ou seja:
c) ∫(x) = x3 – 4 x2 + 3 x, x = 0, x = 2

n 2
V    . f xi  .  i x x − x3
i =1
  d) (x) = , x = −1, x = 1
3
onde xi   xi −1 . xi  e ix = xi – xi-1 e) ∫(x) = ln(x), x = 1, x = e
f) ∫(x) = cos2(x), x = 0, x = 1
Lembrando da definição de integral, resulta: g) (x) = 2 x − 1, x = 1, x = 10.
V =   . [fx)] dx =   [f(x)] dx
b
a
2 b
a
2
h) ∫(x) = x(x – 5)2, x = 0, x = 1

()
i)  x =
5
x+2
x = 0, x = 3

MATEMÁTICA – Prof. DOUGLAS OLIVEIRA biblioteca


telegram t.me/bibliotecaPE @PROFDOUGLASOLIVEIRA _2020 65
APROVADO DE A A Z
TURMA IME ITA
j) ∫(x) = x 4x 2 + 1, x = 0, x = 2
l) ∫(x) = |x|3 x = – 2, x = 6
m) ∫(x) = (x + 1)3 + 1, r = – 2, x = 0

07. Determine o volume do sólido de revolução gerado pela


04. Calcule a área das regiões limitadas pelas seguintes
rotação em torno e reta indicada da região limitada pelas
acurvas:
seguintes curvas:
5
a) y = x2, y = 2 x + a) 2x + y = 2 e o eixo dos y; a reta x = 1
4 b) y = x2; a reta x = 1
b) y = – x2 – 4, y = – 8
c) y = ex, 1 ≤ x ≤ 2; a reta y = 1
c) y = 5 – y2, y = x + 3
d) y = x4, y = 1; a reta y = 2
d) x = y2, y = x + 3, y = – 2, y = 3
e) y = x , y = 1 a reta y =1
e) y3 = x, y = x
f) y = 4 – x2, no primeiro quadrante; a reta x = 2
f) y = – x2 – 1, y = – 2x – 4
g) y = 2x – x2, a reta y = 0
g) x = y2 + 1, y + x = 7
h) y = 4 – x2, y = x2 – 14 h) y = 4 – x2, y = 2, a reta y = 2

i) y = x3, y = 3 x i) y = x , y = 0 e x = 9, a reta x = 9
j) y = x2, y = x4
l) x = y2 – 2, x = 6 – y2 08. Calcule os comprimentos de arco das seguintes curvas,
m) y = x |x|, y = x3 entre os pontos indicados:
a) y = 5x – 2, (– 2, – 12) e (2, 8)
05. Determine o volume do sólido de revolução gerado pela b) 2xy = 4x4 + 3, (1,
7
) e (3,
109
)
rotação em torno do eixo dos x da região limitada pelas 12 12
seguintes curvas: y3 1 7 67
c) x − − = 0, ( , 1) e ( , 3)
a) y = x + 1, x = 0, x = 2, y = 0 3 4y 12 24
b) y = x2 + 1, x = 0, y = 0, x = 2
d) y = ln(x), (x, y) tal que 3 x 8
c) y = x2, y = x3
 1 3 3 2 14
d) y = cos(x), y = sen(x), x = 0, x = e) y = (x + ), (1, ) e (3, )
4 6 x 3 3
e) x + y = 8, x = 0, y = 0 f) x3 + y3 = 23
f) y = x4, y = 1, x = 0
3
1 2
g) y = (x + 2) 2 , (x, y) tal que 0  x  1
g) xy = 1, x = 2, y = 3 3
h) x2 = y3 e x3 = y2
i) y = cos(2x), 0 ≤ x ≤  09. Uma bola de ferro é atraída por um imã com uma força
j) y = xex, y = 0 e x = 1 de 12x-2 N quando a bola está a x metros do imã. Qual o
trabalho realizado para empurrá-la sentido contrário ao do
06. Determine o volume do sólido de revolução gerado imã, do ponto onde x = 2 ao ponto onde x = 6?
pela rotação, em torno a reta indicada da região limitada
pelas seguintes curvas: 10. Uma partícula está localizada a uma distância de x
5x metros da origem. Uma força de (x2 + 2x) N é aplicada
a) x = 1 + sen(y), x = 0, y = 
2 sobre a partícula. Qual é o trabalho realizado para mover a
b) y2 = 4x, y = 0 e x = 4 partícula de x = 1 até x = 3?
1 15
c) y = 1 – 4 , x = 1, y = 0 e y =
x 16 11. Sobre uma partícula que se desloca sobre o eixo
2
d) 9x + 16y = 144 2 dos x atua uma força cujo componente na direção do
e) y = x2 + 1, x = 0 e x = 2 2
deslocamento é f(x) = 2 . Calcule o trabalho realizado pela
f) y2 = x, x = 2 y x
g) y = x 2 + 1, x = 0 e x = 2 força quando a partícula se desloca de x = 1 até x = 2.

h) y = 4
4 − x2 , x = 0 e x = 1 01.
26 1 1
a) b) c) ln (e + 1) d) 2 −1 e)
3 3 4
1  e2 + 1 2 + ln(2)
f) ln g) −1 h) 1 – e-1
2  2  2 2

MATEMÁTICA – Prof. DOUGLAS OLIVEIRA biblioteca


telegram t.me/bibliotecaPE @PROFDOUGLASOLIVEIRA _2020 66
APROVADO DE A A Z
TURMA IME ITA
1  3x + 1  26 206 2 
i) 1 – 2e-1 j) (3 − 2 e  ) l) – ln (3)  a)  b)  c)  d)
 3 15 35
 1 + ln (3)  2
2
13
m) 24 – 65 e-1
512  10  55 
e) f) g)
3 9 6
02. 5 2 (e 3 − 1)
h) i) j)
2 + ln(2) 5ln(5) − 4 2 7 28 2 4
a) −1 b) c) − d)
2 2 ln 2 (5) 3 3
06.
1 5 1
e) 0 f) − g) 15 2 256 3
24 75 2 a) b)  c) d) 64
h) 2 2 5 2
4 
ln(2)
i) 2 ln(2) – 1 j) l) m) e – 64  2 5
2 8 e) 8  f) g)
2 15 3
16 5 
h)
03. 5
4 1 3
a) b) c)
3 2 2 07.
2   4
a) b) c) (e − 5e 2 + 4e + 2)
3 6 2
221  8
d) e) f)
45 6 3

1 e) 1 f)  16 64 2  648 
d) g)  h) i)
6 15 15 5

08.
53 53
a) 4 26 b) c)
6 6
1  3 14 4
g) 36 d) 1 + ln e) f) 12 g)
h)
113 i) 10( 7 − 2 ) 2  2  3 3
12
09. 4J

50
10. J
3
1 l) 20 m) 2
j) (17 17 − 1) 11. 1J
2

DESAFIO IME (ANTIGO)

1) Seja k real fixado e (k + 1)2y2 + x2 + 2(k – 1)xy + mk2y =


0a
equação cartesiana de uma família F de cônicas de
parâmetro k. Determine a equação cartesiana do lugar
04.
geométrico dos centros das cônicas da família
9 32 9 145 1 32
a) b) c) d) e) f) F.
2 3 2 6 2 3
125 4 64 1
g) h) 72 i) 1 j) l) m) (Solução) Tome f(x,y) igual à expressão em x e y.
6 15 3 6

05. Derive parcialmente em relação à x e em relação à y e


iguale a zero ambas

MATEMÁTICA – Prof. DOUGLAS OLIVEIRA biblioteca


telegram t.me/bibliotecaPE @PROFDOUGLASOLIVEIRA _2020 67
APROVADO DE A A Z
TURMA IME ITA
as expressões.
7) Se a, b,c,d, e são inteiros tais que

Encontre x e y em função de k e tente eliminar k. A equação (x 2


)( 3 2
)
+ ax+ b x + cx + dx+ e = x − 9 x − 97 , o valor 5

de a + b + c + d + e é igual a:
em x e y será o
(A) 0 (B) 3 (C) 6
LG dos cenros das cônicas, ok ? ( ficará uma igualde com x,
(D) 9 (E) 12
y e m).
8) O inteiro a para o qual x − x + a divide x + x + 90
2 13

é igual a:
Obs: há determinadas situações em que o centro não (A) −2 (B) −1 (C) 1
existirá. (D) 2 (E) 3

9) Se a e b são inteiros tais que x2 − x − 1 é um fator


de ax + bx + 1 o valor de a é igual a:
17 16

POLINÔMIOS (A) 981 (B) 983 (C) 985


(D) 987 (E) 989

1) Os valores de A e B tais que


27x − 30 A B
= + satisfazem a :
x2 − 5 x + 6 x − 2 x − 3
(A) A + B = 75 (B) A + B = 27 (C) A − B = 27
10) O resto da divisão do polinômio
(D) A − B = 75 (E) 2A + B = −3
x9999 + x8888 + x7777 + x6666 + x5555 + x4444 + x3333 + x2222 + x1111+ 1
Sabendo que P( x) é um polinômio de grau maior que 3 por x + x + x + x + x + x + x + x + x + 1 é igual a:
9 8 7 6 5 4 3 2
2)
(A) 0 (B) 1 (C) 10
tal que P(1) = 2, P(2) = 3 e P(3 ) = 5 , seja R( x) o
(D) 100 (E) 1000
resto da divisão de P( x) por (x − 1)(x − 2)(x − 3 ) . O
valor de R(98 ) é igual a: 11) Seja P( x) um polinômio de coeficientes inteiros tal que
(A) 4751 (B) 4753 (C) 4755 P(21) = 17; P(32 ) = −247 e P(37 ) = 33 .
(D) 4757 (E) 4757
Sabendo que existe um inteiro N tal que
P(N) = N+ 51, o valor de N é igual a:
3) Seja P(x) = x5 + x4 + x3 + x2 + x + 1. O resto da
(A) 20 (B) 22 (C) 24
divisão do polinômio P x ( 12) por P(x) é igual a: (D) 26 (E) 28
(B) 3 − x + x − x (C) 5 − x
2 3
(A) 6
12) Determine a condição necessária e suficiente para que
(D) 2 − x + x − x + x (E) 4 − x + x
2 3 4 2
a1x2 + b1x + c1
expressão: em que a1, b1, c1, a2, b2, c2
a2x2 + b2x + c2
4) O polinômio x2n + 1+ (x + 1)2n não é divisível por
são reais não nulos, assuma um valor que não dependa
x2 + x + 1 se n é igual a: de x.
(A) 17 (B) 20 (C) 21
(D) 64 (E) 65 13) Obter   R de modo que os polinômios p(x) = x4 + 2x3
– 4x + 4 e g(x) = x2 + 2x + 2 verifiquem a condição
5) Se P( x) é um polinômio do 5o grau tal que p(x) = g(x)2.
1= P( 1) = P( 2 ) = P( 3 ) = P( 4 ) = P( 5 ) e P(6 ) = 0 . O valor
de P(0 ) é igual a: 14) Dado p(x) = x6 + ax5 + bx4 + cx3 + dx2 + ex + f um
polinômio tal que:
(A) 0 (B) 1 (C) 2
p(1) = 1, p(2) = 2, p(3) = 3, p(4) = 4, p(5) = 5 e p(6) = 6.
(D) 3 (E) 4
Quanto vale p(7)?

6) Um polinômio do 6o grau é tal que o coeficiente do seu


15) Dado p(x) = x8 + a1x7 + a2x6 + ... + a7x + a8 , n  N,
termo do sexto grau é igual a 1. Sabendo que P(1) = 1,
um polinômio tal que:
P(2 ) = 2 , P(3 ) = 3 , P( 4 ) = 4 , P(5 ) = 5 e P(6 ) = 6 . p(1) = 2, p(2) = 6, p(3) = 12, p(4) = 20, p(5) = 30, p(6) = 42,
O valor de P(7 ) é igual a: p(7) = 56, p(8) = 72. Calcule p(9).
(A) 0 (B) 7 (C) 14
(D) 49 (E) 727

MATEMÁTICA – Prof. DOUGLAS OLIVEIRA biblioteca


telegram t.me/bibliotecaPE @PROFDOUGLASOLIVEIRA _2020 68
APROVADO DE A A Z
TURMA IME ITA
16) (AFA-95) O resto da divisão de x3 + px + q por x2 – x
– 2 é 2x – 1. Então, o valor de p2 + q2 é: 56) (ITA-67) Um polinômio P(x) tem a propriedade P(x) =
a) 8 b) 9 c) 10 d) 11 P(– x – 1). Definindo um novo polinômio Q(x) = P(f(x)),
obteremos Q(x) = Q(– x) quando f(x) for igual a:
17) (AFA-95) Um polinômio P(x) do terceiro grau que, para a) x – 1/2 b) x + 1/2 c) – x – 1 d) x – 1 e) –
todo número real, satisfaz a expressão x+1
P( x) = P( x − 1) + x2 é:
a) x3/3 + x2/2 – x/6 b) x3/3 – x2/2 + x/6 57) (ITA-68) Dizemos que os polinômios p1(x), p2(x) e p3(x)
c) x3/3 + x2/2 + x/6 d) x3/3 – x2/2 – x/6 são linearmente independentes (L.I.) se a relação a1p1(x) +
a2p2(x) + a3p3(x) = 0 implica a1 = a2 = a3 = 0, onde a1, a2, a3
18) (AFA-95) Se o polinômio: P(x) = x5 + 2x3 – x2 + x +  são números reais. Caso contrário, dizemos que p1(x), p2(x)
for divisível por: D(x) = x3 – 2x2 – x + 2, então  +  +  e p3(x) são linearmente dependentes (L.D.). Os polinômios
será: p1(x) = x2 + 2x + 1, p2(x) = x2 + 1 e p3(x) = x2 + 2x + 2 são:
a) 6 b) 17 c) 28 d) 25 a) L.I.
b) nem L.I. nem L.D.
19) (Escola Naval-91/92) O resto da divisão de 1 + x + x2 c) L.I. se p1(x), p2(x) e p3(x) tiverem raízes reais
+ … + x100 por x2 – 1 é: d) L.D.
a) 0 b) x + 1 c) 50x + 50 d) 50x + 51 e) e) nda
51x + 50
58) (ITA-68) Suponhamos que os polinômios P(x), Q(x), p(x)
e q(x) satisfazem as seguintes condições:
P(x).p(x) + Q(x).q(x) = 1 para todo x complexo
20) (Escola Naval-93/94) 2x4 – x3 + mx2 + 2n é divisível P(p(1)) = 0, Q(0) = 0
por x2 – x – 2. O valor de m.n é: Assinale a opção correta:
a) – 8 b) – 10 c) – 12 d) – 14 e) – 16 a) P(x) é divisível por S(x) = x
b) P(x) e Q(x) não são primos entre si
49) (ITA) Determine os valores de a e b, tais que os c) Q(p(1)) = 0
polinômios x3 – 2ax2 + (3a + b)x – 3b e x3 – (a + 2b)x + 2a d) p(x) não é divisível por R(x) = x – 1
sejam divisíveis por x + 1? e) p(0) = 0

50) (ITA) Um polinômio P(x) dividido por x2 + x + 1 dá resto 59) (ITA-69) Os coeficientes A, B, C e D do polinômio P(x)
– x + 1 e dividido por x2 – x + 1 dá resto 3x + 5. Qual o = Ax3 + Bx2 + Cx + D devem satisfazer certas relações para
resto da divisão de P(x) por x4 + x2 + 1? que P(x) seja um cubo perfeito. Assinale a opção correta
para que isto se verifique:
51) (ITA) Se P(x) é um polinômio do 5 grau que satisfaz as a) D = C2A/3B d) C = B2/3A e D =
3 2
condições B /27A
1 = P(1) = P(2) = P(3) = P(4) = P(5) e P(6) = 0, ache P(0). b) C = B/3A3 e D = B2/27A3 e) nda
c) BC = 3A e CD2 = B2A2
52) (ITA-61) Qual a condição necessária necessária e
suficiente que devem satisfazer p e q de modo que P(x) = xp 60) (ITA-70) Um polinômio P(x) = ax3 + bx2 + cx + d é tal
+ 2aqxp – q + ap seja divisível por x + a (p, q  N e p > q). que P(– 2) = – 2, P(– 1) = 3, P(1) = –3 e P(2) = 2. Temos, então,
que:
53) (ITA-62) Se x3 + px + q é divisível por x2 + ax + b e x2 a) b = 0 b) b = 1 c) b = 2 d) b = 3 e) nda
+ rx + s, demostrar que b = – r(a + r).
61) (ITA-71) Dividindo o polinômio P(x) = x3 + x2 + x + 1 pelo
54) (ITA-67) Um polinômio P(x), dividido por x – 1 dá resto polinômio Q(x) obtemos o quociente S(x) = 1 + x e o resto
3. O quociente desta divisão é então dividido por x – 2, R(x) = x + 1. O polinômio Q(x) satisfaz:
obtendo-se resto 2. O resto da divisão de P(x) por (x – 1)(x a) Q(2) = 0 d) Q(1)  0
– 2) será: b) Q(3) = 0 e) N.d.r.a.
a) 3x + 2 b) 3x – 1 c) 2x + 1 d) – x + 4 e) c) Q(0)  0
nda
62) (ITA-71) Seja P(x) = a0 + a1x + a2x2 + a3x3 + … + a100x100,
55) (ITA-67) Um polinômio P(x) dividido por x + 1 dá resto – onde a100 = 1, um polinômio divisível por (x + 9)100. Nestas
1, por x – 1 dá resto 1 e por x + 2 dá resto 1. Qual será o condições temos:
resto da divisão do polinômio por (x + 1)(x – 1)(x – 2)?
a) x2 – x + 1 b) x – 1 c) x2 + x + 1 d) x2 – x – 1
e) nda

MATEMÁTICA – Prof. DOUGLAS OLIVEIRA biblioteca


telegram t.me/bibliotecaPE @PROFDOUGLASOLIVEIRA _2020 69
APROVADO DE A A Z
TURMA IME ITA
100 ! 99!
a) a2 = 50.99.998 b) a2 = c) a2 =
2!98 ! 2!98 ! 69) (ITA-87) Considere Q(x) e R(x), respectivamente, o
100 !92 quociente e o resto da divisão de um polinômio A(x) pelo
d) a2 = e) nda trinômio B(x) = – x2 + 5x – 6. Admita que o grau de A(x) é
2!98 !
quatro e que os restos da divisão de A(x) por x + 1 e x – 2
63) (ITA-71) Qual o resto da divisão por x – a, do polinômio são, respectivamente, 3 e – 1. Supondo também que Q(x) é
divisível por x + 1, obtenha R(x).
1 x x2 x3
1 a a2 a3 70) (ITA-88) Se P(x) e Q(x) são polinômios com
1 b b 2
b3 coeficientes reais, de graus 2 e 4 respectivamente, tais que
2
P(i) = 0 e Q(i) = 0 então podemos afirmar que:
1 c c c3
a) P(x) é divisível por x + 1.
a) 2x3 + c b) 6x2 + 7 c) 5 d) 0 e) nda b) P(x) é divisível por x – 1.
c) P(x).Q(x) é divisível por x4 + 2x2 + 1.
64) (ITA-72) Seja a equação P(x) = 0, onde P(x) é um
d) P(x) e Q(x) são primos entre si.
polinômio de grau m. Se P(x) admite uma raiz inteira, então
e) Q(x) não é divisível por P(x).
P(– 1).P(0).P(1) necessariamente:
a) vale 5 d) é divisível por 3
71) (ITA-88) Sejam A(x) e B(x) polinômios de grau maior
b) vale 3 e) nenhuma das
que um e admita que existam polinômios C(x) e D(x) tais que
respostas anteriores.
a igualdade a seguir se verifica: A(x).C(x) + B(x).D(x) = 1, 
e) é divisível por 5
x  R. Prove que A(x) não é divisível por B(x).
65) (ITA-82) Sabendo-se que o polinômio P(x) = ax3 + bx2 +
2x – 2 é divisível por (x + 1) e por (x – 2), podemos afirmar
72) (ITA-91) Na divisão de P(x) = a5x5 + 2x4 + a4x3 + 8x2 –
que
32x + a3 por x – 1, obteve-se o quociente Q(x) = b4x4 + b3x3
a) a e b tem sinais opostos e são inteiros
+ b2x2 + b1x + b0 e o resto – 6. Sabe-se que (b4, b3, b2, b1) é
b) a e b tem o mesmo sinal e são inteiros
uma progressão geométrica de razão q > 0 e q  1. Podemos
c) a e b tem sinais opostos e são racionais não inteiros
afirmar que:
d) a e b tem o mesmo sinal e são racionais não inteiros
e) somente a é inteiro a) b3 + a3 = 10
b) b4 + a4 = 6
c) b3 + b0 = 12
d) b4 + b1 = 16
66) (ITA-82) Os valores de ,  e  que tornam o polinômio e) n.d.a.
P(x) = 4x5 + 2x4 – 2x3 + x2 + x +  divisível por Q(x) = 2x3 +
x2 – 2x + 1 satisfazem as desigualdades x3 + 4 1 bx + c
a)  >  >  d)  >  >  73) (ITA-94) A identidade: = 1+ + é
x3 + 1 x + 1 x2 − x + 1
b)  >  >  e)  >  > 
válida para todo real x  – 1. Então a + b + c é igual a:
c)  >  > 
a) 5 b) 4 c) 3 d) 2 e) 1

67) (ITA-83) Determine o polinômio P de 3o grau que


74) (ITA-95) A divisão de um polinômio P(x) por x2 – x
representa uma raiz nula e satisfaz a condição P(x – 1) =
resulta no quociente 6x2 + 5x + 3 e resto –7x. O resto da
P(x) + (2x)2 para todo x real. Com o auxílio deste, podemos
divisão de P(x) por 2x + 1 é igual a:
calcular a soma 22 + 42 + … + (2n)2, onde n é um número
a) 1 b) 2 c) 3 d) 4 e) 5
natural, que é igual a:
4 3 2
a) n − 2n2 − n d) 4n3 + 2n2 + n 75) (ITA-97) Sejam p1(x), p2(x) e p3(x) polinômios na
3 3
variável real x de graus n1, n2 e n3, respectivamente, com n1
4 3 2 2
b) n + 2n + n e) n3 + n2 + 2n > n2 > n3. Sabe-se que p1(x) e p2(x) são divisíveis por p3(x).
3 3
Seja r(x) o resto da divisão de p1(x) por p2(x). Considere as
4 3 2 2
c) n − 2n + n afirmações:
3 3
(I) r(x) é divisível por p3(x)

68) (ITA-86) Sejam a, b e c números reais que nesta ordem (II) p1(x) – p2(x) é divisível por p3(x)
formam uma progressão aritmética de soma 12. Sabendo-se
que os restos das divisões de x10 + 8x8 + ax5 + bx3 + cx por (III) p1(x)r(x) é divisível por [p3(x)]2
x – 2 e x +2 são iguais, então a razão desta progressão Então,
aritmética é: a) Apenas (I) e (II) são verdadeiras
a) 1 b) 28/5 c) 37/5 d) 44/15 e) – b) Apenas (II) é vardadeira
3 c) Apenas (I) e (III) são verdadeiras

MATEMÁTICA – Prof. DOUGLAS OLIVEIRA biblioteca


telegram t.me/bibliotecaPE @PROFDOUGLASOLIVEIRA _2020 70
APROVADO DE A A Z
TURMA IME ITA
d) Todas as afirmações são verdadeiras
e) Todas as afirmações são falsas 88) (IME-92/93) Determine o valor de x para que:
x 2 4 6
76) (ITA-98) Seja p(x) um polinômio de grau 4 com x x+ 2 0 10
coeficientes reais. Na divisão de p(x) por x – 2 obtém-se =0
x2 0 4x 4
um quociente q(x) e resto igual a 26. Na divisão de p(x) por
x2 + x – 1 obtém-se um quociente h(x) e resto 8x – 5. Sabe- x 4 10 x− 2
se que q(0) = 13 e q(1) = 26. Então, h(2) + h(3) é igual a:
a) 16 b) zero c) – 47 d) – 38 e) 1 89) (IME-94/95) Prove que o polinômio P(x) = x999 + x888 +
x777 + … + x111 + 1 é divisível por x9 + x8 + x7 + … + x + 1.
77) (ITA-99) Seja p(x) um polinômio de grau 3 tal que p(x)
= p(x + 2) – x2 – 2, para todo x  . Se – 2 é raiz de p(x), 90) (IME-96/97) Determine o resto da divisão do polinômio
então o produto de todas as raízes de p(x) é: (cos  + x.sen )n por (x2 + 1), onde n é um número natural.
a) 36 b) 18 c) – 36 d) – 18 e) 1
91) (IME-97/98) Determine ,  e  de modo que o
78) (IME) Determine os polinômios P(x) do 4 grau tais que  +1 
P(x) = P(1 – x)
polinômio  .x +  .x + 1
, racional inteiro em x,
seja divisível por (x – 1)2 e que o valor numérico do
79) (IME) O polinômio P(x) dividido por x + 2 dá resto 1, por quociente seja igual a 120 para x = 1.
x + 1 dá resto –1 e por x – 1 dá resto 1. Qual o resto da
divisão de P(x) por (x + 2)(x2 – 1) 92) (IME-98/99) Seja o polinômio P(x) de grau (2n + 1) com
todos os seus coeficientes positivos e unitários. Dividindo-
80) (IME) Determinar um polinômio inteiro em x, P(x), se P(x) por D(x), de grau 3, obtém-se o resto R(x).
verificando a identidade: Determine R(x), sabendo-se que as raízes de D(x) são
P(x + 2) – 2P(x + 1) + P(x) = x raízes de A(x) = x4 – 1 e que D(x)  0.

81) (IME) O polinômio P(x), do 3o grau, é tal que P(n) = n3,


para n = 1, 2, 7 e 200. Calcule P(– 1). Demonstre que se P(x) e Q(x) são divisíveis por H(x), então
o resto r(x) da divisão de P(x) por Q(x) também é divisível
por H(x).
82) (IME-74/75) Dado o polinômio 2x4 + x3 + px2 + qx + 2,
determine p e q de modo que seja divisível por (x – 1)2. Na divisão de um polinômio P(x) por A(x) o resto é r1(x); na
divisão de P(x) por B(x) o resto é r2(x); na divisão de P(x)
por A(x).B(x) o resto é r(x). Demonstre que se r(x) é
dividido por A(x) o resto é r1(x) e dividido por B(x) o resto
83) (IME-76/77) Seja P3(x) = (x + 1)(x + 3)(x + 5) + k(x + é r2(x).
2)(x + 4), onde x  C. Determine o lugar geométrico das
raízes de P3(x) quando k assume os valores em +, Sabendo que os restos da divisão de P(x) por (x – a) é r1 e
desenhando este lugar geométrico no plano complexo. de P(x) por (x – b) é r2 (a  b), demonstre que o resto da
divisão de P(x) por (x – a)(x – b) é dado por
84) (IME-79/80) Determine o polinômio p(x) de r1 − r2 ar − br1
coeficientes racionais e do 7 grau, sabendo que
r ( x) = x+ 2
a −b a −b
p(x) + 1 é divisível por (x – 1)4 e que p(x) – 1 é divisível por
(x + 1)4
(V. LIDSKY) Prove que todas as raízes da equação x2 + x +
85) (IME-82/83) Determine o polinômio P(x) do 4o grau,
1 satisfazem a equação
sabendo que P’’(x) = ax2 + bx + c e que P(x) é divisível por
x3m + x3n + 1 + x3p + 2 = 0
P’’(x).
onde m, n e p são números inteiros arbitrários.

86) (IME-83/84) Determine o polinômio P(x) = x4 + ax3 +


(V. LIDSKY) Para que valores de a e n o polinômio xn – axn –
bx2 + cx + d tal que P(x) = P(1 – x), 1
+ ax – 1 é divisível por (x – 1)2
P(0) = 0 e P(–1) = P(6)
(V. LIDSKY) A divisão do polinômio p(x) por x – a dá resto
87) (IME-84/85) Sabe-se que as raízes do polinômio abaixo
A, a divisão por x – b dá resto B e a divisão por x – c dá
são todas reais e distintas:
resto C. Encontre o resto da divisão de p(x) por (x – a)(x –
f(x) = anxn + ... + a1x + a0 ; onde ai  R, i = 0, 1, ..., n; an  0
b)(x – c), com a condição de que a, b e c são diferentes um
Mostre que a derivada f ‘(x) possui também todas as raízes
do outro.
reais e distintas.

MATEMÁTICA – Prof. DOUGLAS OLIVEIRA biblioteca


telegram t.me/bibliotecaPE @PROFDOUGLASOLIVEIRA _2020 71
APROVADO DE A A Z
TURMA IME ITA
(SOMINSKY) Sobre quais condições o polinômio x3 + px + (SOMINSKY) Prove que
q é divisível por um polinômio da forma x x(x − 1) x(x − 1)(x − 2) x(x − 1)...(x − n + 1)
P(x) = 1− + − + ... + (−1)n
x2 + mx – 1? 1 1.2 1.2.3 1.2.3...n
também pode ser expresso da forma
(SOMINSKY) Sobre quais condições o polinômio x4 + px2 + (−1)n
P(x) = (x − 1)(x − 2)...(x − n)
q é divisível por um polinômio da forma n!
x2 + mx + 1?
(SOMINSKY) Determine o coeficiente a tal que o polinômio (SOMINSKY) Prove que se o polinômio P(x) com
x5 – ax2 – ax + 1 tenha –1 como uma raiz de multiplicidade coeficientes inteiros admite o valor 1 para mais do que três
não menor do que 2. valores inteiros distintos de x, então P(x) não pode assumir
o valor –1 para valores inteiros de x.
(SOMINSKY) Determine A e B tal que o polinômio Ax4 +
Bx3 + 1 seja divisível por (x – 1)2.
Questões de Olimpíadas
(SOMINSKY) Determine A e B tal que o polinômio Axn+1 +
Bxn + 1 seja divisível por (x – 1)2. (Brasil) Um certo polinômio p(x) quando dividido por x –
a, x – b e x – c deixa resto a, b, c, respectivamente.
(SOMINSKY) Prove que os seguintes polinômios possuem 1 Qual é o resto da divisão de p(x) por (x – a).(x – b).(x –
como raiz tripla. c)?
a) x2n – nxn+1 + nxn-1 – 1. (NOTA: os números a, b e c são distintos).
b) x2n+1 – (2n + 1)xn+1 + (2n + 1)xn – 1. a) a + b + c b) x2 c) abc d) x
c) (n – 2m)xn – nxn-m + nxm – (n – 2m). e) – (a+b+c)

(SOMINSKY) Qual a condição necessária para que x3m - (Noruega-96 2 Fase) Dado p(x) = x6 + ax5 + bx4 + cx3 + dx2
x3n+1 + x3p+2 seja divisível por x2 – x + 1? + ex + f um polinômio tal que:
p(1) = 1, p(2) = 2, p(3) = 3, p(4) = 4, p(5) = 5 e p(6) = 6.
(SOMINSKY) Qual a condição necessária para que Quanto vale p(7)?
x4 + x2 + 1 divida x3m + x3n+1 + x3p+2 ? a) 0 b) 7 c) 14 d) 49 e) 727

(SOMINSKY) Qual a condição necessária para que Dado p(x) = xn + a1xn-1 + a2xn-2 + ... + an-1x + an , n  N,
x2 + x + 1 divida x2m + xm + 1 ? um polinômio tal que:
p(1) = 1, p(2) = 2n, p(3) = 3n, p(4) = 4n, ..., p(n - 1) = (n - 1)n,
(SOMINSKY) Para quais valores de m x2 + x + 1 divide p(n) = nn. Calcule p(n + 1).
(x +1)m – xm – 1?
(Argentina-93) Seja f(x) = x4 + x3 + x2 + x + 1. Encontrar o
(SOMINSKY) Para quais valores de m x2 + x + 1 divide resto da divisão de f(x5) por f(x).

(x +1)m + xm + 1? (Polônia-92) Determine o polinômio p(x) de coeficientes


racionais e do 5 grau, sabendo que p(x) + 1 é divisível por
(SOMINSKY) Para quais valores de m (x2 + x + 1)2 divide (x – 1)3 e que p(x) – 1 é divisível por (x + 1)3
(x +1)m – xm – 1?
(Polônia-93) Dado um polinômio P com coeficientes inteiros,
(SOMINSKY) Para quais valores de m (x2 + x + 1)2 divide 2 divide P(5), 5 divide P(2) e
(x +1)m + xm + 1? P(0) = 1. Mostre que 10 divide P(7).

(SOMINSKY) Pode (x2 + x + 1)3 dividir o polinômio (International Mathematical Talent Search) Seja
(x +1)m + xm + 1 e (x +1)m – xm – 1? f(x) = x4 + 17x3 + 80x2 + 203x + 125. Determine o polinômio

(SOMINSKY) Prove que se P(xn) é divisível por x – 1, então


( ) (
g(x), de menor grau, tal que f 3  3 = g 3  3 e )
também é divisível por xn – 1. (
f 5 5 ) = g( 5  5 ) .
(SOMINSKY) Prove que se P(xn) é divisível por (x – a)k,
(International Mathematical Talent Search) Seja f(x) um
então também é divisível por (xn – a)k.
polinômio com coeficientes inteiros, e assuma que f(0) = 0 e
f(1) = 2. Prove que f(7) não é um quadrado perfeito.
(SOMINSKY) Prove que se P(x) = p1(x3) + xp2(x3) é
divisível por x2 + x + 1, então p1(x) e p2(x) são divisíveis
(Bélgica-93) O produto
por x – 1.

MATEMÁTICA – Prof. DOUGLAS OLIVEIRA biblioteca


telegram t.me/bibliotecaPE @PROFDOUGLASOLIVEIRA _2020 72
APROVADO DE A A Z
TURMA IME ITA
(1 + x + x2 + … + x100)(1 + x + x2 + … + x25) é um polinômio na (Canadá) Mostre que x12 – x9 + x4 – x + 1 > 0
variável x. O coeficiente de x50 é:
a) 1 b) 25 c) 26 d) 50 e) 51 (Canadá) Prove que se p(x) é um polinômio com coeficientes
inteiros, e p(0) e p(1) são ambos ímpares, então a equação
(Bélgica-97) P e Q são polinômios reais em x, p(x) = 0 não possui soluções inteiras.
respectivamente de graus m e n com 0< n < m. O grau de (P
– Q)(P + Q) vale: (Canadá) Suponha que p(x) = ax2 + bx + c é um polinômio
a) 2m b) m2 c) n2 d) mn e) m2 – n2 quadrático, tal que a equação p(x) = x não possui soluções
reais. Demonstre que a equação p(p(x)) = x não possui
(Argentina) Demonstrar que o resultado da seguinte soluções reais.
expressão não possui termos de ordem ímpar:
(1- x + x2 - x3 + ... - x99 + x100)(1 + x + x2 + x3 + ... + x99 + x100) (Canadá) Fatore x4 – 4x – 1 em um produto de dois
polinômios de graus positivos com coeficientes reais.
(Argentina) Se considera um polinômio P(x) de grau 100,
com coeficientes inteiros, todos distintos entre si, e cujos (Canadá-70) Dado o polinômio: p(x) = xn + a1xn-1 + a2xn-2 + … +
valores são menores ou iguais que 50. Determinar se P(x) é an-1x + an com coeficientes inteiros. Sabendo que para
divisível por (x – 1). quatro inteiros distintos a, b, c e d: f(a) = f(b) = f(c) = f(d)
= 5, prove que não existe nenhum inteiro k tal que f(k) = 8.
(Irlanda-93) O polinômio P(x) = xn + an-1xn-1 + ... + a0, com n >
0 e todos os ai reais, verifica |P(0)| = P(1) e cada raiz xi é (Canadá-81) P(x) e Q(x) são dois polinômios tais que
real e satisfaz 0 < xi < 1. Provar que o produto das raízes satisfazem a identidade P(Q(x)) = Q(P(x)) para todo número
não é maior que 1/2n. real x. Se as equações P(x) = Q(x) não possuem solução real,
mostre que a equação P(P(x)) = Q(Q(x)) também não possui
(Irlanda-97) Determine todos os polinômios p(x) solução real.
satisfazendo a equação: (x – 16)p(2x) = 16(x – 1)p(x) para
todo x. (1ª Olimpíada Brasileira) Mostre que o número de soluções
inteiras positivas da equação
(EUA-96) Encontre todas as soluções de: (x – 1)x(x + 1)(x + x1 + 8x2 + 27x3 +... + 1000x10 = 3025 (*)
2) = –1 é igual ao número de soluções inteiras não negativas de
y1 + 8y2 + 27y3 + ... + 1000y10 = 0.
Usando este fato, conclua que a equação (*) tem uma única
solução inteira e positiva.
Determine esta solução.

(OLÍMPIADA) Resolva a equação (x – 2).(x – 3).(x – 4).(x –


(Áustria-94) Os polinômios Pn(x) são definidos 5) = 360 nos naturais.
recursivamente:
P0(x) = 0 P1(x) = x Pn(x) = xPn-1(x) + (1 – x)Pn-2(x) 12) Qual o resto da divisão de x + x9 + x25 + x49 + x81 por
para n  2. x3 – x?
Para cada inteiro n  1, encontre todos os números reais x 13) Qual o resto da divisão de x100 + 2x99 + 2x + 5 por
tais que Pn(x) = 0. x2 + x – 2.

(Inglaterra-95) O polinômio P(x) = xn + a1xn-1 + ... + an-1x + 1,


onde os coeficientes a1, a2, ... ,an-1 são não negativos, possui
n raízes reais. Prove que P(2)  3n

(Peru-90) Seja P(x) um polinômio com coeficientes inteiros,


TEORIA DOS NÚMEROS
que possui 13 diferentes raízes inteiras. Provar que se n é
um inteiro com P(n)  0 então |P(n)|  7.(6!)2
01
(Brasil-98) O menor múltiplo de 1998 que possui apenas os
(Canadá) As raízes da equação x2 + 4x – 5 = 0 são também
algarismos 0 e 9 é 9990. Qual é o menor múltiplo de 1998
raízes da equação 2x3 + 9x2 – 6x – 5 = 0. Qual é a terceira
que possui apenas os algarismos 0 e 3?
raiz da segunda equação?

02
(Canadá) Mostre que não existe um polinômio cúbico p(x) =
ax3 + bx2 + cx + d com coeficientes inteiros a, b, c, d tais (Brasil-98) São dados 15 números naturais maiores que 1 e
que p(62) = 2 e p(19) = 1. menores que 1998 tais que dois quaisquer são primos entre
si. Mostre que pelo menos um desses 15 números é primo.

MATEMÁTICA – Prof. DOUGLAS OLIVEIRA biblioteca


telegram t.me/bibliotecaPE @PROFDOUGLASOLIVEIRA _2020 73
APROVADO DE A A Z
TURMA IME ITA
14
03 (Canadá-78) Seja n um inteiro. Se o dígito das dezenas de
(Argetina-96) Se n é um número natural, d(n) é o número de n2 é 7, qual o dígito das unidades de n2?
divisores positivos de n. (Canadá-78) Determine todos os pares a e b de inteiros
Por exemplo, d(12) = 6, pois os divisores são 1, 2, 3, 4, 6 e positivos satisfazendo 2a2 = 3b3.
12. (Canadá-80) Se a679b é um número de 5 dígitos (em base
Calcular todos os números naturais n  200 tais que n/d(n) 10) que é divisível por 72, determine a e b.
= 8.
15
04 (Canadá-83) Determine todos os inteiros positivos w, x, y, z
(Argentina-98) Determinar o maior natural n tal que 419 + tais que w! = x! + y! + z!.
498 + 4n é um quadrado perfeito.
16
05 (Canadá-91) Mostre que a equação x2 + y5 = z3 possui
2
(Pré Torneio das Cidades-97) Demonstrar que a equação x infinitas soluções inteiras x, y, z para xyz  0.
+ y2 – z2 = 1997 tem infinitas soluções inteiras.
17
06 (Alberta High School Mathematics Competition-97)
(Número de Ouro-97) Prove que (377 – 1)/2 é ímpar e (a) Suponha que p é um primo ímpar e a e b são
composto. inteiros positivos tais que p4 divide a2 + b2 e p4
também divide a(a + b)2. Prove que p4 também
07 divide a(a + b).
(México-87) Quantos inteiros positivos dividem 20!? (20! = (b) Suponha que p é um primo ímpar e a e b são
1.2.3....19.20) inteiros positivos tais que p5 divide a2 + b2 e p5
também divide a(a + b)2. Mostre através de um
08 exemplo que p5 não necessariamente divide a(a +
(Canadá-69) Mostre que não existem números inteiros a, b, b).
c tais que a2 + b2 – 8c = 6
18
09 (APMO-97) Determine um inteiro n, onde 50  n  1997, tal
2
(Canadá-71) Mostre que, para todos os inteiros n, n + 2n + que n | 2n + 2.
12 não é múltiplo de 121.
19
10 (Bulgária-95) Determine todos os pares de inteiros
(Canadá-71) Seja n um número de cinco dígitos (cujo x 2 + y2
primeiro dígito é não-nulo) e seja m um número de quatro positivos (x, y) para os quais é um inteiro que é
dígitos formado de n com o apagamento de seu dígito do
x−y
meio. Determine todos os números n tais que n/m é um divisor de 1995.
inteiro.
20
11 (Bulgária-95) Determine o número de todos os inteiros n > 1,
3 3 3
(Canadá-72) Prove que a equação x + 11 = y não possui para os quais o número a25 – a é divisível por n, para todo a.
solução nos inteiros positivos x, y.
21
(Bulgária-95) Prove que para todo inteiro positivo n a
seguinte proposição é válida:
“O número 7 é um divisor de 3n + n3 se e somente se 7 é um
divisor de 3n.n3 + 1.”
12
(Canadá-73) Prove que se p e p + 2 são ambos números 22
inteiros primos maiores que 3, então 6 é um fator de p + 1. (Bulgária-95) Determine todos os números primos p para os
quais o número p(2p – 1 – 1) é a k-ésima potência (k > 1) de um
13 inteiro positivo.
(Canadá-77) Se f(x) = x2 + x, prove que a equação 4f(a) =
f(b) não possui solução em inteiros positivos a e b. 23

MATEMÁTICA – Prof. DOUGLAS OLIVEIRA biblioteca


telegram t.me/bibliotecaPE @PROFDOUGLASOLIVEIRA _2020 74
APROVADO DE A A Z
TURMA IME ITA
(Bulgária-96) Prove que para todo inteiro positivo n  3 35
existem inteiros positivos ímpares xn e yn, tal que (Cone Sul-97) A cada número inteiro positivo n, n  99,
7xn2 + yn2 = 2n. subtrai-se a soma dos quadrados de seus dígitos. Para que
valores de n, esta diferença é a maior possível?.
25
(Bulgária-96) Determine todos os números primos p e q para 36
os quais (5p – 2p)(5q – 2q)/pq é um inteiro. (Cone Sul-98) Prove que, pelo menos para 30% dos naturais
n entre 1 e 1.000.000, o primeiro dígito de 2n é 1.
26
(Itália-97) Demonstrar que não existem dois inteiros m e n 37
tais que 19m2 + 95mn + 2000n2 = 1995. (IMO-88) Sejam a e b inteiros positivos tal que ab + 1
. divide a2 + b2. Mostre que (a2 + b2)/(ab + 1) é um quadrado
27 perfeito.
(Espanha-99) Provar que existe uma seqüência de inteiros
positivos a1, a2,…, an, … tal que 38
a12 + a22 +…….+ an2 (IMO-98) Determine todos os pares (a, b) de inteiros
é um quadrado perfeito para todo inteiro positivo n. positivos tais que ab2 + b + 7 divide a2b + a + b.

28
(Báltica-92) Sejam p, q dois números primos ímpares SÓ Divisibilidade
consecutivos. Prove que p + q é um produto de ao menos 3
números naturais > 1 (não necessariamente diferentes). 39
Prove que se a e b são números naturais, então a!b! | (a + b)!
29

2 p −1 − 1 40
(Irã-93) Encontrar todos os primos p tais que é Prove que, para todo número natural k, o produto
p P = (a + 1)(a + 2)...(a + k) é divisível por k!.
quadrado perfeito.
41
30 Prove que se a1, a2, ..., an são números naturais (m > 2), então
(Cone Sul-88) Considera-se um número n de quatro dígitos, a1!a2!...am! | (a1 + a2 + ... + am)!
quadrado perfeito, tal que todos seus dígitos são menores
que 6. Se a cada dígito é somado 1, o número resultante é 42
outro quadrado perfeito. Achar n. Prove que se m e n são números naturais e m é ímpar, então
mdc (2m – 1, 2n + 1) = 1
31
(Cone Sul-92) Achar um número inteiro positivo n de 43
maneira tal que se à sua expressão é colocado um 2 pela Prove que para cada número natural n temos mdc [n! + 1, (n +
esquerda e um 1 pela direita, o número resultante seja igual 1)! + 1] = 1
a 33n.
44
32
Prove que se a, b, c são inteiros tais que mdc (a, b) = 1 e
(Cone Sul-92) Prove que não existem números inteiros c | a + b, então mdc (c, a) = 1 e mdc (c, b) = 1.
positivos x, y, z que satisfaçam x2 + y2 = 3z2
45
33
Prove que se a e b são dois números primos relativos e m é
(Cone Sul-93) Prove que existe uma seqüência a1 ,..., ak ,..., um número natural arbitrário, então a progressão
onde cada ai é um dígito (ou seja ai pertence a { 0, 1, 2, 3, 4, aritmética a + bk (k = 0, 1, 2, ...) possui infinitos números
5, 6, 7, 8, 9 }) e a0 = 6, tal que para cada inteiro positivo n o primos relativos com m.
número xn = a0 + 10a1 + 100a2 + ... + 10n – 1an – 1 verifica que
xn2 – xn é divisível por 10n.
34
(Cone Sul-94) Determinar infinitos ternos x, y, z de 46
inteiros positivos que sejam soluções da equação x2 + y2 = Se os números naturais a, b, c, d satisfazem as condições
2z2, tais que o máximo divisor comum de x, y, z seja 1. mdc (c, d) = 1 e ac = bd, então existe um número natural n
tal que a = nd e b = nc.

MATEMÁTICA – Prof. DOUGLAS OLIVEIRA biblioteca


telegram t.me/bibliotecaPE @PROFDOUGLASOLIVEIRA _2020 75
APROVADO DE A A Z
TURMA IME ITA
58
47 Prove que a equação 3n + 4n = 5n possui somente uma
Se a e b são dois números primos entre si, então todo solução nos números naturais n, n = 2.
número natural n > ab pode ser escrito na forma n = ax + by,
onde x, y são números naturais. 59
Prove que em toda solução inteira da equação x2 + y2 = z2, ao
48 menos um dos números x, y é divisível por 3.
Prove que, se n é um número natural > 1 e d(n) o número de
divisores positivos de n, então a seqüência infinita: n, d(n), 60
d(d(n)), ddd(n), ... independente do termo inicial n, sempre Prove que em toda solução inteira da equação x2 + y2 = z2, ao
tende para 2. menos um dos números x, y, z é divisível por 5.

49 61
Seja (n) a soma dos divisores positivos de n, prove que a Determine todas as soluções inteiras para a equação x2 + y2
igualdade (n) = n + 1 vale se e somente se n é primo. = z2, com mdc (x, y, z) = , no qual z e y são números
consecutivos.
50
Seja (n) a soma dos divisores positivos de n, prove que, 62
para todo número natural m, existem números naturais x, y Prove que não existe dois números naturais cuja soma e
tais que x – y  m e (x2) = (y2). diferença de seus quadrados sejam quadrados.

51 63
Determine todos os números naturais cuja soma dos Prove que o sistema de equações x2 + y2 = u2 e x2 + 2y2 =
divisores positivos é um número ímpar. v2 não possui soluções naturais.

52 64
Seja (n) a soma dos divisores positivos de n, prove que, se Prove que não existem números naturais a, b, c tais que a4 –
n é um número composto, então temos b4 = c2.
(n) > n + (n)1/2.
65
53 Prove que não existem números naturais x, y, z
Defini-se número perfeito como o número cuja soma dos satisfazendo x4 + y4 = z4
divisores positivos é igual a duas vezes o número, ou seja,
(n) = 2n. Prove que se n é um número par, a condição 66
necessária e suficiente para que n seja perfeito é que seja Prove que a equação (x2 – 1)2 + (y2 – 1)2 = (z2 + 1)2 possui
da forma 2s – 1(2s – 1), onde s é um natural e 2s é um primo. infinitas soluções naturais x, y, z.

54 67
Prove que existem infinitos números naturais ímpares tais Prove que não existem três quadrados formando uma
que (n) > 2n. progressão aritmética com a razão sendo um quadrado.

55 68
Determine todos os números naturais n tais que n é igual ao Prove que não existe um triângulo pitagórico cuja área é o
produto de todos os divisores positivos de n, excetuando n. quadrado de um número natural.

69
Prove que a equação x4 + y4 = z2 não possui solução nos
SÓ Equações Diofantinas números naturais x, y, z.
56
Prove que a equação x3 + y3 + z3 = 4 não possui soluções
inteiras.

57 70
Determine todas as soluções inteiras para a equação x2 + y2 Prove que a equação t4 + u4 + v4 = w4 possui infinitas
= z2, com mdc (x, y, z) = 1. soluções naturais t, u, v, w com mdc (t, u, v, w) = 1.

MATEMÁTICA – Prof. DOUGLAS OLIVEIRA biblioteca


telegram t.me/bibliotecaPE @PROFDOUGLASOLIVEIRA _2020 76
APROVADO DE A A Z
TURMA IME ITA
71 84
4 4 4 4 2 2
Prove que o sistema de equações x + y + z = 2t x +y Prove que a equação x2 + 12 = y3 não possui soluções
+ z2 = 2t2 possui infinitas soluções nos naturais x, y, z, t. inteiras.

72 85
Prove que x4 + y4 = 2z2 não possui soluções naturais x, y, z Prove que a equação x2 + 16 = y3 não possui soluções
com mdc (x, y, z) = 1. inteiras.

73 86
Prove que x4 + y4 = 4z2 não possui soluções naturais x, y, z Prove que a equação x2 = y3 + z5 possui infinitas soluções
com mdc (x, y, z) = 1. naturais.

74 87
Prove que x4 + y4 = 5z2 não possui soluções naturais x, y, z Prove que a equação xn + yn = zn – 1 possui infinitas soluções
com mdc (x, y, z) = 1. naturais.

75 88
2 2 3
Prove que a equação x + y = z possui infinitas soluções Prove que a equação xn + yn = zn + 1 possui infinitas soluções
naturais x, y, z com mdc (x, y, z) = 1. naturais.

76 89
Prove que a equação x2 + y2 = z4 possui infinitas soluções Prove, em conexão com o Último Teorema de Fermat, que se
naturais x, y, z com mdc (x, y, z) = 1. n é um número natural maior que 2, então a equação xn + (x
+ 1)n = (x + 2)n não possui soluções nos números naturais.
77
Prove que existe uma seqüência infinita de números naturais 90
a1, a2, ... tais que cada um dos números a12 + a22 + ... + an2, Resolva a equação xy = yx nos números inteiros x e y, com
onde n = 1, 2, ..., é o quadrado de um número natural. x  y.

78
Um número natural é chamado congruente se existem (ao SÓ Números Primos
menos um) números naturais z tais que cada um dos 91
números z2 + h, z2 – h é o quadrado de um número natural. Dado um número primo cujos dígitos são todos iguais a 1
Descreva um modo simples de determinar números (em expansão decimal) , prove que o número de dígitos deve
congruentes. ser um número primo.

79 92
Determine todas as soluções naturais da equação x2 + y2 + Prove que todo número da forma 8n + 1 é composto.
z2 = t2.
93
80 Prove que todo número natural > 11 é a soma de dois
Determine todas as soluções da equação números compostos.
(x + y + z)3 = x3 + y3 + z3 nos inteiros x, y, z.
94
81 Achar todas as soluções naturais da equação
Encontre todos os pares de números naturais x, y para os
x2 − 2 y 4 = 1
quais xy é divisível por x + y.

82
Determine todas as soluções inteiras da equação 1/x + 1/y
= 1/z PROBABILIDADE
83
Determine todas as soluções da equação (x + y + z)2 = x2 + 01
y2 + z2. Use a identidade (1 − x 2 )n = (1 − x) n (1 + x) n para
calcular, com mn ,

MATEMÁTICA – Prof. DOUGLAS OLIVEIRA biblioteca


telegram t.me/bibliotecaPE @PROFDOUGLASOLIVEIRA _2020 77
APROVADO DE A A Z
TURMA IME ITA
2m
 n  n  (D) 67 (E) 69
 (−1)i    , e
i =0  i  2m − i  07
2 m +1
 n  n  (O problema das caixas de fósforos de Banach)
 (−1)i   . Um homem compra duas caixas de fósforos e guarda-as em
i =0  i  2m + 1 − i  seu bolso. Toda vez que ele quer acender um cigarro, ele
E em particular, calcule , considerando os casos n par e n pega aleatoriamente uma ou outra caixa. Após algum tempo,
2 o homem pega uma caixa, abre-a e constata que ela está
n
n
ímpar,  (−1)  
i vazia ( o homem distraídamente pôs a caixa vazia de volta
i =0 i  em seu bolso após ter pego o último palito). A probabilidade
de neste instante a outra caixa contenha 30 palitos se cada
02 (a)!
caixa continha originalmente 50 palitos é , o valor
Considere o quadrado (b)!(c)!2d
Q = {(n, m); n = 1, 2,...,10, m = 1, 2,...,10} formada de a + b + c + d é igual a :
(A) 90 (B) 120 (C) 150
por 10X10 pontos no plano. Calcule o número de quadrados
(D) 180 (E) 210
contidos em Q, cujos vértices são pontos de Q.

08
03
Cada um dos 2n alunos do SISTEMA TRIVIAL lança uma P( A) = 0, 4, P( B) = 0,5, P(C ) = P( A  B) = 0,3, P( A  C ) = 0 e
moeda honesta para decidir se comparece ou não à aula de P( B  C ) = 0,1,
aprofundamento com o prof. Douglas Oliveira de Lima. Se o
lançamento resultar em cara, o estudante irá comparecer e det er min e :
se der coroa, ele não irá. Mostre que a probabilidade P2n a) P( A  B  C ).
da maioria dos estudantes ir à aula é dada por b) P[ A − ( B  C )]
1 1 (2n − 1)(2n − 3)(2n − 5)...5.3.1
P2 n = − n+1 e
2 2 n!
09
calcule P20
Para a Seleção Brasileira de Futebol foram pré-convocados
40 jogadores, sendo 20 casados e 20 solteiros. Estes 40
03
jogadores ficarão concentrados para treinamento durante
Dois apostadores participam de um jogo de “cara ou coroa” alguns dias e, para efeito de alojamento serão divididos aos
no qual uma moeda é lançada sucessivamente e cada vez que pares em cada quarto. Se a escolha de cada par é feita
resultar em cara o apostador A ganha 1 real do apostador B aleatoriamente, a probabilidade de que não haja um par
e se resultar em coroa, A paga 1 real para B. Prove que
(a)!m
pi = ppi +1 + (1 − p) pi −1 e resolva essa equação de casado−solteiro possui a forma o valor de a + b + c
(b)!n(c)!
recorrência considerando todos os casos possíveis para + m + n é igual a :
0<p<1, sendo p a probabilidade de resultar cara em cada (A) 75 (B) 85 (C) 95
lançamento da moeda. (D) 100 (E) 105

05
Seja X um subconjunto de {1,2,3,...,99} com 10 elementos.
Mostre que é possível selecionar dois subconjuntos
disjuntos próprios não-vazios Y e Z de X tais que 10

y = z
yY zZ
Cem pessoas formam uma fila para entrar em um teatro
cuja entrada custa R$ 5,00. Sessenta pessoas possuem
06 apenas uma nota de R$ 5,00 e as outras quarenta possuem
Começando no ponto (0, 0), um objeto se move no plano de apenas uma nota de R$ 10,00. Sabendo que o bilheteiro
coordenadas cartesianas com passos de comprimento igual a esqueceu a chave do caixa, a probabilidade de que a todo
1. Cada um destes passos pode ser feito, instante que uma pessoa cheque ao caixa com uma nota de
equiprobabilísticamente, para a esquerda, para a direita, R$ 10,00, o bilheteiro possua uma nota de R$ 5,00 para dá-
m la de troco é igual a :
para cima ou para baixo. Se é a fração irredutível que 3 1 2
n (A) (B) (C)
traduz a probabilidade do objeto alcançar o ponto (2, 2) em 5 5 3
seis ou menos passos, o valor de m + n é igual a : 21 1
(D) (D)
(A) 61 (B) 63 (C) 65 61 3

MATEMÁTICA – Prof. DOUGLAS OLIVEIRA biblioteca


telegram t.me/bibliotecaPE @PROFDOUGLASOLIVEIRA _2020 78
APROVADO DE A A Z
TURMA IME ITA
11 se o candidato mantém sua escolha ou deseja trocar de
Uma seqüência (r1,r2 ,r3 ,...,rn ) de números reais distintos porta. O candidato deve trocar ou não?
pode ser colocada em ordem crescente através de um ou
mais “Passos Ascendentes”. Um “Passo Ascendente” numa 15
dada seqüência consiste no seguinte: comparamos o Dois times bem equilibrados participam de uma série de
primeiro e o segundo termos e os permutamos se, e partidas em “melhor de quatro”. A probabilidade da série
somente se, o segundo termo for o menor. A seguir, terminar em 6 partidas e em 7 partidas, respectivamente,
comparamos o segundo e o terceiro termos e os é:
permutamos se, e somente se, o terceiro termo for o menor a)
6
e
7
b)
3
e
3
c)
12
e
13
d)
1
e
3
e assim sucessivamente até compararmos o último termo, rn 16 16 8 8 32 32 4 8
com o seu atual predecessor e os permutarmos. Por exemplo 5 5
e) e
a seqüência (1,9 ,8 ,7) através de um “Passo Ascendente” é 16 16
transformada na seqüência (1,8 ,7,9 ) . Supondo que n = 40 e 16

que os termos da seqüência inicial (r1,r2 ,...,r40 ) são todos Há 2 vermelhas, 2 pretas, 2 brancas e um número positivo
mas desconhecido de meias azuis em uma gaveta. Está
p
distintos, seja a fração irredutível que traduz a determinado empiricamente que, se duas meias são
q retiradas da gaveta, aleatoriamente e sem reposição, a
probabilidade de que o número que começa como 1
r20 termine, após um passo ascendente, no 30o lugar , o probabilidade de que elas sejam da mesma cor é .
5
valor de p + q é:
Quantas meias azuis há na gaveta?
(A) 931 (B) 933 (C) 935
a) 1 b) 2 c) 3 d) 4 e) 5
(D) 937 (E) 939

17
12
Dois jogadores de tênis A e B disputam uma partida de
25 cavaleiros do Rei Arthur estão sentados em sua habitual
tênis no qual A tem chance 2/7 de partida, e ganha aquele
Távola Redonda. Três deles são escolhidos aleatoriamente e
que completar dois pontos seguidos(Deuce). Qual a
enviados para matar um desagradável dragão. Se p é a
probabilidade de A ganhar?
fração irredutível que traduz a probabilidade de que pelo
menos dois dos três cavaleiros escolhidos estejam sentados
lado a lado, a soma do seu numerador com o seu
18
denominador é igual a :
O jogo de craps é jogado por um jogador com dois dados da
(A) 51 (B) 53 (C) 55
seguinte forma.
(D) 57 (E) 59
Os dados são lançados e:
a) se a soma é 7 ou 11, o jogador ganha imediatamente.
13
b), se a soma é 2,3, ou 12, o jogador perde imediatamente.
Na cidade do sudoeste as pessoas tem chance de 1/3 de
c) se a soma for qualquer outro número, esse número torna-
falar a verdade, A,B,C e D são moradores do sudoeste e D
se o ponto. Os dados são então lançados novamente até o
afirmou algo. A disse que B disse que C disse que D falou a
ponto ou um 7. Se o ponto for rolado antes do 7, o jogador
verdade. Qual a probabilidade de D realmente ter dito a
ganha; se um 7 sair antes do ponto, o jogador perde.
verdade?
Qual é a probabilidade do jogador de ganhar?

14
Dois homens marcam de se encontrar para duelar entre 5h
e 6h. Cada um espera o outro por 5 min e se o oponente não
aparecer ele vai embora. Qual a probabilidade de ocorrer REVISÃO MATEMÁTICA BÁSICA
um duelo?

15 01
Em um retângulo ABCD , tem-se que M é o ponto médio
(Problema de Monty Hall ) Em um programa de prêmios, o do lado AB e E o ponto de interseção de DM com a
candidato tem diante de si três portas. Atrás de uma diagonal AC . Se EM = 4 e EC = 10 , então a área do
destas portas, há um grande prêmio; atrás das demais há retângulo ABCD possui a forma a b . O valor de a + b é
um bode. O candidato escolhe inicialmente uma das portas. igual a:
O apresentador (que sabe qual é a porta que contém o (A) 51 (B) 53 (C) 55
prêmio)abre uma das portas não indicadas pelo candidato, (D) 57 (E) 59
mostrando necessariamente um bode. A seguir ele pergunta

MATEMÁTICA – Prof. DOUGLAS OLIVEIRA biblioteca


telegram t.me/bibliotecaPE @PROFDOUGLASOLIVEIRA _2020 79
APROVADO DE A A Z
TURMA IME ITA
02
Considere a seqüência de números 07
( 24, 2534, 253534, 25353534, 2535353534, ) Sejam a, b, c, d números reais distintos tais que a e b
Se n é o primeiro número da seqüência que é divisível por são as raízes da e equação x 2 − 3cx − 8d = 0 e c e d são as
99 então, o número de dígitos na representação de n na raízes da equação x 2 − 3ax − 8b = 0 . O valor da soma
base dez é igual a: a + b + c + d é igual a:
(A) 170 (B) 172 (C) 174 (A) 32 (B) 64 (C) 72
(D) 176 (E) 178 (D) 96 (E) 120

03 08
Na figura, o círculo de centro O possui raio igual a 10 , e o A seqüência ( a1 , a2 , a3 , ) satisfaz a a1 = 19 , a9 = 99 e
círculo de centro em P possui raio igual a 4 e é tangente
para todo n  3 , an é a média aritmética dos n − 1
ao primeiro. Se QT é tangente ao círculo de centro em P
primeiros termos. O valor de a2 é igual a:
no ponto T e se PT intersecta o círculo de centro O nos (A) 29 (B) 59 (C) 79
pontos A e B , o produto TA  TB é igual a: (D) 99 (E) 179
(A) 20
Q
09
(B) 40
Uma padaria trabalha com os tipos A, B, C e D de

(C) 60 farinha cujos teores de impureza são 8% , 12% , 16,7% e


O 10,7% respectivamente. Para fabricar o tipo D o padeiro
(D) 80 T
mistura certa quantidade de farinha A com 300 gramas
P de farinha B . Em seguida, substitui 200 gramas dessa
(E) 100 mistura por 200 gramas de farinha tipo C . A
quantidade, em gramas, de farinha tipo A utilizado foi:
(A) 250 (B) 300 (C) 500
04
(D) 700 (E) 750
A soma dos algarismos do menor inteiro positive k tal
que k e k + 1 possua mais de quatro divisores positivos 10
cada um é igual a: Um triângulo isósceles ABC , com AC = BC , está inscrito
(A) 6 (B) 7 (C) 8 em um círculo de centro O e o prolongamento do raio
(D) 9 (E) 10 AO intersecta o lado BC no ponto H . Se as áreas dos
triângulos AHC e ABH são iguais a 10 e 16
05
respectivamente e a medida do raio do círculo é dada por
As alturas AD e BE de um triângulo ABC intersectam-
m n
se no seu ortocentro H . Os pontos médios de AB e CH onde m, n , e p são primos entre si dois a dois, os
p
são X e Y respectivamente. Se DY = 65 , DE = 66 e
AB = 366 , a medida de XY é igual a: valores de m + n + p é igual a:
(A) 230 (B) 232 (C) 234 (A) 90 (B) 92 (C) 94
(D) 236 (E) 238 (D) 96 (E) 98

06

(
Seja g n = MDC 2007 + n , 2007 + ( n + 1)
2 2
) onde n é um 11
inteiro positivo. O menor valor de n para o qual g n A distância entre os centros O1 e O2 de dois círculos 1 e
alcança seu valor máximo é igual a:  2 de raios 1 e 2 , respectivamente, é igual a 2 .
(A) (B) (C) Sabendo que os círculos intersectam-se nos pontos A e B ,
(D) (E)

MATEMÁTICA – Prof. DOUGLAS OLIVEIRA biblioteca


telegram t.me/bibliotecaPE @PROFDOUGLASOLIVEIRA _2020 80
APROVADO DE A A Z
TURMA IME ITA
a medida da corda AC do círculo  2 que é dividida ao (A) 1 (B) 2 (C) 3
meio pelo círculo 1 é igual a: (D) 4 (E) mais de 4
14 14
(A) (B) (C) 14
4 2
(D) 2 14 (E) 4 14
DESIGUALDADES
12
Sejam E , F e G pontos sobre os lados AB, BC e CA
01
respectivamente de um triângulo ABC tais que
Sejam x, y e z números reais positivos. Prove que:
AE BF CG 1
= = = . Sejam ainda K , L e M os pontos x3 + y3 + z3  3xyz.
EB FC GA 3
de interseção de AF com CE ; BG com AF e CE com 02
BG respectivamente. O valor da razão entre a área do Prove que, se x e y são números reais positivos e
triângulo KLM e a área do triângulo ABC é igual a: x + y = 1, então xy  1/4.
4 6 9
(A) (B) (C)
13 13 13 03
10 12 Sejam x e y números reais tais que x + y = 1. Mostre que
(D) (E)
13 13 1 + 1 1 + 1   9 .
 x  y
13
Seja P um ponto do interior de um triângulo ABC de 04
incentro I e círculo circunscrito  tal que: Determine o valor máximo de 8x + 6y – 5z, onde x, y e z
PBA + PCA = PBC + PCB são reais e x2 + y2 + z2 = 5.
Considere então as afirmativas:
1. A medida do ângulo BPC = BIC . 05
2. O quadrilátero BCIP é inscritível. Sejam a1, a2, …, an números reais positivos. Se a1 + a2 + … +
3. Se M é o ponto de interseção de AI com o centro de an = 1, prove que a12 + a22 + … + an2  1/n.
 então IM = PM .
Assinale: 06
(A) Se somente a afirmativa (I) for verdadeira Sejam a1, a2, …, an números reais positivos. Se a1a2…an = 1,
(B) Se somente as afirmativas (II) for verdadeira prove que a12 + a22 + … + an2  n.
(C) Se somente a afirmativa (III) for verdadeira
(D) Se somente as afirmativas (II) e (III) forem verdadeiras 07
(E) Se todas as afirmativas forem verdadeiras Se a, b e c são positivos e a + b + c = 1, então prove que
1 + 1 + 1  9.
14 a b c
A quantidade de números pares n  tais que
1 1 1 1620 08
+ + + = Se a, b e c são números reais positivos, prove que:
d1 d2 dk 1003
(i) (a + b + c)(a2 + b2 + c2)  3(a3 + b3 + c3).
onde d1 , d 2 , , d k são todos os divisores distintos de n é
(ii) bc(b + c) + ca(c + a) + ab(a + b)  2(a3 + b3 + c3)
igual a:
(A) 4 (B) 8 (C) 30 09
Prove que se n  N, n > 1, então:
(D) 2006 (E) infinita
1 + 1 + 1 + ... + 1  1 .
4 9 16 n2

10
Prove que se a  0, b  0, c  0, d  0, então:
15
(a + c)(b + d )  ab + cd
O número de pares ordenados de números primos p e q
para os quais p2 + 3 pq + q2 é um quadrado perfeito é: 11

MATEMÁTICA – Prof. DOUGLAS OLIVEIRA biblioteca


telegram t.me/bibliotecaPE @PROFDOUGLASOLIVEIRA _2020 81
APROVADO DE A A Z
TURMA IME ITA
Se a  0, b  0, prove que a 5 + b 5  a 4 b + ab 4 . 2 n − 2  1 + 1 + 1 + ... + 1  2 n − 1
2 3 n
12
a + b  a + b. 23
Se a > 0, b > 0, prove que
Se m é um inteiro positivo, prove que:
b a
1 + 1 + ... + 1 1
13 m +1 m + 2 m + (2m + 1)

Se a + b  0, a  0, b  0, então
a + b 1+1 . 24
b2 a 2 a b
Se n é um número natural, prove que:
14
Se a + b  0, então ab(a + b)  a3 + b3.
1 + 1 + ... + 1  n − 1
2 2 32 n2 n
15 25
Se a1  0, a2  0, …, an  0, prove que: Prove que se 2x + 4y = 1, então x2 + y2  1/20.

a 1a 2 + a 1a 3 + ... + a 1a n + a 2 a 3 + ... + a n −1a n  26


 n − 1 (a 1 + a 2 + ... + a n ) Prove que se a, b e c são números de mesmo sinal e a < b < c,
2 então:
a3(b2 – c2) + b3(c2 – a2) + c3(a2 – b2) < 0
16
Se a > 0, b > 0, c > 0, Prove que: 27
bc + ac + ab  a + b + c Prove que se a + b = 1, então a4 + b4  1/8.
a b c
28
17 Prove que se |x| < 1 e n  2, então:
Se a  0, b  0, c  0, prove que: (1 – x)n + (1 + x)n < 2n.
ab(a + b) + bc(b + c) + ac(a + c)  6abc
29
18 Prove que se a e b são reais positivos tais que
Se a > 0, b > 0, prove que: 2 2
a + b = 1, então  a + 1  +  b + 1   25 .
a+b  a + b  a  b 2
1+ a + b 1+ a 1+ b
30
19
Suponha que a1, a2, …, an são reais positivos e b1, b2, …, bn é
Se n = 2, 3, 4, …, prove que: uma permutação de a1, a2, …, an. Mostre que:
1 + 2 + ... + n  n a1 a 2 a
+ + ... + n  n .
b1 b 2 bn
20
Se n = 2, 3, 4, …, prove que:
31
1 + 1 + ... + 1  1 Suponha que a, b, c são reais positivos. Prove que:
n +1 n + 2 2n 2 a2b2 + b2c2 + c2a2  abc(a + b + c)

21
Se n  2, prove que: 32
n  1 + 1 + 1 + ... + 1  2 n Se a, b, c são reais positivos, prove que:
2 3 n (a2b + b2c + c2a)(a2c + b2a + c2b)  9a2b2c2

22 33
i) Mostre que, sendo n um inteiro positivo: Sejam x1, x2, …, xn + 1 reais positivos tais que:
1 + 1 + ... + 1 = 1,
2 n +1 − 2 n  1  2 n − 2 n −1 1 + x1 1 + x 2 1 + x n +1
n
ii) Use i) para mostrar que: prove que x1x2…xn + 1  nn + 1.

MATEMÁTICA – Prof. DOUGLAS OLIVEIRA biblioteca


telegram t.me/bibliotecaPE @PROFDOUGLASOLIVEIRA _2020 82
APROVADO DE A A Z
TURMA IME ITA
34 a 2 + b2 + c2 + d2  1 ,
Sendo a, b, c reais positivos tais que
1 + 1 + 1 =1, a+b b+c c+d d+a 2
a b c com igualdade se e somente se a = b = c = d = 1/4.
mostre que (a – 1)(b – 1)(c – 1)  8.
44
35 (Rússia-62) São dados quatro números positivos a, b, c, d,
(Brasil-81 banco) Provar as desigualdades: e sabe-se, que abcd = 1. Prove que
a) (a + b)(b + c)(c + a)  8abc, a, b e c reais positivos; a2 + b2 + c2 + d2 + ab + ac + ad + bc + bd + dc  10.
b) (1 + a1)(1 + a2)…(1 + an)  2n,
se a1.a2….an = 1, ai  0. 45
(Leningrado-87) Dados os números reais a, b, c e d. Prove
36 que:
(Brasil-85 banco) Demonstre que, se a, b, c são números (1 + ab)2 + (1 + cd)2 + (ac)2 + (bd)2  1
reais tais que a, b, c > 0 e a + b + c = 1, então a2 + b2 + c2 
1/3.
37
(Seleção Cone Sul Brasil-94) Sejam a, b, c, d números reais 46
1 + 1 + 4 + 16  64 (Leningrado-88) Se os números reais x e y são tais que 0 
positivos. Prove que .
a b c c a+b+c+d x + y 1.
x  1 e 0  y  1, prove que
y +1 x +1
38
(Chile-94) Considere 10 números inteiros positivos, não 47
necessariamente distintos, que somem 95. Encontre o (Leningrado-90) Se A, B e C são números reais tais que 0 
menor valor possível da soma de seus quadrados. A, B, C  1, prove que:
A + B + C 2
39
1 + BC 1 + AC 1 + AB
(Canadá-69) Seja c o comprimento da hipotenusa de um
triângulo retângulo cujos dois lados possuem comprimento a
48
e b. Prove que a + b  2c . Quando ocorre a igualdade? (Leningrado-91) Os números reais x1, x2, …, xn pertencem ao
intervalo [– 1, 1], e a soma dos seus cubos é igual a zero.
40 Prove que a soma
(USAMO-78) Dado que a, b, c, d, e são números reais tais x1 + x2 + … + + xn não excede n/3.
que
a + b + c + d + e = 8, a2 + b2 + c2 + d2 + e2 = 16. 49
Determine o máximo valor de e. (APMO-89) Seja x1, x2, ..., xn números reais positivos, e seja
S = x1 + x2 + ... + xn.
41 Prove que
(Noruega-95) Seja n um número natural, e sejam x1, x2, …, 2 3 n

xn, y1, y2, …, yn > 0. Prove que


(1 + x 1 )(1 + x 2 )...(1 + x n )  1 + S + S + S + ... + S
2! 3! n!
((x 1 )
+ y1 ) 2 + ( x 2 + y 2 ) 2 + ... + ( x n + y n ) 2 .
50
 
. 1 + 1 + ... + 1   4n 2 (Cone Sul-banco) Demonstrar que se x, y, z são números
 x 1 y1 x 2 y 2 x n yn 
reais positivos, então:

42 x + y + z 3.
(Irlanda-98) Prove que se a, b, c são números reais
y+z x+z x+y 2
positivos, então: 51
(IMO-78) {ak} é uma seqüência de inteiros positivos
(i)
9  2 1 + 1 + 1  distintos. Prove que para todos os inteiros positivos n,
a+b+c a+b b+c c+a
a1 a 2 a 3 a
(ii)
1 + 1 + 1  1  1 + 1 + 1  2
+ 2 + 2 + ... + n2  1 + 1 + 1 + ... + 1
a +b b+c c+a 2a b c 1 2 3 n 2 3 n

43
(Irlanda-99) Sejam a, b, c e d números reais positivos cuja
soma é 1. Prove que:

MATEMÁTICA – Prof. DOUGLAS OLIVEIRA biblioteca


telegram t.me/bibliotecaPE @PROFDOUGLASOLIVEIRA _2020 83
APROVADO DE A A Z
TURMA IME ITA

MATEMÁTICA – Prof. DOUGLAS OLIVEIRA biblioteca


telegram t.me/bibliotecaPE @PROFDOUGLASOLIVEIRA _2020 84

Você também pode gostar